Top 900 Complete Physics MCQs - Watermark

You might also like

Download as pdf or txt
Download as pdf or txt
You are on page 1of 462

Top 900 Complete Physics MCQs

1. A body of mass is moving in a circle of radius with an angular velocity


of . The centripetal force is

(1)
(2)
(3)
(4)

Correct Answer : (2)


Explanation : Centripetal force

2. The acceleration of electron in the first orbit of hydrogen atom is

(1)

(2)

(3)

(4)

Correct Answer : (3)

Explanation : From

Acceleration,

3. The equivalent capacitance of three capacitors of capacitance and are connected in


parallel is 12 units and product unit. When the capacitors and are
connected in parallel, the equivalent capacitance is 6 units. Then the capacitances are

(1) 2, 3, 7
(2) 1, 5, 2, 5, 8
(3) 1, 5, 6
(4) 4, 2, 6

Correct Answer : (4)


Explanation : …(i)
…(ii)
…(iii)
From equation (i) and (iii)
…(iv)
From equation (ii) and (iv)
Also

…(v)
On solving (iii) and (v)

4. When cathode-rays strike a metal target of high melting point with a very high velocity, then
which of the following are produced

(1) -rays
(2) -rays
(3) Ultraviolet rays
(4) -waves

Correct Answer : (2)

5. Chromatic aberration of lens can be corrected by

(1) Reducing its aperature

(2) Proper polishing of its two surfaces


(3) Suitably combining it with another lens

(4) Providing different suitable curvature to its two surfaces

Correct Answer : (3)

6. A copper voltmeter is connected in series with a heater coil of resistance A steady


current flows in the circuit for twenty minutes and mass of of copper is deposited at the
cathode. If electrochemical equivalent of copper is then heat generated in the
coil is

(1)
(2)
(3)
(4)

Correct Answer : (1)

Explanation :
Hence heat generated in the coil is

7. A block of mass 15 kg is resting on a rough inclined plane as shown in figure. The block is tied
by a horizontal string which has a tension of 50 N. The coefficient of friction between the
surfaces of contact is ( )

(1) 1/2
(2) 3/4
(3) 2/3
(4) 1/4
Correct Answer : (1)
Explanation : Figure shows free body diagram of the block

For equilibrium, along the place

…(i)
For equilibrium, in direction perpendicular to inclined plane,

Put in Eq. (i),


(50+15 ) = (15 )

8. The ratio of rotational and translatory kinetic energies of a sphere is

(1)

(2)

(3)

(4)

Correct Answer : (3)


Explanation :

9. In which orientation the resultant magnetic moment of two magnets, will be zero, if magnetic
moment of each magnets is in the following figures?

(1)

(2)

(3)

(4)

Correct Answer : (2)


Explanation : . As magnetic moments are in a closed loop in Fig. (

In Fig. (c)
In Fig. (d)

10. In the circuit shown below, the ac source has voltage volts
with . The amplitude of the current will be nearest to
(1) First clockwise then anticlockwise

(2) In clockwise direction

(3) In anticlockwise direction

(4) First anticlockwise then clockwise

Correct Answer : (3)


Explanation : A metallic ring is attached with the wall of a room. When the north pole of a
magnet is brought near to it, the induced current in the ring will be

11. An elevator car, whose floor to ceiling distance is equal to 2.7 starts ascending with
constant acceleration of 2 after the start, a bolt begins falling from the ceiling of
the car. The free fall time of the bolt is

(1)
(2)
(3)
(4)

Correct Answer : (3)

Explanation :
As and lift is moving upward with acceleration
12. Two putty balls of equal mass moving with equal velocity in mutually perpendicular
directions, stick together after collision. If the balls were initially moving with a velocity
of each, the velocity of their combined after collision is

(1)
(2)
(3)
(4)

Correct Answer : (2)


Explanation : Initial momentum

Final momentum
By conservation of momentum

13. A ray of light passes an equilateral prism such that an angle of incidence is equal to the

angle of emergence and the latter is equal to th the angle of prism. The angle of deviation is

(1)
(2)
(3)
(4)

Correct Answer : (4)


Explanation : Angle of incidence = angle of emergence,
Ie,
Also, angle of equilateral prism

=
Thus, angle of deviation
=
=(45 +45 -60 )=30

14. A transparent cube of 2.1 m edge contains a small air bubble. Its apparent distance when
viewed through one face of the cube is 0.10 m and when viewed from the opposite face is 0.04
m. The actual distance of the bubble from the second face of the cube is

(1) 0.06 m
(2) 0.17 m
(3) 0.05 m
(4) 0.04 m

Correct Answer : (1)

Explanation :

15. The current flowing in a step down transformer 220 V to 22 V having impedance 220 is

(1) 1 A
(2) 0.1 A
(3) 2 mA
(4) 0.1 mA

Correct Answer : (2)

Explanation : A
16. The magnetic susceptibility does not depend upon the temperature in

(1) Ferrite substances


(2) Ferromagnetic substances

(3) Diamagnetic substances


(4) Paramagnetic substances

Correct Answer : (3)

17. In isothermic process, which statement is wrong

(1) Temperature is constant


(2) Internal energy is constant

(3) No exchange of energy


(4) [a] and [b] are correct

Correct Answer : (3)


Explanation : In isothermal process, exchange of energy takes place between system and
surrounding to maintain the system temperature constant

18. (1) Centre of gravity (C. G.) of a body is the point at which the weight of the body acts
(2) Centre of mass coincides with the centre of gravity if the earth is assumed to have infinitely
large radius
(3) To evaluate the gravitational field intensity due to any body at an external point, the centre
mass of the body can be considered to be concentrated at its C.G.
(4) The radius of gyration of any body rotating about an axis is the length of the perpendicular
dropped from the C.G. of the body to the axis
Which one of the following pairs of statements is correct

(1) (4) and (1)


(2) (1) and (2)
(3) (2) and (3)
(4) (3) and (4)

Correct Answer : (1)


19. A beam of protons is moving parallel to a beam of electrons. Both the beams will tend to

(1) Repel each other


(2) Come closer
(3) Move more apart
(4) Either [b] or [c]

Correct Answer : (3)


Explanation : Current corresponding to the beams of protons and electrons are in opposite
direction. Therefore, both will experience a force of repultion and therefore move more apart.

20. A wire has length 1 m and cross-sectional area 1 The work


required to increase the length by 2 mm is

(1) 0.4 J
(2) 4 J
(3) 40 J
(4) 400 J

Correct Answer : (1)

Explanation : Work done =

=4

21. The distance between a proton and electron both having a charge of a
hydrogen atom is The value of intensity of electric field produced on electron due
to proton will be

(1)
(2)
(3)
(4)

Correct Answer : (4)

Explanation :

22. When a hydrogen atom is bombared, the atom is excited to then state. The energy
released, when the atom goes from state to the ground state is

(1) 1.275 eV
(2) 12.75 eV
(3) 5 eV
(4) 8 eV

Correct Answer : (2)


Explanation : Energy released

23. A mass is suspended from the two coupled springs connected in series. The force
constant for springs are and The time period of the suspended mass will be

(1)

(2)

(3)
(4)

Correct Answer : (3)

Explanation : In series so time period

24. A capacitor having a capacity of 2 pF. Electric field across the capacitor is changing with a
value of . The displacement current is

(1) 2 A
(2) 4 A
(3) 6 A
(4) 10 A

Correct Answer : (1)

Explanation :

25. The following figure shows a logic gate circuit with two inputs A and B and the output C. The
voltage waveforms of A, B and C are as shown below

The logic circuit gate is

(1) AND gate


(2) NAND gate
(3) NOR gate
(4) OR gate

Correct Answer : (1)


Explanation : From truth table it is clear that output is high if at least one input is low. The
Boolean expression which satisfies the output of this logic gate is which is for AND
gate.

26. A battery having e.m.f. and internal resistance is connected with a resistance
of then the voltage at the terminals of battery is

(1)
(2)
(3)
(4)

Correct Answer : (1)

Explanation :

27. Forces of 100 N each are applied in opposite directions on the upper and lower faces of a
cube of side 20 cm. The upper face is shifted parallel to itself by 0.25 cm. If the side of the cube
were 10 cm, then the displacement would be

(1) 0.25 cm
(2) 0.5 cm
(3) 0.75 cm
(4) 1 cm

Correct Answer : (2)

Explanation :
If is halved, then is doubled.
28. A force of , making an angle with the horizontal, acting on an object displaces it
by along the horizontal direction. If the object gains kinetic energy of , the horizontal
component of the force is

(1)
(2)
(3)
(4)

Correct Answer : (2)


Explanation : Work done on the body = K.E. gained by the body

29. A gramophone record is revolving with an angular velocity . A coin is placed at a


distance from the centre of the record. The static coefficient of friction is . The coin will
revolve with the record if

(1)
(2)

(3)

(4)

Correct Answer : (4)


Explanation : Here friction force provides centripetal force so
but

So
30. A body is thrown with a velocity of making an angle of with the horizontal. It will
hit the ground after a time

(1)
(2)
(3)
(4)

Correct Answer : (2)

Explanation :

31. Select the incorrect statements from the following


S1 : Average velocity is path length divided by time interval
S2 : In general, speed is greater than the magnitude of the velocity
S3 : A particle moving in a given direction with a non-zero velocity can have zero speed
S4 : The magnitude of average velocity is the average speed

(1) S2 and S3
(2) S1 and S4
(3) S1, S3 and S4
(4) All four statements

Correct Answer : (3)

Explanation : Average velocity


A particle moving in a given direction with non-zero velocity cannot have zero speed.
In general, average speed is not equal to magnitude of average velocity. However, it can be so if
the motion is along a straight line without change in direction

32. The effective capacitance between points is


(1) 9
(2) 3
(3) 6
(4) 1

Correct Answer : (1)

Explanation :
Here, all the three capacitors are connected in parallel. Therefore, equivalent capacitance
between points is
.

33. The curve drawn between velocity and frequency of a photon in vacuum will be

(1) Straight line parallel to frequency axis


(2) Straight line parallel to velocity axis

(3) Straight line passing through origin and making an angle of with frequency axis
(4) Hyperbola

Correct Answer : (2)


Explanation : Velocity of photon in vacuum is constant for all frequencies

34. Which physical quantities have the same dimension

(1) Couple of force and work


(2) Force and power

(3) Latent heat and specific heat


(4) Work and power

Correct Answer : (1)


Explanation : Couple of force =
Work

35. When a ceiling fan is switched off, its angular velocity falls to half while it makes 36
rotations. How many rotations will it make before coming to rest?

(1) 24
(2) 36
(3) 18
(4) 12

Correct Answer : (4)


Explanation : From third equation of angular motion,

or

or

or …(i)
According to question again applying the third equation of angular motion

or
But cycle
So,
36. If energy, gravitational constant, impulse and mass, the dimensions

of are same as that of

(1) Time
(2) Mass
(3) Length
(4) Force

Correct Answer : (1)


Explanation :

37. A man does a given amount of work in 10 s. Another man does the same amount of work in
20 s. The ratio of the output power of first man to the second man is

(1) 1

(2)

(3)
(4) None of these

Correct Answer : (3)


Explanation : Given ,

power

or

38. The proton is energy 1 MeV describes a circular path in plane at right angles to a uniform
magnetic field of The mass of the proton is The cyclotron
frequency of the proton is very nearly equal to

(1)
(2)
(3)
(4)

Correct Answer : (4)


Explanation : Cyclotron frequency is given by

39. The frequency for which a capacitor has a reactance of is given by

(1)

(2)

(3)
(4)

Correct Answer : (1)

Explanation :

40. Ionization potential of hydrogen atom is 13.6 eV. Hydrogen atoms in the ground state are
exicted by monochromatic radiation of photon energy 12.1 eV. The spectral lines emitted by
hydrogen atom according to Bohr’s theory will be
(1) One
(2) Two
(3) Three
(4) Four

Correct Answer : (3)


Explanation : Total energy of electron in excited state which
corresponds to third orbit. The possible spectral lines are when electron jumps from orbit 3rd
to 2nd; 3rd to 1st and 2nd to 1st

41. The truth table shown in figure is for

A0011

B 0101

Y1001

(1) XOR
(2) AND
(3) XNOR
(4) OR

Correct Answer : (3)


Explanation : For ‘XNOR’ gate

42. In thermal equilibrium, the average velocity of gas molecules is


(1) Proportional to
(2) Proportional to
(3) Proportional to
(4) Zero

Correct Answer : (1)

Explanation :

43. Keeping dissimilar poles of two magnets of equal pole strength and length same side, their
time period will be

(1) Zero
(2) One second
(3) Infinity
(4) Any value

Correct Answer : (3)

Explanation :
Here

44. The rms value of the electric field of the light coming from the sun is 720 NC-1. The average
total energy density of the Electromagnetic Wave is

(1)
(2)
(3)
(4)

Correct Answer : (1)


Explanation : Total average energy

45. A square of side has charge at its centre and charge q at one of the corners. The work
required to be done in moving the charge q from the corner to the diagonally opposite corner is

(1) Zero

(2)

(3)

(4)

Correct Answer : (1)


Explanation : The potential due to charge at distance is given by

If be the work done in moving the charge from then the potential difference is
given by

Both work (W) and charge (q) are scalar quantities hence potential difference will also
be a scalar quantity.
Here,

Since, is same for both,


46. In an L-C-R series AC circuit the voltage across L, C and R is 10 V each. If the inductor is short
circuited, the voltage across the capacitor would become

(1) 10 V

(2)
(3)

(4)

Correct Answer : (4)


Explanation : Circuit is resonant.
Hence supply voltage equals

Also,
As the voltage drops are equals across them when L is shortened

or

47. Kinetic energy of emitted cathode rays is dependent on

(1) Only voltage


(2) Only work function

(3) Both [a] and [b]


(4) It does not depend upon any physical quantity

Correct Answer : (3)


Explanation : Higher the voltage, higher is the Higher the work function, smaller is the

48. The acceleration of a body due to the attraction of the earth (radius ) at a distance from
the surface of the earth is ( = acceleration due to gravity at the surface of the earth)

(1)

(2)

(3)
(4)

Correct Answer : (1)

Explanation :

49. A volume and pressure diagram was obtained from state 1 to state 2 when a given
mass of a gas is subjected to temperature changes. During this process the gas is

(1) Heated continuously


(2) Cooled continuously

(3) Heated in the beginning and cooled towards the end


(4) Cooled in the beginning and heated towards the end

Correct Answer : (3)


Explanation :
Draw two isothermals one passing through points 1 and 2 the other through mid point of
straight line joining 1 and 2
at point 1 temperature is and that at mid point is and then at point 2 again it is
The gas is first heated and then cooled towards end

50. Energy conversion in a photoelectric cell takes place form

(1) Chemical to electrical


(2) Magnetic to electrical

(3) Optical to electrical


(4) Mechanical to electrical

Correct Answer : (3)

51. Two parallel infinite line charges and are placed with a separation distance in free
space. The net electric field exactly mid-way between the two line charges is

(1) Zero

(2)

(3)

(4)

Correct Answer : (2)

52. Two bodies of identical mass are moving with constant velocity but in the opposite
directions and stick to each other, the velocity of the compound body after collision is
(1)
(2)
(3) Zero

(4)

Correct Answer : (3)


Explanation :

53. Water falls from a height of 500 m. What is the rise in temperature of water at the bottom if
whole energy is used up in heating water?

(1) 0.96
(2) 1.02
(3) 1.16
(4) 0.23

Correct Answer : (3)

Explanation : Heat absorbed by water = Heat produced

54. An automobile weighing 1200 kg climbs up a hill that rises 1 m in 20 s. Neglecting frictional
effects. The minimum power developed by the engine is 9000 W. If , then the
velocity of the automobile is

(1)
(2)
(3)
(4)

Correct Answer : (2)


Explanation : Minimum force so, minimum power is given by

or

or

55. A wire in the form of a circular loop of one turn carrying a current produces a magnetic
field at the centre. If the same wire is looped into a coil of two turns and carries the same
current, the new value of magnetic induction at the centre is

(1)
(2)
(3)
(4)

Correct Answer : (3)


Explanation : Magnetic field at the centre of a current carrying loop is given by

Here, no. of turns in loop

For

…(i)

When turns and radius

or …(ii)
Now, from Eqs. (i) and (ii)

Hence,
56. In the following diagram the work done in moving a point charge from point to
point and is respectively as and then

(1)
(2)
(3)
(4)

Correct Answer : (2)


Explanation : According to the figure, there is no other charge. A single charge when moved in
a space of no field, does not experience any force. No work is done

57. The height of a TV antenna is 200 m. The population density is . Find the
population benefited

(1)
(2)
(3)
(4)

Correct Answer : (2)


Explanation : ;
Population converted population density

58. When a spring is stretched by a distance , it exerts a force, given by .


The work done, when the spring is stretched from to is
(1)
(2)
(3)
(4)

Correct Answer : (1)


Explanation :

Work done,

59. On which principle does sonometer works?

(1) Hooke’s law


(2) Elasticity
(3) Resonance
(4) Newton’s law

Correct Answer : (3)


Explanation : Sonometer works on the principle of resonance. At resonance the wire of
sonometer vibrate with maximum amplitude.

60. The pendulum bob has a speed of at its lowest position. The pendulum is 0.5 m
long. The speed of the bob, when the length makes an angle of to the vertical will
be

(1)
(2)
(3)
(4)

Correct Answer : (4)

Explanation : KE at the lowest position =

When the length makes an angle to the vertical, the bob of the pendulum will have
both KE and PE. If is the velocity of bob at this position and is the height of the bob w.r.t. ,
then total energy of the bob

But

According to law of conservation of energy


61. A 100 turns coil shown in figure carries a current of in a magnetic
field . The torque acting on the coil is

(1) 0.32 tending to rotated the side out of the page

(2) 0.32 tending to rotated the side into the page

(3) 0.0032 tending to rotated the side out of the page

(4) 0.0032 tending to rotated the side into the page

Correct Answer : (1)


Explanation :
Direction can be found by Fleming’s left hand rule

62. An n-p-n transistor power amplifier in CE configuration gives

(1) Voltage amplification only

(2) Currents amplification only

(3) Both current and voltage amplifications

(4) Only power gain of unity

Correct Answer : (3)

63. A particle moving in a magnetic field increases its velocity then its radius of the circle

(1) Decreases
(2) Increases
(3) Remains the same
(4) Becomes half

Correct Answer : (2)


Explanation : For motion of a charged particle in a magnetic field, we have

64. Dimensions of charge are

(1)
(2)
(3)
(4)

Correct Answer : (4)


Explanation : Charge

65. A particle of mass moving with velocity makes an elastic one dimensional collision with
a stationary particle of mass . They are in contact for a very short time . Their force of
interaction increases from zero to linearly in time , and decreases linearly to zero in
further time . The magnitude of is

(1)
(2)
(3)
(4) None of these

Correct Answer : (2)


Explanation : In one dimensional elastic collision when two particles of same mass colloid with
each other their velocity are interchange
change in momentum of particle
But impulse = change in momentum

66. In a stationary wave all the particles

(1) On either side of a node vibrate in same phase

(2) In the region between two nodes vibrate in same phase

(3) In the region between two antinodes vibrate in same phase

(4) Of the medium vibrate in same phase

Correct Answer : (2)


Explanation : In stationary wave all the particles in one particular segment ( between two
nodes) vibrates in the same phase

67. The decrease in the potential energy of a ball of mass which falls from a height
of is

(1)
(2)
(3)
(4) None of these

Correct Answer : (2)


Explanation :
68. A wavelength is produced in air and it travels at a speed of It will be an

(1) Audible wave


(2) Infrasonic wave
(3) Ultrasonic wave
(4) None of the above

Correct Answer : (3)

Explanation :
Wave is ultrasonic

69. Magnification at least distance of distinct vision of a simple microscope having its focal
length 5 cm is

(1) 2
(2) 4
(3) 5
(4) 6

Correct Answer : (4)


Explanation : If final image is formed at least distance of distinct vision, magnification

Or magnification

70. When a certain circuit consisting of a constant e.m.f. an inductance and a


resistance is closed, the current in it increases with time according to curve 1
After one parameter is changed, the increase in current follows curve
2 when the circuit is closed second time. Which parameter was changed and in
what direction
(1) is increased
(2) is decreased
(3) is increased
(4) is decreased

Correct Answer : (1)

Explanation : slope of graph; slope of graph (2) < slope of graph (1)

so Also

71. A plumb line is suspended from a ceiling of a car moving with horizontal acceleration of .
What will be the angle of inclination with vertical?

(1)

(2)

(3)

(4)

Correct Answer : (1)


Explanation : Let the angle from the vertical be . The diagram showing the different forces is
given
72. Which one of the following statements is true for the speed and the acceleration of a
particle executing simple harmonic motion

(1) When is maximum, is maximum


(2) Value of is zero, whatever may be the value of

(3) When is zero, is zero


(4) When is maximum, is zero

Correct Answer : (4)


Explanation : In S.H.M. and when
and

73. A coil of and area is placed in a magnetic field .


The normal to the plane of the coil makes an angle of with the direction of the magnetic
field. The magnetic flux linked with the coil is

(1)
(2)
(3)
(4)

Correct Answer : (1)


Explanation :

74. For a particle in uniform circular motion the acceleration a at a point on the circle of
the radius R is (here is measured from the axis)

(1)

(2)

(3)

(4)

Correct Answer : (3)


Explanation : For a particle in uniform circular motion
75. A long, straight wire is turned into a loop of radius (see figure). If a current of 8
amperes is passed through the lop, then the value of the magnetic field and its direction as the
centre of the loop shall be close to

(1) Newton/(amp-meter), upward

(2) Newton/(amp-meter), upward

(3) Newton/(amp-meter), downward

(4) Newton/(amp-meter), upward

Correct Answer : (2)


Explanation : at the centre of a coil carrying a current, is

[upward]

due to wire [downward]


Given

Magnetic field at centre ,

(upwar [downward]
76. The phase difference between two points separated by 0.8 m in a wave of frequency 120 Hz
is . The wave velocity is

(1)
(2)
(3)
(4)

Correct Answer : (2)


Explanation : Here ,
.

From

77. How much heat energy is gained when of water at is brought to its boiling point
(specific heat of water )

(1)
(2)
(3)
(4)

Correct Answer : (1)


Explanation :

78. If a body loses half of its velocity on penetrating 3 cm in a wooden block, then how much
will it penetrate more before coming to rest?

(1) 1 cm
(2) 2 cm
(3) 3 cm
(4) 4 cm

Correct Answer : (3)


Explanation : Let initial velocity of body at point is is 3 cm.

From

Let on penetrating 3 cm in a wooden block, the body moves distance form


So, for

79. One mole of an ideal gas having initial volumeV, pressure 2p and temperature T undergoes
a cyclic process ABCDA as shown below.

The net work done in the complete cycle is

(1) Zero

(2)
(3) RT In 2
(4)

Correct Answer : (3)


Explanation : Work done
At constant pressure

At constant temperature

Net work done in the complete cycle is

80. A hot liquids is filled in a container and kept in a room of temperature of . The liquid
emits heat at the rate of . When its temp. is . When the temperature of the liquid
becomes , the rate of heat loss in is

(1) 160
(2) 140
(3) 80
(4) 60

Correct Answer : (4)

81. Two spherical vessel of equal volume, are connected by a narrow tube. The apparatus
contains an ideal gas at one atmosphere and Now if one vessel is immersed in a bath of
constant temperature and the other in a bath of constant temperature Then the
common pressure will be

(1)

(2)

(3)

(4)

Correct Answer : (3)


Explanation :

82. The amplitude of a damped oscillator becomes rd in 2 s. If its amplitude after 6 s

is times the original amplitude, the value of is

(1)
(2)
(3)
(4)

Correct Answer : (4)


Explanation : We have
In this case after 6 s amplitude becames times.

83. Solar energy is mainly cause due to

(1) Fission of uranium present in the sun

(2) Fusion of protons during synthesis of heavier elements

(3) Gravitational contraction

(4) Burning of hydrogen in the oxygen

Correct Answer : (2)

84. A body starts from rest, with uniform acceleration. If its velocity after seconds is then its
displacement in the last two seconds is

(1)

(2)

(3)

(4)

Correct Answer : (4)


Explanation :

Now, distance travelled in . and

distance travelled in

=
85. If the earth did not have atmosphere, its surface temperature on a day time would be

(1) Higher
(2) Lower
(3) Same as now
(4) Not sure

Correct Answer : (1)

86. Two boys of masses 10 kg and 8 kg are moving along a vertical rope, the former climbing up
with acceleration of 2 . 2 while later coming down with uniform velocity of 2 . Then
tension in rope at fixed support will be(Take )

(1) 200 N
(2) 120 N
(3) 180 N
(4) 160 N

Correct Answer : (2)


Explanation : Since, moves with constant velocity

N
Since, boy of mass moves with acceleration in upward direction
N

87. The waves relevant to telecommunications are

(1) Visible light


(2) Infrared
(3) Ultraviolet
(4) Microwave

Correct Answer : (4)


Explanation : The critical frequency of sky wave undergoing reflection from a layer of
ionosphere is

Where is electron density per

88. Which is having minimum wavelength?

(1) X-rays
(2) Ultraviolet rays
(3) -rays
(4) Cosmic rays

Correct Answer : (3)

Explanation : minimum the wavelength, the maximum the energy of a ray. Therefore
rays have minimum wave length
89. A geo-synchronous satellite is

(1) Located at a height of 34860 km to ensure global coverage

(2) appears stationary over a place on earth's magnetic pole

(3) not really stationary, but orbits the earth within 24 h

(4) Always at a fixed location in state and simply spins about its own axis

Correct Answer : (3)

90. The masses of and respectively are connected by a massless spring as shown in
figure. A force of acts on the mass. At the instant shown, the mass has
acceleration . What is the acceleration of mass

(1)
(2)
(3)
(4) Zero

Correct Answer : (2)


Explanation : As the mass of has acceleration therefore it apply force on
mass in a backward direction
Net forward force on mass

Acceleration

91. The following four wires of length L and radius r are made of the same material. Which of
these will have the largest extension, when the same tension is applied?
(1) L = 400 cm, r = 0.8 mm
(2) L = 300 cm, r = 0.6 mm

(3) L = 200 cm, r = 0.4 mm


(4) L = 100 cm, r = 0.2 mm

Correct Answer : (4)

Explanation :
Option (has the largest extension when the same tension is applied.

92. The maximum distance upto which TV transmission from a TV tower of height can be
received is proportional to

(1)
(2)
(3)
(4)

Correct Answer : (1)


Explanation :

93. The ratio of the radii of gyration of a circular disc to that of a circular ring, each of same
mass and radius, around their respective axes is

(1)
(2)
(3)
(4)
Correct Answer : (4)

Explanation : Radius of gyration of circular disc


Radius of gyration of circular ring

Ratio

94. The average value of electric energy density in an Electromagnetic Waves is (E0 is peak value)

(1)

(2)
(3)

(4)

Correct Answer : (4)


Explanation : Electric energy density

95. Capacitance of a capacitor made by a thin metal foil is F. If the foil is folded with paper of
thickness 0.15 mm, dielectric constant of paper is 2.5 and width of paper is 400 mm, the length
of foil will be

(1) 0.34 m
(2) 1.33 m
(3) 13.4 m
(4) 33.9 m
Correct Answer : (4)
Explanation : If length of the foil is them

96. A current carrying circular wire of radius is placed in a magnetic field perpendicular
to its plane. The tension along the circumference of wire is

(1)
(2)
(3)
(4)

Correct Answer : (1)


Explanation : For small element proton

97. In satellite communication, the communication satellite;


(1) Acts as a reflector for a beam of modulated microwave from transmitter sent directly
towards it

(2) Acts as a repeater for a signal reaching there, without any change in frequency

(3) Receives the coming modulated microwave signal, amplifies it and returns it to earth at a
different frequency

(4) None of the above

Correct Answer : (3)


Explanation : A communication satellite receives the coming modulated microwave signal,
amplifies it and return it to earth at a different frequency

98. Supposing the distance between the atoms of a diatomic gas to be constant, its specific heat
at constant volume per mole (gram mole) is

(1)

(2)
(3)

(4)

Correct Answer : (1)

Explanation :
For diatomic gas

99. Figure shows three spherical and equipotential surfaces A, B and C round a point charge q.
The potential difference . If and be the distance between them. Then
(1)
(2)
(3)
(4)

Correct Answer : (3)


Explanation : Potential difference between two equipotential surfaces A and B.

Or

Or
Similarly,
Since,
therefore

100. The specific heat of an ideal gas is

(1) Proportional to
(2) Proportional to
(3) Proportional to
(4) Independent of
Correct Answer : (4)
Explanation : According to the equilibrium theorem, the molar heat capacities should be
independent of temperature. However, variations in and are observed as the
temperature changes. At very high temperatures, vibrations are also important and that affects
the values of and for diatomic and polyatomic gases. Here in this question according to
given information (may be correct answer

101. It is seen that in proper ventilation of building, windows must be opened near the bottom
and the top of the walls, so as to let pass

(1) In hot near the roof and cool air out near the bottom
(2) Out hot air near the roof

(3) In cool air near the bottom and hot air our near the roof
(4) In more air

Correct Answer : (3)


Explanation : Moist and hot air being lighter rises up and leaves the room throught the
ventilator near the roof and fresh air rushes into the room throught the doors.

102. A double convex lens, lens made of a material of refractive index , is placed inside two
liquids or refractive indices and , as shown. . A wide, parallel beam of light is
incident on the lens from the left. The lens will give rise to

(1) A single convergent beam


(2) Two different convergent beams

(3) Two different divergent beams


(4) A convergent and a divergent beam

Correct Answer : (4)


Explanation : As the upper half of the lens will become diverging
As , the lower half of the lens will become converging

103. 100 g of water is heated from Ignoring the slight expansion of the water, the
change in its internal energy is
(Specific heat of water is 4184 J/kg/K)

(1) 8.4 kJ
(2) 84 kJ
(3) 2.1 kJ
(4) 4.2 kJ

Correct Answer : (1)


Explanation : As work done=0

=84 kJ

104. Pressure versus temperature graph of an ideal gas at constant volume of an ideal gas is
shown by the straight line Now mass of the gas is doubled and the volume is halved, then
the corresponding pressure versus temperature graph will be shown by the line

(1)
(2)
(3)
(4) None of these

Correct Answer : (2)


Explanation :
So, at constant volume pressure-versus temperature graph is a straight line passing through

origin with slope As the mass is doubled and volume is halved slope becomes four times.
Therefore, pressure versus temperature graph will be shown by the line

105. A radar sends the waves towards a distant object and receives the signal reflected by
object. These waves are

(1) Sound waves


(2) Light waves
(3) Radio waves
(4) Micro waves

Correct Answer : (4)


Explanation : Now a days microwaves are used to locate the flying objects by radar

106. Velocity of Electromagnetic Waves in a medium depends upon

(1) Thermal properties of medium

(2) Mechanical and electrical properties of medium

(3) electrical and magnetic properties of the medium

(4) Mechanical and magnetic properties of the medium

Correct Answer : (3)

107. The valency of the impurity atom that is to be added to germanium crystal so as to make it
a -type semiconductor, is

(1) 6
(2) 5
(3) 4
(4) 3

Correct Answer : (2)

108. If orbital velocity of planet is given by , then

(1)
(2)

(3)
(4)

Correct Answer : (2)

Explanation :

109. What is the angle of incidence for an equilateral prism of refractive index so that the ray
is parallel to the base inside the prism?

(1)
(2)
(3)
(4) Either

Correct Answer : (3)

Explanation :
Or

Or

110. When an aeroplane attains a speed higher than the velocity of sound in air, a loud bang is
heard. This is because

(1) It explodes

(2) It produces a shock wave which is received as the bang

(3) Its wings vibrate so violently that the bang is heard

(4) The normal engine noises undergo a Doppler shift to generate the bang

Correct Answer : (2)

111. A body of mass 10 kg is moving on a horizontal surface by applying a force of 10 N in


forward direction. If body moves with constant velocity, the work done by force of fiction for a
displacement of 2m is

(1)
(2) 10 J
(3) 20 J
(4)

Correct Answer : (1)


Explanation : Since body moves with constant velocity, so. Net force on the body is zero.
Here,

112. Hysteresis loss is minimized by using

(1) Alloy of steel


(2) Shell type of core

(3) Thick wire which has low resistance


(4) Mu metal

Correct Answer : (4)


Explanation : Hysteresis loss is minimised by using Mu metal.

113. The half life of a radioactive isotope is 50 years. It decays to another element which is
stable. The two elements and were found to be in the ratio of 1:16 in a sample of a given
rock. The age of the rock was estimated to be

(1) 100 years


(2) 150 years
(3) 200 years
(4) 250 years

Correct Answer : (3)

Explanation : After second fractional amount of left is or

114. A particle undergoes uniform circular motion. About which point on the plane of the circle,
will the angular momentum of the particle remain conserved?

(1) center of the circle


(2) on the circumference of the circle

(3) inside the circle


(4) outside the circle

Correct Answer : (1)


Explanation : In uniform circular motion the only force acting on the particle is centripetal
(towards centr. Torque of this force about the centre is zero. Hence angular momentum about
centre remains conserved.
115. When two ends of a rod wrapped with cotton are maintained at different temperatures
and after same time every point of the rod attains a constant temperature, then

(1) Conduction of heat at different points of the rod stops because the temperature is not
increasing

(2) Rod is bad conductor of heat

(3) Heat is being radiated from each point of the rod

(4) Each point of the rod is giving heat to its neighbour at the same rate at which it is receiving
heat

Correct Answer : (4)


Explanation : Because steady state has been reached

116. An engine pumps water through a hose pipe. Water passes through the pipe and leaves it
with a velocity of 2 m/s. The mass per unit length of water in the pipe is 100 kg/m. What is the
power of the engine

(1)
(2)
(3)
(4)

Correct Answer : (1)

Explanation : Power

117. A wire of area of cross-section is increased in length by 0.1%. The tension


produced is . The Young’s modulus of wire is
(1)
(2)
(3)
(4)

Correct Answer : (1)

Explanation :

118. A metallic ring of radius r and cross-sectional area A is fitted into a wooden circular disc of
radius R(R>r). If the Young’s modulus of the material of the ring is Y , the force with which the
metal ring expands is

(1)

(2)

(3)

(4)

Correct Answer : (2)

Explanation : Young’s modulus Y

Force F

119. At the centre of a circular coil of radius 5 cm carrying current, magnetic field due to earth
is What should be the current flowing through the coil so that it annuals the
earth’s magnetic field?
(1) 40 A
(2) 4 A
(3) 0.4 A
(4) 0.2 A

Correct Answer : (3)


Explanation : Magnetic field at the centre of a coil of radius carrying current is

Given,

120. Imagine a light plant revolving around a very massive star in circular orbit of radius with a
period of revolution . If the gravitational force of attraction between the planet and the star is
proportional to . Then the correct relation is

(1)
(2)
(3)
(4)

Correct Answer : (2)


Explanation : Since, gravitation provides centripetal force

So that

121. In the given network, the value of , so that an equivalent capacitance


between and is
(1)
(2)

(3)

(4)

Correct Answer : (2)

Explanation :
or

122. Which of the following gates will have an output of 1

(1)

(2)

(3)

(4)

Correct Answer : (3)


Explanation : For ‘NAND’ gate (option , output

123. A convex lens of focal length 30 cm produces 5 times magnified real image of an object.
What is the object distance?

(1) 36 cm
(2) 25 cm
(3) 30 cm
(4) 150 cm

Correct Answer : (1)

Explanation :

124. Light of wavelength 4000 is incident on a metal plate whose work function is 2 eV. The
maximum KE of the emitted photoelectron would be

(1) 0.5 eV
(2) 1.1 eV
(3) 1.5 eV
(4) 2.0 eV

Correct Answer : (2)

Explanation : Maximum KE
125. A satellite of mass is placed at a distance from the centre of earth (mass . The
mechanical energy of the satellite is

(1)

(2)

(3)

(4)

Correct Answer : (4)

126. Calculate the work done, if a wire is loaded by weight and the increase in length is

(1)
(2) Zero
(3)
(4)

Correct Answer : (3)

Explanation : Work done

127. A particle of mass is executing oscillations about the origin on the -axis. Its potential
energy is where is a positive constant. If the amplitude of oscillation is then its
time period is

(1) Proportional to
(2) Independent to
(3) Proportional to
(4) Proportional to
Correct Answer : (1)

Explanation : …(i)

Also, for SHM and

acceleration

…(ii)

From equation (i) & (ii) we get

128. Four wires of the same material are stretched by the same load. Which one of them will
elongate most if their dimensions are as follows

(1) L = 100 cm, r = 1mm


(2) L = 200 cm, r = 3mm

(3) L = 300 cm, r = 3mm


(4) L = 400 cm, r = 4mm

Correct Answer : (1)

Explanation :
Because, wires of the same material are stretched by the same load. So, F and Y will be
constant.
We can see that, L=100 cm and r = 1mm will elongate most.

129. If density of earth increased 4 times and its radius become half of what it is, our weight will

(1) Be four times its present value


(2) Be doubled

(3) Remain same


(4) Be halved

Correct Answer : (2)


Explanation :

130. The centripetal acceleration of a body moving in a circle of radius 100 m with a time period
of 2 s will be

(1)
(2)
(3)
(4)

Correct Answer : (4)


Explanation : Centripetal acceleration,
131. In ac circuit of capacitance the current from potential is

(1) Forward
(2) Backward

(3) Both are in the same phase


(4) None of these

Correct Answer : (1)

132. If in a resonance tube a oil of density higher than that water is used then at the resonance
frequency would

(1) Increase
(2) Decrease
(3) Slightly increase
(4) Remain same

Correct Answer : (4)


Explanation : In a resonance tube, water works as a reflector and the resonance frequency is
independent of the substance (liquiwhich is filled in the tube.

133. A bomb of mass at rest explodes into two fragments of masses and . The total
energy released in the explosion is . If and represent the energies carried by
masses and respectively, then which of the following is correct?

(1)

(2)

(3)

(4)
Correct Answer : (1)

134. A car accelerates from rest at a constant rate for some time, after which it decelerates at
a constant rate and comes to rest. If the total time elapsed is then the maximum velocity
acquired by the car is

(1)

(2)

(3)

(4)

Correct Answer : (4)


Explanation : Let the car accelerate at rate for time then maximum velocity attained,

Now, the car decelerates at a rate for time and finally comes to rest. Then,

135. The correct order of ionizing capacity of and rays is

(1)
(2)
(3)
(4)

Correct Answer : (2)


Explanation : Ionizing property depends upon the charge and mass

136. The equation of progressive wave is , where x and y are in metre and
t is in second. The velocity of propagation of the wave is

(1)
(2)
(3)
(4)

Correct Answer : (1)


Explanation : Velocity of propagation

137. Water of volume 2 L in a container is heated with a coil of 1 kW at 27 . The lid of the
container is open and energy dissipates at rate of 160 . In how much time temperature will
rise from 27 to 77 [Given specific heat of water is 4.2 kJ ]

(1) 8 min 20 s
(2) 6 min 2 s
(3) 7 min
(4) 14 min

Correct Answer : (1)


Explanation : Energy gained by water (in 1 s)
= Energy supplied energy lost
= (1000 J-160 J)=840 J
Total heat required to raise the temperature of water from 27 . Hence, the
required time,
= 8 min 20 s.

138. There are two electric bulbs of and Which one will be brighter when first
connected in series and then in parallel

(1) in series and in parallel

(2) in series and in parallel

(3) both in series and parallel will be uniform

(4) both in series and parallel will be uniform

Correct Answer : (1)

Explanation : Resistance of bulb is times than In

series, and in parallel, So in series and in parallel will glow brighter

139. Out of the following electromagnetic radiation, which has the shortest wavelength?

(1) Radiowaves
(2) Infrared
(3) Ultraviolet
(4) X-rays

Correct Answer : (4)


Explanation : The -rays ahs the shortest wavelength among the following radiations
140. The points resembling equal potentials are

(1) P and
(2) S and
(3) S and R
(4) P and R

Correct Answer : (1)


Explanation : The points S and R are inside the uniform electric field, so these will be at equal
potential.

141. An electron is accelerated through a potential difference of volt. The speed of electrons
will be

(1)

(2)

(3)

(4)

Correct Answer : (2)


Explanation : If is the velocity attained by electron, then
v=

142. A uniformly charged thin spherical shell of radius carries uniform surface charge density
of per unit area. It is made of two hemispherical shells, held together by pressing them with
force (see figure). is proportional to

(1)

(2)

(3)

(4)

Correct Answer : (1)


Explanation : Electrostatic repulsive force;

143. Fading is the variation in the strength of a signal at a receiver due to

(1) Interference of waves


(2) Diffraction of waves

(3) Polarisation of waves


(4) None of these

Correct Answer : (1)

144. A cylindrical tube containing air is open at both ends. If the shortest length of the tube for
resonance with a given fork is 2 cm, the next shortest length for resonance with the same fork
will be

(1) 60 cm
(2) 40 cm
(3) 90 cm
(4) 80 cm

Correct Answer : (2)


Explanation : As the tube is open at both ends, therefore, next shortest length for
resonance .

145. The far point of a myopia eye is at . For removing this defect, the power of lens
required will be

(1) 40
(2)
(3)
(4) 0.25

Correct Answer : (3)


Explanation : For myopic eye (defected far point)

146. Magnetic field induction at the centre of a square loop of side ‘ ’ carrying current as
shown in figure is
(1)

(2)

(3)
(4) Zero

Correct Answer : (4)


Explanation : and and are in the opposite direction pairs. They are so situated
that currents of each pair produce equal and opposite magnetic fields at the centre of the
loop. Hence, the resultant magnetic field induction at the centre of the loop is zero

147. The first member of the Balmer’s series of the hydrogen has a wavelength , the
wavelength of the second member of its series is

(1)

(2)

(3)
(4) None of these

Correct Answer : (2)


Explanation : The wavelength of series for is given by

=
were is Rydberg’s constant.
For Balmer series =3 gives the first member of series and =4 gives the second member of
series. Hence,
=

= …(i)

= …(ii)

148. If a body is executing simple harmonic motion then

(1) At extreme positions, the total energy is zero

(2) At equilibrium position, the total energy is in the form of potential energy

(3) At equilibrium position, the total energy is in the form of kinetic energy

(4) At extreme position, the total energy is infinite

Correct Answer : (3)


Explanation : At equilibrium position, potential energy of the body is zero. So, the total energy
at equilibrium position is completely kinetic energy.

149. The root mean square value of the alternating current is equal to

(1) Twice the peak value


(2) Half the peak value

(3) times the peak value


(4) Equal to the peak value

Correct Answer : (3)


150. A bullet of mass 5 g is shot from a gun of mass 5 kg. The muzzle velocity of the bullet
is . The recoil velocity of the gun is

(1)
(2)
(3)
(4) Data is insufficient

Correct Answer : (1)


Explanation : Let be mass of bullet and gun and be their initial velocities and
and be their final velocities. From law of conservation of momentum.

kg

151. Time period of a simple pendulum of length and time period of a uniform rod of
the same length pivoted about one end and oscillating in a vertical plane is . Amplitude of

oscillations in both the cases is small. Then is

(1)
(2)

(3)

(4)

Correct Answer : (4)


Explanation : Time period of simple pendulum is given by

and time period of uniform rod in given position is given by

Here, inertia factor=moment of inertia of rod at one end

Spring factor=restoring torque per unit angular displacement

(if is small)

152. The magnetic field between the plate of a capacitor when is given by

(1)

(2)
(3)
(4) Zero

Correct Answer : (1)


Explanation : Consider a loop of radius between the two circular plates, placed
coaxially with them. The area of the loop =
By symmetry magnetic field is equal in magnetic at all points on the loop. If is the
displacement current crossing the loop and is the total displacement current between

plates . Using Ampere Maxwell’ law we have,

or

153. To get three images of a single object, one should have two plane mirrors at an angel of

(1)
(2)
(3)
(4)

Correct Answer : (2)

Explanation :

154. Two similar coils of radius are lying concentrically with their planes at right angles to each
other. The currents flowing in them are and , respectively. The resultant magnetic field
induction at the centre will be

(1)
(2)

(3)

(4)

Correct Answer : (1)

Explanation :

155. Two simple harmonic motions are represented

by and The resultant amplitude is

(1) 7
(2) 1
(3) 5
(4)

Correct Answer : (1)

Explanation :

The phase difference both are along the same line

The resultant amplitude is 7 units


156. Electric potential at the centre of a charged hollow metal sphere is

(1) Zero
(2) Twice as that on the surface

(3) Half of that on the surface


(4) Same as that on the surface

Correct Answer : (4)


Explanation : Electric potential of charged spherical shell

∴ Electric potential at centre=Electric potential on the surface.

157. A straight conductor carrying current If the magnetic field at a distance is 0.4 T, then
magnetic field at a distance will be

(1) 0.4 T
(2) 0.1 T
(3) 0.8 T
(4) 0.2 T

Correct Answer : (4)

Explanation :

or

or

158. In an -circuit time constant is that time in which current grows from zero to the value
(where is the steady state current)
(1)
(2)
(3) 0.37
(4)

Correct Answer : (1)


Explanation : Current at any instant of time after closing an - circuit is given

by

Time constant

of

159. If the curves of two samples of P and Q of iron are as shown below, then which one
of the following statements is correct?

(1) Both P and Q are suitable for making permanent magnet

(2) P is suitable for making permanent magnet and Q for making electromagnet

(3) P is suitable for making electromagnet and Q is suitable for permanent magnet

(4) Both P and Q are suitable for making electromagnets

Correct Answer : (2)


Explanation : For permanent magnet we prefer a material with high retentivity (so as to make
a stronger magnet) and high coercivity (so that magnetization may not be wiped out easily). For
electromagnet we prefer high saturated magnetism, low coercivity and least possible area of
hysteresis loop so that electromagnet develops high magnetization, is easily demagnetized and
energy loss in a magnetization cycle is least. Therefore, P is suitable for making permanent
magnet and Q for making electromagnet.

160. A ball of mass 1 kg hangs in equilibrium from two strings and as shown in figure.
What are the tensions in strings and (Take )

(1) 5 N, zero
(2) Zero, N
(3) 5 N, 5 N
(4) 5 N, 5 N

Correct Answer : (3)


Explanation : Various forces acting on the ball are as shown in figure. The three concurrent
forces are in equilibrium. Using Lami’s theorem

and N

161. When the wave of hydrogen atom comes from infinity into the first orbit then the value of
wave number is
(1)
(2)
(3)
(4) None of these

Correct Answer : (1)

Explanation : Wave number and

162. Calculate the phase velocity of electromagnetic wave having electron density and
frequency of D-layer, electron kHz

(1)
(2)
(3)
(4)

Correct Answer : (3)

Explanation :

Phase velocity,

163. If the mean free path of atoms is doubled then the pressure of gas will become

(1)
(2)
(3)
(4)

Correct Answer : (2)

Explanation : Mean free path If is doubled then becomes half

164. When an aeroplane attains a speed higher than the velocity of sound in air, a loud bang is
heard. This is because

(1) It explodes

(2) It produces a shock wave which is received as the bang

(3) Its wings vibrate so violently that the bang is heard

(4) The normal engine noises undergo a Doppler shift to generate the bang

Correct Answer : (2)

165. A transistor conducts when

(1) Both collector and emitter are positive with respect to the base

(2) Collector is positive and emitter is negative with respect to the base

(3) Collector is positive and emitter is at same potential as the base

(4) Both collector and emitter are negative with respect to the base

Correct Answer : (2)

Explanation :

166. In a circuit having and in series. The resonance


frequency in per second is
(1) 700
(2) 600
(3) 500
(4) 500

Correct Answer : (3)


Explanation : Resonance frequency in is

167. A beam of monochromatic blue light of wavelength in air travels in water


. Its wavelength in water will be

(1) 2800
(2) 5600
(3) 3150
(4) 4000

Correct Answer : (3)

Explanation :

168. The Electromagnetic theory of light failed to explain

(1) Photoelectric effect


(2) Polarization
(3) Diffraction
(4) Interference

Correct Answer : (1)


Explanation : The electromagnetic theory of light failed to explain photoelectric effect.

169. The length of a conductor is doubled and its radius is halved, its specific resistance is

(1) Unchanged
(2) Halved
(3) Doubled
(4) Quadrupled

Correct Answer : (1)


Explanation : Specific resistance is independent of dimensions of conductor but depends on
nature of conductor.

170. If and represent the permittivity and permeability of vacuum and and represent
the permittivity and permeability of medium, then refractive index of the medium is given by

(1)

(2)

(3)

(4)

Correct Answer : (2)

Explanation :
171. The motion of a particle executing S.H.M. is given by where is
in and time is in seconds. The time period is

(1)
(2)
(3)
(4)

Correct Answer : (2)

Explanation :

172. A satellite moves in elliptical orbit about a planet. The maximum and minimum velocities
of satellites are and respectively. What is the minimum distance of
satellite from planet, if maximum distance is

(1)
(2)
(3)
(4)

Correct Answer : (3)


Explanation : If no external torque acts on a system, then angular momentum of the system
does not change.
173. A disc of mass 2 kg and radius 0.2 m is rotating with angular velocity 30 . What is
angular velocity, if a mass of 0.25 kg is put on periphery of the disc?

(1)
(2)
(3)
(4)

Correct Answer : (1)


Explanation : If no external torque acts on a system of particle then angular momentum of the
system remains constant, that is,

constant

…(i)
Here, kg, 0.25 kg, 0.2m.

Hence, we get after putting the given values in Eq. (i)

174. The velocity of water waves may depend upon their wavelength , the density of
water and the acceleration due to gravity . The method of dimensions gives the relation
between these quantities as

(1)
(2)
(3)
(4)

Correct Answer : (3)


Explanation : Let . Now by submitting the dimensions of each quantities and
equating the powers of and

175. Consider telecommunication through optical fibres. Which of the following statements is
no true?

(1) Optical fibres can be of graded refractive index

(2) Optical fibres are subjected to electro-magnetic interference from outside

(3) Optical fibres have extremely low transmission loss

(4) Optical fibres may have homogeneous core with a suitable cladding

Correct Answer : (2)


Explanation : Some of the characteristics of an optical fibre are as follows.
(i) This works on the principle of total internal reflection.
(ii) It consists of core made up of glass/silica/plastic with refractive index which is
surrounded by a glass or plastic cladding with refractive index The
refractive index of cladding can be either changing abruptly or gradually changing
(Graded index fibr.
(iii) There is a very little transmission loss through optical fibres.
(iv) There is no interference from stray electric and magnetic fields to the signals
through optical fibres.

176. Fraunhoffer spectrum is

(1) Line absorption spectrum


(2) Band absorption spectrum

(3) Line emission spectrum


(4) Band emission spectrum

Correct Answer : (1)


Explanation : The atoms in the chromosphere absorb certain wavelengths of light coming from
the photosphere. This gives rise to absorption lines

177. The deflection in a moving coil galvanometer is reduced to half when it is shunted with a
40 coil. The resistance of the galvanometer is

(1) 15
(2) 20
(3) 40
(4) 80

Correct Answer : (3)


Explanation :

178. A solid sphere rolls down without slipping on an inclined plane at angle 60 over a distance
of 10 m. the acceleration is

(1) 4
(2) 5
(3) 6
(4) 7

Correct Answer : (3)


Explanation : Here,

For solid sphere,


179. A car weighing 1400 kg is moving at a speed of up a hill when the motor stops. If
it is just able to reach the destination which is at a height of 10 m above the point, then the
work done against friction (negative of the work done by the friction) is [Take ]

(1) 10 kJ
(2) 15 kJ
(3) 17.5 kJ
(4) 25 kJ

Correct Answer : (3)

Explanation :
or
or
or

180. If dielectric is inserted in charged capacitor (battery removed ), then quantity that remains
constant is

(1) Capacitance
(2) Potential
(3) Intensity
(4) Charge

Correct Answer : (4)


Explanation : Variation of different variables of parallel plate capacitor when
dielectric ( ) is introduced when battery is removed is
181. Two particles having position vectors and are
moving with velocities and . If they collide after 2 seconds,
the value of ‘ ’ is

(1) 2
(2) 4
(3) 6
(4) 8

Correct Answer : (4)


Explanation : It is clear from figure that the displacement vector between
particles and is

….(i)
Now, as the particles are moving in same direction

and are , the relative velocity is given by

…(ii)

Now, we know
Substituting the values of and from equation (i) and (ii) and , then on solving we
get

182. The electrical circuit used to get smooth DC output from a rectifier circuit is called

(1) Filter
(2) Oscillator
(3) Logic gates
(4) Amplifier

Correct Answer : (1)


Explanation : To get smooth DC output from a rectifier circuit filter is used.

183. The figure shows the path of a positively charged particle 1 through a rectangular region of
uniform electric field as shown in the figure. What is the direction of electric field and the
direction of particles 2, 3 and 4

(1) Top; down, top, down


(2) Top; down, down, top

(3) Down; top, top, down


(4) Down; top, down, down

Correct Answer : (1)


Explanation : The figure shows the path of a charged particle (1) through a rectangular
region of uniform electric field.

Since, charged particle moves as a parabolic path in electric field, it means the direction of
electric field is upward. The direction of particle (2) which is is downward.
The direction of deflection of particle (3) which is is upward and direction of deflection of
particle (4) is downward

184. If the speed of light ( ), acceleration due to gravity and pressure are taken as the
fundamental quantities, then the dimension of gravitational constant is

(1)
(2)
(3)
(4)

Correct Answer : (2)


Explanation : Let
By substituting the following dimensions:

and by comparing the powers of both sides


we can get

185. In the circuit shown below, the ac source has voltage volts
with . The amplitude of the current will be nearest to

(1)
(2) 3.3
(3)
(4)

Correct Answer : (1)


Explanation :

Amplitude of current
186. The total energy of a particle, executing simple harmonic motion is
Where is the displacement from the mean position.

(1)
(2)
(3) Independent of
(4)

Correct Answer : (3)


Explanation : In simple harmonic motion when a particle displaced to a position from its mean
position then its kinetic energy gets converted in potential energy. Hence, total energy of
particle remains constant or the total energy in simple harmonic motion does not depend is
displacement .

187. A gas expands adiabatically at constant pressure, such that its temperature . The
value of of the gas is

(1) 1.30
(2) 1.50
(3) 1.67
(4) 2.00

Correct Answer : (2)


Explanation : For adiabatic expansion, we have the formula
…(i)
Gas equation is ,
pV=RT

…(ii)
From Eqs. (i) and (ii), we obtain

…(iii)

But (given)
as …(iv)
Thus, using Eqs. (iii) and (iv) togther, we get

188. A battery consists of a variable number of identical cells having internal resistances
connected in series. The terminals of battery are short circuited and the current is measured.
Which of the graph below slows the relationship between and

(1)

(2)

(3)

(4)

Correct Answer : (4)

Explanation : Short circuited current doesn’t depend upon


189. Planck’s constant has the dimensions of

(1) Energy
(2) Mass
(3) Frequency
(4) Angular momentum

Correct Answer : (4)


Explanation : Planck’s constant,

Angular momentum,

190. In -type semiconductors, majority charge carriers are

(1) Holes
(2) Protons
(3) Neutrons
(4) Electrons

Correct Answer : (4)


Explanation : In -type semiconductors, electrons are majority charge carriers

191. The circuit shown in the figure contains two diodes each with a forward resistance
of and with infinite backward resistance. If the battery is 3V, the current through
the resistance (in ampere) is

(1) Zero
(2) 0.01
(3) 0.02
(4) 0.03

Correct Answer : (3)


Explanation : In the circuit the upper diode is reverse biased and the lower diode is
forward biased. Thus there will be no current across upper diode junction. The effective circuit
will be as shown in figure.

Total resistance of circuit

Current in circuit,

192. Figure here shown an incident pulse reflected from a rigid support. Which one
of represents the reflected pulse correctly

(1)

(2)

(3)

(4)

Correct Answer : (4)


Explanation : When pulse is reflected from a rigid support, the pulse is inverted both
lengthwise and sidewise

193. If an ideal gas is compressed isothermally then

(1) No work is done against gas


(2) Heat is released by the gas

(3) The internal energy of gas will increase


(4) Pressure does not change

Correct Answer : (2)


Explanation : In isothermal process, heat is released by the gas to maintain the constant
temperature

194. and are the velocities of sound at the same temperature in two monoatomic gases of

densities and respectively. If then the ratio of velocities and will be

(1)
(2)
(3)
(4)

Correct Answer : (3)


Explanation : At given temperature and pressure

195. The apparent frequency of the whistle of an engine changes in the ratio 9:8 as the engine
passes a stationary observer. If the velocity of the sound is 340 , then the velocity of the
engine is
(1)
(2)
(3)
(4)

Correct Answer : (1)


Explanation : From doppler’s effect, perceived frequency

196. When a certain weight is suspended from a long uniform wire, its length increases by 1 cm.
If the same weight is suspended from another wire of the same material and length but having
a diameter half of the first one, the increase in length will be

(1) 0.5 cm
(2) 2 cm
(3) 4 cm
(4) 8 cm

Correct Answer : (3)

Explanation :

197. Imagine an atom made up of proton and a hypothetical particle of double the mass of
electron, but having the same charge as that of electron. Apply the Bohr atom model and
consider all possible transitions of this hypothetical particle to the first excited level. The
longest wavelength photon that will be emitted has wavelength (given in terms of Rydberg
constant for hydrogen atom) equal to
(1)

(2)

(3)

(4)

Correct Answer : (3)

Explanation : In hydrogen atom,


Also, , where is the mass of the electron. Here, the electron has been replaced by a
particle, whose mass is double the mass of an electron. Therefore, this hypothetical atom,
energy is th orbit will be given by

The longest wavelength (or minimum energy) photon will correspond to the transition of
particle from to

198. An observer is standing 500 m away from a vertically hill. Starting between the observer
and the hill a police van having a siren of frequency 1000 Hz moves towards the hill with a
uniform speed. If the frequency of the sound heard directly from the siren is 970 Hz, the
frequency of the sound heard after reflection from the hill (in Hz) is about, (velocity of sound =
)

(1) 1042
(2) 1032
(3) 1022
(4) 1012

Correct Answer : (2)


Explanation : Sound geard directly

Or
The frequency of reflected sound is given by

199. If a radioactive substance reduces to of its original mass in , what is its half life

(1)
(2)
(3)
(4) None of these

Correct Answer : (1)

Explanation :

Also

200. In the given figure two tiny conducting balls of identical mass and identical
charge hang from non-conducting threads of equal length Assume that is so small
that then for equilibrium is equal to
(1)

(2)

(3)

(4)

Correct Answer : (1)

201. A Carnot engine whose source is at 400 K take 200 cal of heat and rejects 150 cal to the
sink. What is the temperature of the sink?

(1) 800 K
(2) 400 K
(3) 300 K
(4) Cannot say

Correct Answer : (3)

Explanation : As

202. A ray of light strikes a transparent rectangular slab (of refractive index ) at an angle of
incidence of . The angle between the reflected and refracted rays is

(1) 75
(2) 90
(3) 105
(4) 120
Correct Answer : (3)
Explanation : Here,
Applying Snell’s law at air-glass surface, we get

From figure,

Hence, the angle between reflected and refracted rays is

203. A circular disc of radius is made from an iron plate of thickness , and another
disc of radius 4 is made from an iron plate of thickness /4. Then the relation between the
moment of inertia and is

(1)
(2)
(3)
(4)

Correct Answer : (4)


Explanation : Mass of disc
Where, density of material of disc
…(i)
Mass of disc

and
…(ii)

204. Seven capacitors each of the capacitance are be connected in a configuration to obtain

an effective capacitance of . Which of the combination (S) shown in figure will achieve the
desired result?

(1)

(2)

(3)

(4)

Correct Answer : (1)


Explanation : Net capacity of 5 capacitors joined in parallel . now it is connected

with two capacitors of 2 each in series, hence equivalent capacitance is

205. A wire of density is stretched between two clamps 1m part and is subjected
to an extension of . The lowest frequency of transverse vibration in the wire
is

(1) 40 Hz
(2) 35 Hz
(3) 30 Hz
(4) 25 Hz

Correct Answer : (2)


Explanation : Given :
From the formula

Or

=
=44.1
206. A pulse of a wave train travels along a stretched string and reaches the fixed end of the
string. It will be reflected with

(1) A phase change of 180° with velocity reversed

(2) The same phase as the incident pulse with no reversal of velocity

(3) A phase change of 180° with no reversal of velocity

(4) The same phase as the incident pulse but with velocity reversed

Correct Answer : (3)


Explanation : A pulse of a wave train when travels along a stretched string and reaches the
fixed end of the string, then it will be reflected back to the same medium and the reflected ray
suffers a phase change of π with the incident wave but wave velocity after reflection does not
change.

207. The natural frequency of a circuit is equal to

(1)

(2)

(3)

(4)

Correct Answer : (2)

208. What is the value of sink temperature when efficiency of engine is 100%?

(1) Zero
(2) 300 K
(3) 273 K
(4) 400 K

Correct Answer : (1)

Explanation : Efficiency
where, sink temperature,
source temperature.
For 100% efficiency,

209. A piston fitted in cylindrical pipe is pulled as shown in the figure. A tuning fork is sounded
at open end and loudest sound is heard at open length 13cm, 41 cm and 69 cm, the frequency
of tuning fork if velocity of sound is is

(1) 1250 Hz
(2) 625 Hz
(3) 417 Hz
(4) 715 Hz

Correct Answer : (2)


Explanation : In a closed organ pipe in which length of air-column can be increased or
decreased, the first resonance occurs at and second resonance occurs at
Thus, at first resonance

And a second resonance


Subtracting Eq.(i) from Eq.(ii), we have

Hence, frequency of tuning fork

210. is the dimension of

(1) Work
(2) Power
(3) Force
(4) Momentum

Correct Answer : (2)

Explanation : Power

[Power]
211. The de-Broglie wavelength associated with an electron having kinetic energy is given
by the expression

(1)

(2)
(3)

(4)

Correct Answer : (1)

Explanation :

212. A ball of mass 1 carrying a charge Cmoves from a point A at potential 600 V to a
point B at zero potential. The change in its KE is

(1) erg
(2) J
(3) J
(4) erg

Correct Answer : (3)


Explanation : As work is done by the field, KE of the body increase by
KE =

213. There are two spherical balls and of the same material with same surface, but the
diameter of is half that of . If and are heated to the same temperature and then allowed
to cool, then

(1) Rate of cooling is same in both


(2) Rate of cooling of is four times that of

(3) Rate of cooling of is twice that of

(4) Rate of cooling of is times that of

Correct Answer : (3)

Explanation : Rate of cooling

Since diameter of is half that of so it’s rate of cooling will be doubled that of

214. When a body moves in a circular path, no work is done by the force since

(1) force and displacement are perpendicular other


(2) the force is always away from the center

(3) there is no displacement


(4) there is no net force

Correct Answer : (1)


Explanation : When a body moves on a circular path then force and distance are perpendicular
to each other. Therefore, work done by the force is

215. The voltage of clouds is with respect to ground. In a lightening strike


lasting a charge of 4 coulombs is delivered to the ground. The power of lightening
strike is

(1)
(2)
(3)
(4)

Correct Answer : (2)

Explanation : Work done

Power

216. The current from is increasing in magnitude. What is the direction of induced
current, if any, in the loop shown in figure.

(1) No current is induced


(2) Clock-wise current

(3) Anti-clock-wise current


(4) Alternating current

Correct Answer : (2)


Explanation : Magnetic flux through the loop is upward and its is increasing due to increasing
current along . Current induced in the loop should have magnetic flux in the downward
direction so at to oppose the increase in flux. Therefore, current induced in the loop is
clockwise.

217. A body thrown vertically upwards with an initial velocity reaches maximum height in 6
seconds. The ratio of the distances travelled by the body in the first second and the seventh
second is

(1)
(2)
(3)
(4)
Correct Answer : (2)

Explanation : Time of ascent

Distance in first second


Distance in seventh second will be equal to the distance in first second of vertical downward
motion

218. You drive a car at seed of in a straight road for and then the car runs out
of petrol. You walk for to reach a petrol pump at a distance of . The average
velocity from the beginning of your drive till you reach the petrol pump is

(1)
(2)
(3)
(4)

Correct Answer : (1)

Explanation :

Average speed

219. A cylindrical bar magnet is kept along the axis of a circular coil. If the magnet is rotated
about its axis, then
(1) A current will be induced in a coil
(2) No current will be induced in a coil

(3) Only an e.m.f. will be induced in the coil


(4) An e.m.f and a current both will be induced in the coil

Correct Answer : (2)


Explanation : Because there is no change in flux linked with coil

220. A steamer taken 12 days to reach from part to part . Every day only one steamer sets
out from both the ports. How many steamers does each steamer meet in the open sea?

(1) 23
(2) 25
(3) 27
(4) 21

Correct Answer : (1)

221. Beats are produced by two travelling waves each of loudness I and nearly equal
frequencies and . The beat frequency will be …. and maximum loudness hard will be

(1) 2
(2) 4
(3) 3
(4)

Correct Answer : (2)


Explanation : Beat frequency=number of beats

And maximum loudness


222. The wavelength of a 1 keV photon is m. What is the frequency of 1 MeV
photon?

(1) Hz
(2) Hz
(3) Hz
(4) Hz

Correct Answer : (2)


Explanation : …(i)
…(ii)
Dividing Eq.(ii) by Eq. (i) we get,
Hz

223. Two tuning fork, A and B produce notes of frequencies 258 Hz and 262 Hz. An unknown
note sounded with a produces certain beats. When the same note is sounded with B, the beat
frequency gets doubled, the unknown frequency is

(1) 256 Hz
(2) 254 Hz
(3) 300 Hz
(4) 280 Hz

Correct Answer : (2)


Explanation :

Let n is the frequency of unknown tuning fork. It produces x beats with 258 and 2x with 262

224. A particle moves a distance in time according to equation . The


acceleration of particle is proportional to

(1)
(2)
(3)
(4)

Correct Answer : (2)

Explanation :

Acceleration,

225. A particle of mass m moving with horizontal speed as shown in figure.


If than for one dimensional elastic collision, the speed of lighter particle after collision
will be

(1) in original direction


(2) opposite to the original direction

(3) opposite to the original direction


(4) in original direction

Correct Answer : (1)

Explanation :

Substituting

the lighter particle will move in original direction with the speed of

226. A metallic solid sphere is placed in a uniform electric field. The lines of force follow the
paths shown in figure
(1) 1
(2) 2
(3) 3
(4) 4

Correct Answer : (4)


Explanation : In a uniform electric field, field line should be straight but line of force cannot
pass through the body of metal sphere and must end/start from the sphere normally. All these
conditions are fulfilled only in plot (.

227. The displacement of the particle varies with time according to the

relation Then the velocity of the particle is

(1)

(2)
(3)
(4) None of these

Correct Answer : (1)

Explanation :

228. The galaxy in which we live is

(1) Milkyway
(2) Radio galaxy
(3) Spiral galaxy
(4) None of the above

Correct Answer : (1)


Explanation : Milkway is a glowing belt in sky formed due to the combined light of the large
number of stars constituting it. Our solar system is part of Milkway.

229. The process of superimposing signal frequency ( audio wave) on the carrier wave is
known as

(1) Transmission
(2) Reception
(3) Modulation
(4) Detection

Correct Answer : (3)


Explanation : Carrier + signal modulation

230. If the area to be covered for TV telecast is doubled, then height of transmitting antenna (TV
tower) will have to be

(1) Doubled
(2) Halved
(3) Quadrupled
(4) Kept unchanged

Correct Answer : (1)


Explanation : In optical communication, light is transmitted in form of light pulses along an
optical fibre is transmission medium. Since, wavelength of light is very small as compared o the
radio waves, it provides a very large band width and can carry a huge amount of information.
In optical fibres there occurs a total internal reflection of light so interference phenomenon
must not occur.

231. A steel wire of 1m long and cross section area is hang from rigid end. When weight
of is hung from it then change in length will be (given )

(1)
(2)
(3)
(4)

Correct Answer : (3)

Explanation :

232. The escape velocity of a body from earth’s surface is . The escape velocity of the same
body from a height equal to from earth’s surface will be

(1)

(2)

(3)

(4)

Correct Answer : (3)

Explanation : where is a position of body from the surface

233. A body at rest breaks into two pieces with unequal mass

(1) Both of them have equal speeds

(2) Both of them move along a same line with unequal speeds
(3) Sum of their momentum is non zero

(4) They move along different lines with different speeds

Correct Answer : (2)

234. - diagram of an ideal gas is as shown in figure. Work done by the gas in process is

(1)
(2)
(3)
(4)

Correct Answer : (3)


Explanation : and

235. A magnet of length 0.1 m and pole strength A-m is kept in a magnetic field of 30
Wb at an angle The couple acting on it is …….. Nm.

(1) 7.5
(2) 3.0
(3) 4.5
(4) 1.5

Correct Answer : (4)


Explanation : Torque,
236. Which of the following is dimensionless?

(1)

(2)

(3)
(4)

Correct Answer : (1)


Explanation : The velocity of a body at highest point of vertical circle is,

Or

Or

Hence, is dimensionless.

237. A sheet of aluminium foil of negligible thickness is introduced between the plates of a
capacitor. The capacitance of the capacitor

(1) Decreases
(2) Remain unchanged
(3) Becomes infinite
(4) Increases

Correct Answer : (2)


Explanation : From the formula ,

Here,

So,
238. The time period of geostationary satellite at a height 36000 km is 24 h. A spy satellite orbits
earth at a height 6400km. What will be the time period of sky satellite?
(Radius of earth = 6400 km )

(1) 5 h
(2) 4 h
(3) 3 h
(4) 12 h

Correct Answer : (2)


Explanation : From Kepler’s third law of planetary motion

239. An object is moving through the liquid. The viscous damping force acting on it is
proportional to the velocity. Then dimension of constant of proportionality is

(1)
(2)
(3)
(4)

Correct Answer : (4)

Explanation :

240. A police jeep is chasing with velocity of a thief in another jeep moving with
velocity . Police fires a bullet with muzzle velocity of . The velocity with which
it will strike the car of the thief is

(1)
(2)
(3)
(4)

Correct Answer : (1)


Explanation : Effective speed of bullet
speed of bullet + speed of police jeep

Velocity of bullet thief’s car

241. If the temperature is raised by 1 K from 300 K the percentage change in the speed of
sound in the gaseous mixture is (R=8.31 J/mol-K)

(1) 0.167%
(2) 0.334%
(3) 1%
(4) 2%

Correct Answer : (1)


Explanation : From

242. A mass of 50 kg is raised through a certain height by a machine whose efficiency is 90%,
the energy is 5000 J. If the mass is now released, its KE on hitting the ground shall be

(1) 5000 J
(2) 4500 J
(3) 4000 J
(4) 5500 J

Correct Answer : (2)


Explanation : Because the efficiency of machine is 90%, hence, potential energy gained by the
mass

energy spend
When the mass is released now, gain in KE on hitting the ground
Loss of potential energy

243. Latent heat of of steam is then its value in is

(1)
(2)
(3)
(4) None

Correct Answer : (1)

Explanation :
244. One man takes 1 minute to raise a box to a height of 1 m and another man takes minute
to do so. The energy of the two is

(1) Different
(2) Same

(3) Energy of the first is more


(4) Energy of the second is more

Correct Answer : (2)


Explanation : Energy required
In both cases, is the same. Hence, energy given by both is same. [It is worth noting here that
powers of two men will be different as power is the energy expense per unit time and times are
different]

245. Capacitance of a parallel plate capacitor becomes times its original value, if a dielectric

slab of thickness is inserted between the plates [ is the separation between the plates].
The dielectric constant of the slab is

(1) 4
(2) 8
(3) 2
(4) 6

Correct Answer : (3)


Explanation : The capacitance of air capacitor

When a dielectric slab of thickness is inserted between plates, the capacity becomes
or
or

246. The relation between the displacement of an object produced by the application of the
variable force is represented by a graph shown in the figure. If the object undergoes a
displacement from to the work done will be approximately equal to

(1)
(2)
(3)
(4)

Correct Answer : (1)


Explanation : Work done = Area under curve and displacement axis
= Area of trapezium

As the area actually is not trapezium so work done will be more than approximately
247. The maximum energy in thermal radiation from a source occurs at the wavelength 4000 .
The effective temperature of the source is

(1) 7325 K
(2) 800 K
(3) K
(4) K

Correct Answer : (1)


Explanation : According to Wien’s displacement law

248. We combined a convex lens of focal length and concave lens of focal lengths and
their combined focal length was . The combination of these lenses will behave like a concave
lens, if

(1)
(2)
(3)
(4)

Correct Answer : (1)

Explanation : will be negative if

249. What is the moment of inertia of solid sphere of density and radius about its diameter?

(1)

(2)
(3)

(4)

Correct Answer : (3)

Explanation :

250. The particle accelerator that uses the phenomenon of electromagnetic induction is the

(1) Cyclotron
(2) Betatron

(3) Van de Graff generator


(4) Cockroft- Walton generator

Correct Answer : (2)


Explanation : Betatron uses the phenomenon of electromagnetic induction.

251. In Colpitt oscillator the feedback network consists of

(1) Two inductors and a capacitor


(2) Two capacitors and an inductor

(3) Three pairs of R-C circuit


(4) Three pairs of R-L circuit

Correct Answer : (2)


Explanation : In Colpitt oscillator two capacitors are placed across a common inductor and the
centre of the two capacitors is tapped.
252. The figure shows the volume versus temperature graphs for a certain mass of a
perfect gas at two constant pressures of and What inference can you draw from the
graphs

(1)
(2)

(3)
(4) No interference can be drawn due to insufficient information

Correct Answer : (1)

Explanation :

Form

Hence

253. A heater is operated with a power of in a line. It is connected in combination


with a resistance of and a resistance to a line as shown in figure. What should be
the value of so, that the heater operates with a power of

(1)
(2)

(3)
(4)

Correct Answer : (4)


Explanation : Resistance of heater

Total resistance of circuit

Current in heater

254. An -particle of energy 5 MeV is scattered through by a fixed uranium nucleus. The
distance of the closest approach is of the order of

(1) 1
(2) cm
(3) cm
(4) cm

Correct Answer : (3)


Explanation : According to law of conservation of energy, kinetic energy of -particle
= potential energy of -particle at distance of closest approach

MeV=
255. The angular momentum of electron in hydrogen atom is proportional to

(1)
(2)
(3)
(4)

Correct Answer : (1)

Explanation : Angular momentum

256. Two inductors of inductance each are connected in series with opposite magnetic fluxes.
What is the resultant inductance? (Ignore mutual inductance)

(1) Zero
(2)
(3) 2
(4) 3

Correct Answer : (3)


Explanation : Ignoring mutual induction, resultant, inductance

257. Radius of orbit of satellite of earth is . Its kinetic energy is proportional to

(1)
(2)
(3)

(4)

Correct Answer : (1)

Explanation :

258. Identify the incorrect statement from the following

(1) detection is carried out using a rectifier and an envelop detector

(2) Pulse Position denotes the time of rise or fall of the pulse amplitude

(3) Modulation index is kept , to avoid distortion

(4) Facsimile (FAX) scans the contents of the document to create electronic signals

Correct Answer : (3)


Explanation : Modulation index is kept to avoid distortion

259. In a triode amplifier, the value of maximum gain is equal to

(1) Half the amplification factor


(2) Amplification factor

(3) Twice the amplification factor


(4) Infinity

Correct Answer : (2)


Explanation : The maximum voltage gain
(Which is obtained when )
260. Two charged spheres of radii and are connected by a thin wire. No charge will
flow, if they have

(1) The same charge on each


(2) The same potential

(3) The same energy


(4) The same field on their surface

Correct Answer : (2)


Explanation : Because current flows from higher potential to lower potential

261. The maximum velocity and the maximum acceleration of a body moving in a simple
harmonic oscillator are and Then angular velocity will be

(1)
(2)
(3)
(4)

Correct Answer : (4)

Explanation : and

262. Dimension of is

(1)
(2)
(3)
(4) None of these
Correct Answer : (2)

Explanation :

263. A compass needle which is allowed to move in a horizontal plane is taken to a


geomagnetic pole. It

(1) Will become rigid showing no movement

(2) Will stay in any position

(3) Will stay in north-south direction only

(4) Will stay in east-west direction only

Correct Answer : (2)


Explanation : Since magnetic field is in vertical direction and needle is free to rotate in
horizontal plane only so magnetic force can not rotate the needle in horizontal plane so needle
can stay in any position

264. A tuning fork of frequency 200 Hz is in unison with a sonometer wire. The number of beats
heard per second when the tension is increased by 1% is

(1) 1
(2) 2
(3) 4
(4) 1/2

Correct Answer : (1)

Explanation :
Number of beats

265. Two Cu wires of radii such that Then which of the following is true?

(1) Transverse wave travels after in thicker wire


(2) Transverse wave travels faster in thinner wire

(3) Travels with the same speed in both the wires


(4) Does not travel

Correct Answer : (2)


Explanation : The velocity of a transverse wave

Because the velocity of wire depend on the radius. So transverse wave travels faster in thinner
wire.

266. In kinetic theory of gases, a molecule of mass of an ideal gas collides with a wall of
vessel with velocity The change in the linear momentum of the molecule is

(1)
(2)
(3)
(4) Zero

Correct Answer : (1)


Explanation :

267. The sound wave was produced in a gas is always

(1) Longitudinal
(2) Transverse
(3) Stationary
(4) Electromagnetic

Correct Answer : (1)


Explanation : Sound waves are longitudinal waves. They produce alternately the states of
compression and rarefaction at a point in the medium.

268. The volume of a gas at pressure and


temperature is If then the quantity of gas in will be

(1) 15
(2) 42
(3) 7
(4) 14

Correct Answer : (3)

Explanation :

269. A projectile can have the same range for two angles of projection. If and be the
times of flights in the two cases, then the product of the two times of flights is directly
proportional to

(1)
(2)
(3)
(4)

Correct Answer : (3)


Explanation : We know that range of projectile is same for complementary
angles for

270. A car is moving on a circular level road of radius of curvature 300 m. If the coefficient of
friction is 0.3 and acceleration due to gravity , the maximum speed the car can have is
(in km )

(1) 30
(2) 81
(3) 108
(4) 162

Correct Answer : (3)


Explanation : Here,
271. The temperature of cold junction of thermocouple is If the neutral temperature
is then the inversion temperature is

(1)
(2)
(3)
(4)

Correct Answer : (1)

Explanation : Neutral temperature,


or

272. An electron enters into a region of uniform magnetic field of strength with a
speed of which of the following is not possible

(1) The electron may or may not experience an acceleration

(2) The electron may experience an acceleration but can continue to move with same speed

(3) The electron may experience an acceleration and continue to move with same velocity

(4) The kinetic energy of the electron remains unchanged

Correct Answer : (3)

273. The radius of the proton is about m. The radius of the observable universe is m.
identify the distance which is half-way between these two extremes on a logarithmic scale.

(1) m
(2) m
(3) m
(4) m

Correct Answer : (2)


Explanation : m

Log

= m

274. Two positive point charges of 12 are placed 10 cm apart in air. The work
needed to bring them 4 cm closer is

(1) 2.4 J
(2) 3.6 J
(3) 1.6 J
(4) 6.0 J

Correct Answer : (2)


Explanation : Potential energy of charges distance apart

For m,

For

Work done = (9 – 5.4) J = 3.6 J

275. A body radiates energy at a temperature of . If the temperature is increased


to , then it radiates energy at the rate of

(1)
(2)
(3)
(4)

Correct Answer : (3)

Explanation : Rate of energy

276. Consider the following statement. When jumping from some height, you should bend your
knees as you come to rest instead of keeping your legs stiff. Which of the following relations
can be useful in explaining the statement?

Where symbols have their usual meaning

(1)
(2)

(3)
(4)

Correct Answer : (3)


Explanation : Change of momentum F

By doing so time of change in momentum increases and impulsive force on knees decreases.

277. Which state of triply ionised beryllium ( ) has the same orbital radius as that of ground
state of hydrogen?

(1)
(2)
(3)
(4)
Correct Answer : (4)
Explanation : Radius of orbit of electron in th excited state of hydrogen

But

So,
Here,
1 = 1(ground state of hydrogen),
1 = 1(atomic number of hydrogen),
2 = 4(atomic number of beryllium)

= (1)2
or
or 2

278. White light is passed through a dilutee solution of potassium permanganate. The spectrum
produced by the emergent light is

(1) Band emission spectrum


(2) Line emission spectrum

(3) Band absorption spectrum


(4) Line absorption spectrum

Correct Answer : (3)

279. The fundamental physical quantities that have same dimensions in the dimensional
formulae of torque and angular momentum are

(1) Mass, time


(2) Time, length
(3) Mass, length
(4) Time, mole

Correct Answer : (3)


Explanation : Torque = [ ], Angular momentum =
So mass and length have the same dimensions

280. As we go from the equator to the poles, the value of

(1) Remains the same


(2) Decreases

(3) Increases
(4) Decreases upto a latitude of

Correct Answer : (3)

281. The luminosity of the Rigel star is 17000 times that of the sun. Assume both to be perfectly
black bodies. If the surface temperature of the sun is 6000 K, then the temperature of the star
is

(1) 68400 K
(2) K
(3) 12000 K
(4) 68400

Correct Answer : (1)


Explanation : Luminosity of a star depends upon the total radiations emitted by the star.
The star emits 17000 times the radiations emitted by the sun.

Hence,

So, (Given,
282. A p-n photodiode is made of a material with a band gap of 2.0 eV. The minimum
frequency of the radiation that can be absorbed by the material is nearly

(1) Hz
(2) Hz
(3) Hz
(4) Hz

Correct Answer : (2)


Explanation : p-n photodiode is a semiconductor diode that produces a significant current
when illuminated. It is reversed biased but is operated below the breakdown voltage.
Energy of radiation = band gap energy
i.e., v = 2.0 eV

or v= Hz

283. Two galvanometer and require and respectively to produce the same
deflection of 10 divisions. Then

(1) is more sensitive than


(2) is more sensitive than

(3) and are equally sensitive


(4) Sensitiveness of is 5/3 times that of

Correct Answer : (1)

Explanation : Sensitivity

284. A graph is shown between stress and strain for a metal. The part in which Hooke’s law
holds good is
(1)
(2)
(3)
(4)

Correct Answer : (1)


Explanation : In the figure , stress strain Hooke’s law hold good

285. If the shortest wavelength in the Lyman series is 911.6 the longest wavelength in the
same series will be

(1)
(2)
(3)
(4)

Correct Answer : (3)

Explanation :

286. Pure inductance of is connected as shown below. The equivalent inductance of the
circuit is
(1)
(2)
(3)
(4)

Correct Answer : (1)

Explanation : The inductances are in parallel

287. The capacitance of a metallic sphere will be , if its radius is nearly

(1)
(2)
(3)
(4)

Correct Answer : (1)


Explanation :

288. An ideal gas is expanded adiabatically. How many times has the gas to be
expanded to reduce the root mean square velocity of molecules 2.0 times?

(1) 4 times
(2) 16 times
(3) 8 times
(4) 2 times

Correct Answer : (2)


Explanation :
is to reduce two times, , the temperature of the gas will have to reduce force times or

During adiabatic process,

or

289. and are the velocities of sound at the same temperature in two monoatomic gases of

densities and respectively. If then the ratio of velocities and will be

(1)
(2)
(3)
(4)

Correct Answer : (3)


Explanation : At given temperature and pressure

290. In the circuit shown here and battery The switch is first
closed. It is then opened and afterwards is closed. What is the charge finally on
(1)
(2)
(3)
(4)

Correct Answer : (3)

Explanation : Common potential


So, charge on capacitor

291. The resistance of a resistance thermometer has


values and at and . The temperature at which the resistance
is is

(1)
(2)
(3)
(4)

Correct Answer : (2)


Explanation : Change in resistance corresponds to interval of
temperature
So change in resistance
Corresponds to change in temperature

292. A bulb working on and a bulb working on have


(1) Resistances in the ratio of 4 :1

(2) Maximum current ratings in the ratio of 1 :4

(3) Resistances in the ratio of 2 :1

(4) Maximum current ratings in the ratio of 1 :2

Correct Answer : (2)

Explanation :

Maximum current rating

So and

293. An alternating mf is applied across a parallel combination of a resistance ,


capacitance and an inductance . If are the current through and respectively,
then the diagram which correctly represents the phase relationship among and
source mf , is given by

(1)

(2)

(3)

(4)
Correct Answer : (3)

Explanation : lags behind by a phase of while leads by a phase of

294. Two thermometers and are exposed in sun light. The valve of is pointed black, but
that of is not pointed. The correct statement regarding this case is

(1) Temperature of will rise faster than but the final temperature will be the same in both

(2) Both and show equal rise in beginning

(3) Temperature of will remain more than

(4) Temperature of will rise faster

Correct Answer : (1)


Explanation : As for a black body rate of absorption of heat is more. Hence
thermometer shows faster rise in temperature but finally both will acquire the atmosphere
temperature

295. A man having height 6 m, observes image of 2 m height erect, then mirror used is

(1) Concave
(2) Convex
(3) Plane
(4) None of the above

Correct Answer : (2)


Explanation : When an object is placed in front of convex mirror, then for all positions of
object, convex mirror forms erect and diminished image of that object. So, it is obvious that
man seeing his image shorter than his height uses convex mirror.

296. A radioactive nucleus can decay simultaneously by two different processes which have
decay constant .The effective decay constant of the nuclide is where
(1)
(2)

(3)
(4)

Correct Answer : (1)


Explanation : As disintegration by two different processes is simultaneous, therefore, effective
decay constant

297. Efficiency of Carnot engine is 100% if

(1)
(2)
(3)
(4)

Correct Answer : (2)

Explanation : for 100%, efficiency which gives

298. Assuming earth to be a sphere of radius R, if is value of acceleration due to gravity at


latitude of and at the equator, the value of is

(1)

(2)
(3)

(4)
Correct Answer : (2)
Explanation : The value of acceleration due to gravity at latitude λ is given by

299. A circular current carrying coil has a radius The distance from the centre of the coil on

the axis of the coil, where the magnetic induction is th of its value at the centre of the coil is

(1)
(2)

(3)

(4)

Correct Answer : (1)


Explanation : For a circular coil of radius carrying a current the magnetic field at
point distance from coil is given by

At the centre of coil


Given,

Given,

300. Two equal mass and are hung from balance whose scale pans differ in vertical height
by . Calculate the error in weighing. If any, in terms of density of earth .

(1)

(2)

(3)

(4)

Correct Answer : (2)


Explanation : As with height varies as
and in according with figure so will be lesser than and

or

or

301. Which of the following quantities measured from different inertial reference frames are
same

(1) Force
(2) Velocity
(3) Displacement
(4) Kinetic energy

Correct Answer : (1)

302. , where is speed and is displacement. The unit of is

(1)
(2)
(3)
(4)

Correct Answer : (1)


Explanation : is unitless. Unit of is

303. If the temperature is raised by 1 K from 300 K the percentage change in the speed of
sound in the gaseous mixture is (R=8.31 J/mol-K)

(1) 0.167%
(2) 0.334%
(3) 1%
(4) 2%

Correct Answer : (1)

Explanation : From

304. The density of air at the top of mesosphere in comparison to that of near the earth’s
surface is

(1) times
(2) times
(3) times
(4) times

Correct Answer : (2)


Explanation : The density of air in mesosphere with height decreases from 1/
times that due to the surface of earth

305. A horizontal loop is moved across the pole pieces of a magnet as shown in fig. with a
constant speed . When the edge of the loop enters the pole pieces at time , which
one of the following graphs represents correctly the induced emf in the coil
(1)

(2)

(3)

(4)

Correct Answer : (4)


Explanation : When loop enters in field between the pole pieces, flux linked with the coil first
increases (constantly) so a constant emf induces. When coil enters completely within the field,
there is no flux change, so
When coil exists, flux linked with the coil decreases, hence again emf induces, but in opposite
direction

306. Two parallel long wires carry currents and with . When the currents are in the
same direction, the magnetic field midway between the wires is . When the direction
of is reversed, it becomes . The ratio is

(1)
(2)
(3)
(4)

Correct Answer : (4)


Explanation : Initially when wires carry currents in the same direction as shown:
Magnetic field at mid point due to wires 1 and 2 are respectively

and

Hence net magnetic field at

If the direction of is reversed then

and

So

Dividing equation (ii) by (i)

307. Two springs, of force constants , are connected to a mass as shown. The
frequency of the mass is . If both are made four times their original values, the
frequency of oscillation becomes
(1)
(2)
(3)
(4)

Correct Answer : (4)

Explanation :

308. A particle falls towards earth from infinity. It’s velocity on reaching the earth would be

(1) Infinity
(2)
(3)
(4) Zero

Correct Answer : (2)


Explanation : This should be equal to escape velocity

309. An ac supply gives which passes through a resistance. The power


dissipated in it is

(1)
(2)
(3)
(4) 45

Correct Answer : (2)

Explanation :

310. Boiling water is changing into steam. At this stage then specific heat of water is

(1)
(2)
(3)
(4)

Correct Answer : (2)

Explanation : hence becomes

311. Magnetic field at the centre of a circular loop of area is . The magnetic moment of the
loop will be

(1)

(2)

(3)

(4)

Correct Answer : (4)


Explanation :

Also,

Magnetic moment,

312. A 100 W bulb and two 60 W bulbs are connected to a 250 V source as shown
in figure. Now are the output powers of the bulbs respectively, then

(1)
(2)
(3)
(4)

Correct Answer : (4)

Explanation : As resistance of a bulb

Hence
Now the combined potential difference across is same as the potential difference
across . Hence, is more than , being in series, carry same current and ,
therefore ,

313. Flash light equipped with a new set of batteries, produces bright white light. As the
batteries were out
(1) The light intensity gets reduced with no change in its colour

(2) Light colour changes first to yellow and then red with no change in intensity

(3) It stops working suddenly while giving white light

(4) Colour changes to red and also intensity gets reduced

Correct Answer : (4)


Explanation : As batteries wear out, temperature of filament of flash light attains a lesser
value, therefore intensity of radiation reduces. Also dominating wavelength in spectrum,
which is the red colour, increases.

314. The periodic time of a communication satellite is

(1)
(2)
(3)
(4)

Correct Answer : (4)

315. If the kinetic energy of a body is increased 2 times , its momentum will

(1) Half
(2) Remain unchanged
(3) Be doubled
(4) increase times

Correct Answer : (4)

Explanation : Kinetic energy of particle ,

When kinetic energy =2k


316. How much water should be filled in a container in height, so that it appears half
filled when viewed from the top of the container (given that )

(1) 8.0
(2) 10.5
(3) 12.0
(4) None of the above

Correct Answer : (3)


Explanation : To see the container half-filled from top, water should be filled up to height so
that bottom of the container should appear to be raised upto height
As shown in figure apparent depth
Real depth

317. The magnetic moment of a diamagnetic atom is

(1) Much greater than one


(2) 1

(3) Between zero and one


(4) Equal to zero

Correct Answer : (4)


318. Two bars of circular cross-section and of same volume and made of the same
material are subjected to tension. If the diameter of A is half that of B and if the force applied to
both the rods is the same and it is in the elastic limit, the ratio of extension of A to that of B will
be

(1) 16
(2) 8
(3) 4
(4) 7

Correct Answer : (1)

Explanation :

Now,

319. A pulse of a wave train travels along a stretched string and reaches the fixed end of the
string. It will be reflected with

(1) A phase change of 180° with velocity reversed

(2) The same phase as the incident pulse with no reversal of velocity

(3) A phase change of 180° with no reversal of velocity

(4) The same phase as the incident pulse but with velocity reversed

Correct Answer : (3)


Explanation : A pulse of a wave train when travels along a stretched string and reaches the
fixed end of the string, then it will be reflected back to the same medium and the reflected ray
suffers a phase change of π with the incident wave but wave velocity after reflection does not
change.

320. An observer moves towards a stationary source of sound with a velocity one-fifth of the
velocity of sound. What is the percentage increases in the apparent frequency?

(1) Zero
(2) 0.5%
(3) 5%
(4) 20%

Correct Answer : (4)


Explanation : Given,

When observers moves towards the stationary source, then

Hence, percentage increases

321. The escape velocity of a sphere of mass from earth having mass and radius is given
by

(1)
(2)

(3)

(4)

Correct Answer : (1)


Explanation : Escape velocity does not depend on the mass of the projectiles

322. Intensity of -rays depends upon the number of

(1) Electrons
(2) Protons
(3) Neutrons
(4) Positrons

Correct Answer : (1)


Explanation : Intensity of -rays depends upon the number of electron striking the target

323. Half life of a radioactive element is 10 days. The time during which quantity remains 1/10
of initial mass will be

(1) 100 days


(2) 50 days
(3) 33 days
(4) 16 days

Correct Answer : (3)

Explanation :
324. A small particle of mass is projected at an angle with the -axis with an initial

velocity in the - plane as shown in the figure. the angular momentum of


the particle is

(1)
(2)

(3)

(4)

Correct Answer : (3)


Explanation :

325. A 5. F capacitor is charged to a potential difference of 800 V and discharged through a


conductor. the energy given to the conductor during the discharge is
(1)
(2) 3.2 J
(3) 1.6 J
(4) 4.2 J

Correct Answer : (3)

Explanation : Energy given to conductor

= 1.6 J

326. A resistor of , an inductor of L = 1 H and a capacitor of are connected


in series with an AC source. Find the Q - factor.

(1) 3.72
(2) 40
(3) 2.37
(4) 80

Correct Answer : (2)

Explanation :

327. A long solenoid of length has a mean diameter It has layers of winding of turns
each. If it carries a current the magnetic field at its centre will be

(1) Proportional to
(2) Inversely proportional to

(3) Independent of
(4) Proportional to
Correct Answer : (3)
Explanation : Magnetic field due to solenoid is independent of diameter

328. The initial temperature of a body is . If its temperature falls to in and


in to then the temperature of surrounding will be

(1)
(2)
(3)
(4)

Correct Answer : (2)

Explanation : According to Newton’s law


Initially,

Finally

On solving equation (i) and (ii),

329. A particle of mass 200 g executes SHM. The restoring force is provided by a spring of force
constant 80 N/m. The time period of oscillation is

(1) 0.31 s
(2) 0.15 s
(3) 0.05 s
(4) 0.02 s

Correct Answer : (1)


Explanation :

330. Two blocks of equal masses are released from the top of a smooth fixed wedge as
shown in the figure. The acceleration of the centre of mass of the two blocks is

(1)

(2)

(3)

(4)

Correct Answer : (2)


Explanation : The acceleration of the centre of mass of the block,

331. A fixed horizontal wire carries a current of . Another wire having a mass per unit
length is placed below the first wire at a distance of and parallel to it. How
much current must be passed through the second wire if it floats in air without any support?
What should be the direction of current in it

(1) 25 A (direction of current is same to first wire )

(2) 25 A (direction of current is opposite to first wire )


(3) 49 A (direction of current is same to first wire )

(4) 49 A (direction of current is opposite to first wire )

Correct Answer : (3)


Explanation : For floating the second wire

(Direction of current is same to first wir

332. A simple pendulum is oscillating without damping. When the displacement the bob is
less than maximum, its acceleration vector is correctly show in figure.

(1)

(2)

(3)
(4)

Correct Answer : (3)


Explanation : When the displacement of bob is less than maximum, there will two

compounding acceleratins and of the bob as shown in figure. Their resultant


acceleration will be represented by the diagonal of the parallelogram

333. A slab of copper of thickness is inserted in between the plates of parallel plate capacitor
as shown in figure. The separation between the plates is if , then the ratio of
capacities of capacitors after and before inserting the slab will be

(1)
(2) 2 : 1
(3) 1 : 1
(4) 1 :

Correct Answer : (2)

Explanation :
After inserting the slab
334. Three sources of equal intensities with frequencies 400, 401 and 402 vib/s are sounded
together. The number of beats/s is

(1) Zero
(2) 1
(3) 2
(4) 4

Correct Answer : (3)


Explanation : Beats are the periodic and repeating functions heard in the intensity of sound,
when two sound waves of very similar frequency interface with ine another.
Beats = defference in frequencies.
Maximum number of beats =402-400=2

335. What is the potential difference between points in the circuit shown?

(1) 2 V
(2) 4 V
(3) 3 V
(4) 12 V

Correct Answer : (1)


Explanation : Consider the charge distribution as shown. Considering the branch on upper
side, we have

Here,

…(i)

…(ii)
From Eqs. (i) and (ii), we get

Similarly for the lower side branch

…(iii)

...(iv)
From Eqs. (iii) and (iv)
336. A transformer has 100 turns in the primary coil and carries current. If input power is
one kilowatt, the number of turns required in the secondary coil to have output will be

(1) 100
(2) 200
(3) 400
(4) 300

Correct Answer : (3)

Explanation :

337. By using only two resistances coils-singly, in series or in parallel one should be able to
obtain resistance of 3,4,12 and 16 ohm. The separate resistance of the coil are

(1) 3 and 4
(2) 4 and 12
(3) 12 and 16
(4) 16 and 13

Correct Answer : (2)


Explanation : If we take , , then in series resistance

In parallel, resistance

338. Two spherical conductors A and B of radii 1mm and 2mm are separated by a distance of
5cm and are uniformly charged. If the spheres are connected by a conducting wire then in
equilibrium condition, the ratio of the magnitude of the electric fields at the surfaces of spheres
A and B is

(1) 4:1
(2) 1;2
(3) 2:1
(4) 1:4

Correct Answer : (1)


Explanation : When the two conducting spheres are connected by a conducting wire, charge
will flow from one sphere (having higher potential) to other (having lower potential) till both
acquire the same potential.

So,

339. The energy of a photon of light with wavelength is approximately This way
the energy of an -ray photon with wavelength would be

(1)
(2)
(3)
(4)

Correct Answer : (3)

Explanation :

340. If there is change of angular momentum from to in 5 s, then the torque is

(1)

(2)

(3)
(4) None of these

Correct Answer : (2)


Explanation : We know that rate of change of angular momentum of a body is equal to the
external torque acting upon the body.

.
Given,

Hence,

341. A hollow cylinder has a charge coulomb within it. If is the electric flux in units
of associated with the curved surface the flux linked with the plane surface in
units of will be

(1)

(2)

(3)

(4)

Correct Answer : (1)

Explanation :
and [assumed]
342. The resistance across and in the figure.

(1)
(2)
(3)
(4)

Correct Answer : (1)


Explanation : Two resistances of each side of triangle are connected in parallel. Therefore, the

effective resistance of each arm of the triangle would be . The two arms
and are in series and they together are in parallel with third one.

Total resistance

343. A hollow copper sphere and a hollow copper cube , both of negligible thin walls of same
area, are filled with water at and allowed to cool in the same environment. The graph that
correctly represents their cooling is

(1)
(2)

(3)

(4)

Correct Answer : (3)

Explanation :

For given sphere and cube is constant so for both rate of fall of

temperature constant

344. Find equivalent resistance between and

(1)

(2)
(3)
(4)

Correct Answer : (3)


Explanation : Given circuit can be redrawn as follows

345. If the wavelength of incident light changes from 400 nm to 300 nm, the stopping potential
for photoelectrons emitted from a surface becomes approximately

(1) 1.0 V greater


(2) 1.0 V smaller
(3) 0.5 V greater
(4) 0.5 V smaller

Correct Answer : (1)


Explanation :

But being stopping potential. Thus, stopping potential for photoelectrons from a
surface becomes approximately 1.0 V greater.

346. The electric field of an electromagnetic wave travelling through vacuum is given by the
equation . The quantity that is independent of wavelength is

(1)
(2)
(3)
(4)

Correct Answer : (1)


Explanation :
Comparing with standard equation we will get

Wavelength =

347. The velocity of a particle at which the kinetic energy is equal to its rest energy is

(1)

(2)

(3)

(4)

Correct Answer : (4)


Explanation : The relativistic kinetic energy of a particle of rest mass is given by

where is the mass of the particle moving with velocity v.

According to problem
kinetic energy = rest energy

or

348. For harder -rays

(1) The wavelength is higher


(2) The intensity is higher

(3) The frequency is higher


(4) The frequency is higher

Correct Answer : (3)

349. Three capacitors 2, 3 and 6 are joined with each other. What is the minimum effective
capacitance?

(1)
(2) 1
(3) 2
(4) 3

Correct Answer : (2)


Explanation : Minimum effective capacitance

350. A body of moment of inertia of - rotating with an angular velocity of has


the same kinetic energy as a mass of moving a velocity of

(1)
(2)
(3)
(4)

Correct Answer : (4)

Explanation :

351. The ratio of the wavelengths for 2 1 transition in and H is

(1) 1:2:3

(2)
(3) 1:4:1
(4) 3:2:1

Correct Answer : (2)

Explanation : From Bohr’s formula , the wave number is given by


where is atomic number, the Rydberg’s constant and the quantum number.

352. The work done in carrying a charge of from a point to a point in an electric field
is The potential difference is then

(1)
(2)
(3)
(4)

Correct Answer : (1)

Explanation : Work done

353. When one electron is taken towards the other electron, then the electric potential energy
of the system

(1) Decreases
(2) Increases
(3) Remains unchanged
(4) Becomes zero

Correct Answer : (2)


Explanation : Potential energy of the system will be given by

As decreases, potential energy increases

354. An object is puts at a distance of 5 cm from the first focus of a convex lens of focal length
10 cm. If a real image is formed, its distance from the lens will be
(1) 15 cm
(2) 20 cm
(3) 25 cm
(4) 30 cm

Correct Answer : (4)

Explanation :

Or

Or
Or

355. If mass of He is 4 times that of hydrogen, then mean velocity of He is

(1) 2 times of H-mean value

(2) times of H-mean value

(3) 4 times of H-mean value

(4) Same as H-mean value

Correct Answer : (2)


Explanation : Average speed or mean speed of gas molecules

or
Here,
356. A mass is vertically suspended from a spring of negligible mass; the system oscillates
with a frequency What will be the frequency of the system if a mass is suspended from
the same spring

(1)
(2)
(3)
(4)

Correct Answer : (3)

Explanation :

357. The work function of a metal is

(1) The energy for the electron to enter into the metal

(2) The energy for producing -ray

(3) The energy is required for an electron to come out from metal surface

(4) None of these

Correct Answer : (3)

358. The wavelength of infrared rays is of the order of

(1)
(2)
(3) Diverge more
(4) None of these

Correct Answer : (3)


Explanation : The wavelengths of infrared rays lie between to 0.004 cm

359. A piece of wood has dimensions Its relative density is d. It is floating in water
such that the side c is vertical. It is now pushed down gently and released. The time period is

(1)

(2)

(3)

(4)

Correct Answer : (4)


Explanation : Let the distance of vertical disc of block be pushed in liquid, when block is
floating, then Buoyancy force

The mass of piece of wood

So acceleration

Hence, time period,

360. Which of the following shows green house effect?

(1) Ultraviolet rays


(2) Infrared rays
(3) X-rays
(4) None of these

Correct Answer : (2)

361. The strain-stress curves of three wires of different materials are shown in the
figure. and are the elastic limits of the wires. The figure shows that

(1) Elasticity of wire is maximum


(2) Elasticity of wire is maximum

(3) Tensile strength of is maximum


(4) None of the above is true

Correct Answer : (4)


Explanation : As stress is shown on -axis and strain on -axis

So we can say that


So elasticity of wire is minimum and of wire is maximum

362. The direction of induced current is such that it opposes the very cause that has produced
it. This is the law of

(1) Lenz
(2) Faraday
(3) Kirchhoff
(4) Fleming

Correct Answer : (1)

363. The wavelength of the first line of Lyman series for hydrogen atom is equal to that of the
second line of Balmer series for a hydrogen like ion. The atomic number of hydrogen like ion
is

(1) 2
(2) 3
(3) 4
(4) 1

Correct Answer : (1)

Explanation :

364. Two magnets held together in earth’s magnetic field with same polarity together make

12 when opposite poles together make 4 ratio of magnetic


moments is

(1) 9 :1
(2) 1 : 3
(3) 1 : 9
(4) 10 : 8

Correct Answer : (4)

Explanation :

365. A bullet is fired with a speed of in order to hit a target away.


If , the gun should be aimed

(1) Directly towards the target


(2)
(3)
(4)

Correct Answer : (2)

Explanation : Bullet will take to reach target.


During this period vertical distance (downwartravelled by the

bullet
So the gun should be aimed above the target

366. The hollow shaft is….. than a solid shaft of same mass, material and length.

(1) Less stiff


(2) More stiff
(3) Squally stiff
(4) None of these

Correct Answer : (1)

367. An ideal gas heat engine operates in a Carnot’s cycle between It


absorbs at high temperature. The amount of heat converted into work is

(1)
(2)
(3)
(4)

Correct Answer : (2)


Explanation : Using the relation
368. At a temperature of C, the susceptibility of a ferromagnetic material is found to be X.
Its susceptibility at C is

(1)
(2)
(3)
(4)

Correct Answer : (4)


Explanation : Susceptibility of ferromagnetic material is inversely proportional to temperature

Given ,

369. If no external voltage is applied across - junction, there would be

(1) No electric field across the junction

(2) An electric field pointing from -type to -type side across the junction
(3) An electric field pointing from -type to -type side across the junction

(4) A temporary electric field during formation of - junction that would subsequently
disappear

Correct Answer : (2)


Explanation : Across the junction, a barrier potential is developed whose direction is
from region to region

370. A parallel plate capacitor is charged to . Due to a radioactive source, the plate loss
charge at the rate of The magnitude of displacement current is

(1)
(2)
(3)
(4)

Correct Answer : (1)


Explanation : Displacement current is given by

371. The output of a NAND gate is 0

(1) If both inputs are 0

(2) If one input is 0 and the other input is 1

(3) If both inputs are 1

(4) Either if both inputs are 1 or if one of the inputs is 1 and the other 0

Correct Answer : (3)


Explanation : If inputs are and then output for NAND gate is
If

372. Pure at has equal number of electron and hole concentrations


of . Doping by indium increases to . The doped semiconductor
is of

(1) type with electron concentration

(2) type having electron concentration

(3) type with electron concentration

(4) type with electron concentration

Correct Answer : (2)


Explanation :

Semiconductor is -type and

373. Kepler discovered

(1) Laws of motion


(2) Laws of rotational motion

(3) Laws of planetary motion


(4) Laws of curvilinear motion

Correct Answer : (3)


374. A wire of length 50 cm moves with a velocity of 300 m-mi perpendicular to a magnetic
field. If the emf induced in the wire is 2 V, the magnitude of the field in telsa is

(1) 2
(2) 5
(3) 0.4
(4) 0.8

Correct Answer : (4)


Explanation : Emf induced in the wire is given by

Given,

min-1=

Magnetic field,

375. On which principle does sonometer works?

(1) Hooke’s law


(2) Elasticity
(3) Resonance
(4) Newton’s law

Correct Answer : (3)


Explanation : Sonometer works on the principle of resonance. At resonance the wire of
sonometer vibrate with maximum amplitude.

376. Ionization potential of hydrogen atom is13.6 . Hydrogen atoms in the ground state are
excited by monochromatic radiation of photon energy 12.1 . According to Bohr’s theory, the
spectral lines emitted by hydrogen will be

(1) Two
(2) Three
(3) Four
(4) One

Correct Answer : (2)


Explanation : Ionization energy corresponding to ionization potential

Photon energy incident


So,the energy of electron in excited state

energy of electron in excited state corresponds to third orbit.


The possible spectral lines are when electron jumps from orbit 3rd to 2nd; 3rd to 1st and 2nd to
1st. Thus, 3 spectral lines are emitted.

377. A body of mass is observed to fall with an acceleration of . The opposing


force of air on the body is

(1)
(2)
(3)
(4) Zero

Correct Answer : (1)


Explanation : Here, mass of the body

Acceleration
378. Nickel shows ferromagnetic property at room temperature. If the temperature is
increased beyond Curie temperature, then it will show

(1) Paramagnetism
(2) Anti-ferromagnetism

(3) No magnetic property


(4) Diamagnetism

Correct Answer : (1)


Explanation : If the temperature of a ferromagnetic material is raised above a certain critical
value, called the Curie temperature, the exchange coupling ceases to be effective. Most such
materials then become simply paramagnetic; that is, the dipoles still tend to align with an
external field but much more weakly, and thermal agitation can now more easily disrupt the
alignment.

379. Image formed by a convex mirror is

(1) Virtual
(2) Real
(3) Enlarged
(4) Inverted

Correct Answer : (1)


Explanation : Image formed by convex mirror is virtual for real object placed anywhere

380. Two lenses having has combination to make no dispersion. Find the ratio of
dispersive power of glasses used

(1)
(2)
(3)
(4)
Correct Answer : (1)

Explanation :

381. Three charges each of + 1 are placed at the corners of an equilateral triangle. If the force
between any two charges be F, then the net force on either charge will be

(1)
(2)
(3) 2F
(4) 3F

Correct Answer : (2)


Explanation : Angle between two forces due to individual charges is equal to 600

382. A perfectly reflecting mirror has an area of 1 cm2 Light energy is allowed to fall on it for 1h
at the rate of 10 Wcm-2. The force that acts on the mirror is

(1)
(2)
(3)
(4)

Correct Answer : (2)


Explanation : Let Energy falling on the surface per second = 10 J
Momentum Of photons

On reflection,

Change in momentum per second


We know that,
Change in momentum per second = force

383. A piece of semiconductor is connected in series in an electric circuit. On increasing the


temperature, the current in the circuit will

(1) Decrease
(2) Remain unchanged
(3) Increase
(4) Stop flowing

Correct Answer : (3)


Explanation : Because with rise in temperature, resistance of semiconductor decreases, hence
overall resistance of the circuit decreases which in turn increases the current in the circuit

384. In an experiment, to measure the height of a bridge by dropping stone into water
underneath, if the error in measurement of time is at the end of , then the error in
estimation of height of bridge will be

(1)
(2)
(3)
(4)

Correct Answer : (3)

Explanation : From
385. Which one has the dimensions different from the remaining three

(1) Power
(2) Work
(3) Torque
(4) Energy

Correct Answer : (1)

Explanation : Power =

386. If the sum of the two unit vectors is also a unit vector, then magnitude of their difference is

(1)
(2)
(3)
(4)

Correct Answer : (2)


Explanation : Let then using parallelogram law of vectors we have

or

or

or
or
.
Now the angle between is 60
The resultant of

=
387. The electric field at a distance from the centre of a charged conducting spherical shell of

radius The electric field at a distance from the centre of the sphere is

(1) Zero
(2)

(3)

(4)

Correct Answer : (1)


Explanation : Electric field inside shell is zero

388. The fundamental frequency of a sonameter wire is v. if its radius is doubled and its tension
becomes half, the material of the wire remains same, the new fundamental frequency will be

(1) V

(2)

(3)

(4)

Correct Answer : (4)


Explanation : Frequency of sonometer wire is given by

Where m is mass of string per unit length, and T is tension in the string.
Also, m=πr2d
R being radius of string per unit length, and T is tension in the string.
So,

Or

Or
Hence,

Or

389. If the wavelength of light is then the number of waves in length will be

(1) 25
(2) 0.25
(3)
(4)

Correct Answer : (3)

Explanation : Number of waves

390. A tuning fork of frequency resonates with length of a string under


tension . If length of the string is decreased by 2%, keeping the tension constant, the
number of beats heard when the string and the tuning fork made to vibrate simultaneously is

(1) 4
(2) 6
(3) 8
(4) 12

Correct Answer : (3)

Explanation :
If length is decreased by 2% then frequency increases by 2%
391. A bar magnet of magnetic moment M and moment of inertia I is freely suspended such
that the magnetic axial line is in the direction of magnetic meridian. If the magnet is displaced
by a very small angle ( ), the angular acceleration is (Magnetic induction of earth’s horizontal
field )

(1)

(2)

(3)

(4)

Correct Answer : (1)


Explanation : When magnet is displaced by a very small angle then restoring couple acting
on the magnet is

Negative sing shows the restoring nature of torque. Now since and for small
angular displacement

Or angular acceleration

392. A 100 kg block is suspended with the help of three string and The tension in the
string is
(1) 50
(2) 100
(3) 20
(4) 20

Correct Answer : (2)


Explanation : g

393. Young’s modulus of perfectly rigid body material is

(1) Infinite
(2) Zero
(3)
(4)

Correct Answer : (1)


Explanation : Young’s modulus of a material is given by

For a perfectly rigid body,

(infinit
394. The maximum peak to peak voltage of an AM wire is and the minimum peak to
peak voltage is . The modulation factor is

(1) 10%
(2) 20%
(3) 25%
(4) 50%

Correct Answer : (4)

Explanation : Here, and

Now,

395. A couple produces

(1) No motion
(2) Linear and rotational motion

(3) Purely rotational motion


(4) Purely linear motion

Correct Answer : (3)


Explanation : A couple consists of two equal and opposite forces acting at a separation, so that
net force becomes zero. When a couple acts on a body it rotates the body but does not produce
any translatory motion. Hence, only rotational motion is produced.

396. Two satellites of earth, are moving in the same orbit. The mass of is four
times the mass of Which one of the following statements is true?

(1) The time period of is four times that of

(2) The potential energies of earth and satellite in the two cases are equal
(3) and are moving with the same speed

(4) The kinetic energies of the two satellites are equal

Correct Answer : (3)


Explanation : When two satellite of earth are moving in same orbit, then time period of both
are equal. From Kepler’s third law

Time period is independent of mass, hence their time periods will be equal.
The potential energy and kinetic energy are mass dependent, hence the PE and KE of satellites
are not equal.
But, if they are orbiting in a same orbit, then they have equal orbital speed.

397. All capacitors used in the diagram are identical and each is of capacitance Then the
effective capacitance between the points and is

(1)
(2)
(3)
(4)

Correct Answer : (1)

398. A bullet of mass and velocity is fired into a large block of mass . The final velocity of
the system is

(1)

(2)

(3)
(4)

Correct Answer : (2)

Explanation :
Initially bullet moves with velocity and after collision bullet get embedded in block and both
move together with common velocity
By the conservation of momentum

399. A solid copper sphere (density and specific heat capacity ) of radius at an initial
temperature is suspended inside a chamber whose walls are at almost . The time
required (in ) for the temperature of the sphere to drop to is

(1)

(2)

(3)

(4)

Correct Answer : (2)

Explanation : [In the given problem fall in temperature of


body , temp. of surrounding , Initial temperature of
body ]
400. The impact of an image on the retina remains for

(1)
(2)
(3)
(4)

Correct Answer : (1)

401. A thin spherical conducting shell of radius R has a charge q. Another charge is placed at
the centre of the shell. The electrostatic potential at a point P at a distance R/2 from the centre
of the shell is

(1)

(2)

(3)

(4)

Correct Answer : (3)


Explanation : At due to shell, potential

at due to , potential

Net potential at P
402. A monkey can jump a maximum horizontal distance of 20 m. Then the velocity of the
monkey is

(1)
(2)
(3)
(4)

Correct Answer : (2)

Explanation :

403. In the above question, if , the gas may

(1) Monoatomic
(2) Diatomic

(3) A mixture of monoatomic and diatomic gases


(4) A mixture of diatomic and triatomic gases

Correct Answer : (3)

Explanation : For monatomic gas,

And for diatomic gas


As actual . Therefore, gas must be a mixture of monoatomic and diatomic gases
404. In the following figure, the charge on each condenser in the steady state will be

(1)
(2)
(3)
(4)

Correct Answer : (4)


Explanation : In steady state current flows through resistance only and it

is Potential difference across resistance is


Hence, potential difference across each capacitor is
So charge on each capacitor

405. An electron enters the space between the plates of a charged capacitor as shown. The
charge density on the plate is Electric intensity in the space between the plates is .A uniform
magnetic field also exists in the space perpendicular to the direction of The electron moves
perpendicular to both E and B without any change in direction. The time taken by the electron
to travel a distance in the space is

(1)

(2)

(3)
(4)

Correct Answer : (3)


Explanation : Here, magnetic force =electrostatic force

The time taken by electron to travel a distance in that space

406. The reading of the ammeter as per figure shown is

(1)

(2)

(3)
(4)

Correct Answer : (2)

Explanation : Resistance across

Total resistance
Current through ammeter

407. In the following, which one of the diodes is reverse biased?

(1)

(2)

(3)

(4)

Correct Answer : (4)


Explanation : For reverse biasing of an ideal diode, the potential of -side should be higher
than potential of -side. Only option (is satisfying the criterion for reverse biasing.

408. A wire can be broken by applying a load of 200 N. The force required to break another wire
of the same length and same material, but double in diameter, is

(1) 200 N
(2) 400 N
(3) 600 N
(4) 800 N

Correct Answer : (4)


Explanation : Young’s modulus

or
or

Given,

or

409. Three rods each of length and mass are placed along and axes in such a way
that one end of each rod is at the origin. The moment of inertia of the system about -axis is

(1)

(2)

(3)

(4)

Correct Answer : (2)

Explanation : Moment of inertia of a rod about one end


As,
410. The acceleration of system of two bodies over the wedge as shown in figure is

(1)
(2)
(3)
(4)

Correct Answer : (1)

Explanation :
Let be the tension in the string
Let be acceleration of the system
The equations of motion are
…(i)
And …(ii)
Adding (i) and (ii, we get

411. A parallel beam of white light falls on a convex lens. Images of blue, yellow and red light
are formed on other side of the lens at a distance of and respectively.
The dispersive power of the material of the lens will be
(1) 619/1000
(2) 9/200
(3) 14/205
(4) 5/214

Correct Answer : (3)


Explanation : For a lens

412. A hollow metal sphere of radius 10 cm is charged such that the potential on its surface
becomes 80V. The potential at the centre of the sphere is

(1) 80 V
(2) 800 V
(3) 8 V
(4) Zero

Correct Answer : (1)


Explanation : The potential at the centre of the sphere is 80 V because it remains same at each
point under the metallic hollow sphere.

413. Find the luminous intensity of the sun if it produces the same illuminance on the earth as
produced by a bulb of at a distance . The distance between the sun and
the earth is

(1)
(2)
(3)
(4)

Correct Answer : (3)


Explanation : Illuminance produced by the sun

Illuminance produce by the bulb

According to problem

414. Two short magnets with pole strengths of 900 ab amp-cm and 100 ab-amp-cm are placed
with their axes in the same vertical line, with similar poles facing each other. Each magnet has a
length of 1 cm. When separation between the nearer poles is 1 cm, the weight of upper magnet
is supported by the repulsive force between the magnets. If g , then the mass of
upper magnet is

(1) 100 g
(2) 55 g
(3) 45 g
(4) 77.5 g

Correct Answer : (2)

Explanation : In CGS system,


In equilibrium, net repulsion due to magnetic interaction = weight of upper magnet. Therefore,
as is clear from figure.
g

415. The phase difference between two points separated by 0.8 m in a wave of frequency 120
Hz is . The wave velocity is

(1)
(2)
(3)
(4)

Correct Answer : (2)


Explanation : Here ,
.

From

416. A parallel plate capacitor of capacitance 100 pF is to be constructed by using paper sheets
of 1 mm thickness as dielectric. If the dielectric constant of paper is 4, the number of circular
metal foils of diameter 2 cm each required for the purpose is

(1) 40
(2) 20
(3) 30
(4) 10

Correct Answer : (4)


Explanation : The arrangement of metal plates separated by dielectric acts as parallel
combination of capacitors.

Therefore,
Here,
or

417. The potential at a point P which is forming a corner of a square of side 93mm with
charges, nC, located at the other three corners is nearly

(1) 16kV
(2) 4kV
(3) 400V
(4) 160V

Correct Answer : (2)

Explanation :

418. A tennis ball bounces down flight of stairs striking each step in turn and rebounding to the
height of the step above. The coefficient of restitution has a value

(1)
(2) 1
(3) 1/
(4) 1/2

Correct Answer : (3)


Explanation : As shown in adjoining figure ball is falling from height 2 and rebounding to a
height only. It means that velocity of ball jus before collision

and velocity just after collision

419. A small object placed on a rotating horizontal turn table just slips when it is placed at a
distance of 4 cm from the axis of rotation, if the angular velocity of the turn table is doubled the
object slips when its distance from the axis of rotation is

(1) 1 cm
(2) 2 cm
(3) 4 cm
(4) 8 cm

Correct Answer : (1)


Explanation : The object will slip if centripetal force acting on it is more than friction force.
420. The number of telephonic message which are carried by a fiber at an instant with much
less intensity loss are

(1) 2400
(2) 2200
(3) 2000
(4) 1800

Correct Answer : (3)


Explanation : 2000 number of telephonic messages can be carried by a optical fiber at an
instant

421. A coil of number of turns is wound tightly in the form of a spiral with inner and outer
radii and respectively. When a current of strength is passed through the coil, the magnetic
field at its centre is

(1)

(2)

(3)

(4)

Correct Answer : (4)


Explanation : Consider an element of thickness at a distance from the centre of spiral coil.

Number of turns in coil


Number of turns per unit length

Number of turns in element


Number of turns per unit length in element

Magnetic field at its centre due to element is

422. Two stretched strings have length 𝒍 and 2𝒍 while tensions are T and 4T respectively. If they
are made of same material the ratio of their frequencies is

(1) 2:1
(2) 1:2
(3) 1:1
(4) 1:4

Correct Answer : (3)


Explanation : Frequency is given by

∴first frequency

And second frequency

∴hence, the ratio of frequencies


423. The dimension of is that of

(1) Velocity
(2) Time
(3) Capacitance
(4) Distance

Correct Answer : (1)

Explanation : velocity of light

424. A thin rod of length and mass is bent at the middle point at an angle of . The
moment of inertia of the rod about an axis passing through and perpendicular to the plane of
the rod will be

(1)

(2)

(3)

(4)

Correct Answer : (2)

Explanation : Moment of inertia of a uniform rod about one end


Moment of inertia of the system
425. The mass and length of a wire are and respectively. The density of the material of the
wire is . On applying the force on the wire, the increase in length is , then the Young’s
modulus of the material of the wire will be

(1)

(2)

(3)

(4)

Correct Answer : (4)

Explanation :

As Volume density

426. A police car moving at , changes a motorcyclist. The police man sounds his horn at
176 Hz, while both of them move towards a stationary siren of frequency 165 Hz. Calculate the
speed of the motorcycle, if it is given that he does not observe any beats.

(1)
(2)
(3) Zero
(4)
Correct Answer : (2)
Explanation : The motorcyclist observes no beats. So the apparent frequency observed by him
from the two sources must be equal.

Solving this equation we get,

427. If a bar is made of copper whose coefficient of linear expansion is one and a half times
that of iron, the ratio of the force developed in the copper bar to the iron bar of identical
lengths and cross-sections, when heated through the same temperature range (Young’s
modulus for copper may be taken equal to that of iron) is

(1) 3/2
(2) 2/3
(3) 9/4
(4) 4/9

Correct Answer : (1)


Explanation :

428. The element used for radioactive carbon dating for more than 56000 yr is

(1) C-14
(2) U-234
(3) U-238
(4) Po-94

Correct Answer : (1)


Explanation : C-14 is the element used in radioactive carbon dating
429. The current in an induction coil varies with time according to the graph shown

in figure. Which of the following graphs shows the induced emf ( ) in the coil with time

(1)

(2)

(3)

(4)

Correct Answer : (3)


Explanation : Emf induces during ‘ ’ = 0
emf induces during ‘ ’ is constant throughout emf induces during ‘ ’ is constant throughout
magnitude of emf induced during ‘ ’ is equal to the magnitude of emf induced during ‘ ’. But
the direction opposite
430. For an electron in the second orbit of Bohr’s hydrogen atom, the moment of linear
momentum is

(1)
(2) 2

(3)

(4)

Correct Answer : (4)


Explanation : The moment of linear momentum is angular momentum

=
Here, =2

431. What is the name of the level formed due to impurity atom in -type in the forbidden gap?

(1) Donor level


(2) Acceptor level
(3) Conduction level
(4) Forbidden level

Correct Answer : (2)


Explanation : In -type doping, the impurities have left holes in the atomic structure which
tend to attract and hold free electrons. This pulls the fermi level down until it gets close to the
valence bond. Hence, name of level formed due to impurity atom is -type in the forbidden gap
is acceptor level.
432. Two small conducting spheres of equal radius have
charges and respectively and placed at a distance from each other
experience force . If they are brought in contact and separated to the same distance, they
experience force . The ratio of to is

(1) 1 : 8
(2)
(3) 1 : 2
(4)

Correct Answer : (2)

Explanation :

433. Relative permeability of iron is 5500, then it magnetic susceptibility will be

(1)
(2)
(3) 5501
(4) 5499

Correct Answer : (4)


Explanation :

434. A body slides down a frictionless track which ends in a circular loop of diameter . Then
the minimum height of the body in terms of so that it may just complete the loop, is

(1)
(2)

(3)
(4)

Correct Answer : (3)


Explanation : For looping the loop, the velocity at the lowest point of loop should be
or

435. When a mass hangs attached to a spring of length , the spring stretches
by . The mass is pulled down until the length of the spring becomes . What is the
amount of elastic energy stored in the spring in this condition, if

(1)
(2)
(3)
(4)

Correct Answer : (2)


Explanation : Force constant of a spring

Increment in the length

436. The ratio where and are the accelerations due to gravity at the surface of the
earth and at a height above the earth’s surface respectively, is

(1)
(2)

(3)

(4)

Correct Answer : (1)


Explanation : The value of acceleration due to gravity at height above the surface of earth
is

Where is radius of earth.

437. In the given figure, a hollow spherical capacitor is shown. The electric field will not be zero
at

(1)
(2)
(3)
(4)

Correct Answer : (4)


Explanation : The electric field of a hollow spherical capacitor is localised in between inner and
outer surface of the spherical conductor.
Therefore, at point , the electric field will not be zero.

438. Two bodies of mass 4 kg and 6 kg are attached to the ends of a string passing over a
pulley. The 4 kg mass is attached to the table by another string. The tension in this string is

(1) 19.6 N
(2) 25 N
(3) 10.6 N
(4) 10 N

Correct Answer : (1)


Explanation : For body of mass 6 kg
N
For body of mass 4 kg
N

439. Four curves and are drawn in the adjoining figure for a given amount of gas. The
curves which represent adiabatic and isothermal changes are

(1) and respectively


(2) and respectively
(3) and respectively
(4) and respectively

Correct Answer : (3)


Explanation : As we know that slope of isothermal and adiabatic curves are always negative
and slope of adiabatic curve is always greater than that of isothermal curve
Hence in the given graph curve and represents adiabatic and isothermal changes
respectively

440. First Bohr radius of an atom with is . Radius of its third orbit is

(1) 9
(2) 6
(3) 3
(4)

Correct Answer : (1)


Explanation : Radius of Bohr's Orbit is

441. In the adjacent shown circuit, a voltmeter of internal resistance when connected

across and reads Neglecting the internal resistance of the battery, the value of is

(1)
(2)
(3)
(4)

Correct Answer : (3)


Explanation : Internal resistance of voltmeter is
Therefore effective resistance across and

is given by

According to ohm’s law

…(ii)
Now, total resistance of circuit

Now,

442. Work done in the given cyclic process is


(1)
(2)
(3)
(4) zero

Correct Answer : (3)


Explanation : For cyclic process p-V curve is closed curve and area enclosed by closed path
represent the work done.

443. Momentum of a photon is The corresponding wavelength is

(1)
(2)
(3)
(4)

Correct Answer : (4)


Explanation : hence =

444. A steel wire can withstand a load up to 2940 N. A load of 150 kg is suspended from a rigid
support. The maximum angle through which the wire can be displaced from the mean position,
so that the wire does not break when the load passes through the position of equilibrium, is

(1)
(2)
(3)
(4)

Correct Answer : (2)


Explanation :

In equilibrium,

445. In order to obtain a time constant of 10 s in a circuit containing a resistance of ,


the capacity of the condenser should be

(1)
(2) 100
(3) 1000
(4) 10000

Correct Answer : (4)

Explanation : F

446. If denotes the inductance of an inductor through which a current is flowing, then the
dimensional formula of is

(1)
(2)

(3)
(4) Not expressible in terms of

Correct Answer : (2)


Explanation : Both and represent energy.

447. 10 moles of an ideal monoatomic gas at is mixed with 20 moles of another


monoatomic gas at Then the temperature of the mixture is

(1)
(2)
(3)
(4)

Correct Answer : (4)


Explanation :

448. In beta decay

(1) The parent and daughter nuclei have same number of protons

(2) The daughter nucleus has one proton less than the parent nucleus

(3) The daughter nucleus has one proton more than the parent nucleus

(4) The daughter nucleus has one neutron more than the parent nucleus

Correct Answer : (3)

449. At a common temperature, a block of wood and a block of metal feel equally cold or hot.
The temperature of block of wood and block of metal are

(1) Equal to temperature of the body


(2) Less than the temperature of the body
(3) Greater than temperature of the body
(4) Either [b] or [c]

Correct Answer : (1)


Explanation : When the temperature of an object is equal to that of human body, no heat is
transferred from the object to body and vice versa. Therefore block of wood and block of metal
feel equally cold and hot if they have same temperature as human body

450. A binary star consists of two stars (mass 2.2 ) and B (mass 11 ), where is the
mass of the sun. They are separated by distance and are rotating about their centre of mass,
which is stationary. The ratio of the total angular momentum of the binary star to the angular
momentum of star about the centre of mass is

(1) 7
(2) 6
(3) 9
(4) 10

Correct Answer : (2)

Explanation : (as will be same in both cases)

(as

451. A double convex lens made out of glass (refractive index has both radii of
curvature of magnitudes 20 cm. Incident light rays parallel to the axis of this lens will converge
at a distance such that

(1) cm

(2)
(3)
(4)
Correct Answer : (4)

Explanation : Use where,

452. The driver of a car travelling with speed towards a hill sounds a horn
of frequency If the velocity of sound in air is the frequency of
the reflected sound as heard by the driver is

(1)
(2)
(3)
(4)

Correct Answer : (1)


Explanation : Frequency of reflected sound heard by the driver.

453. Two springs have force constants and . There are extended through the same

distance . If their elastic energies are and , then is equal to

(1)
(2)
(3)
(4)

Correct Answer : (1)


Explanation :

454. During an adiabatic expansion of 2 moles of a gas, the change in internal energy was
found . The work done during the process is

(1) Zero
(2)
(3)
(4)

Correct Answer : (4)


Explanation : For adiabatic forces

455. A person used force , shown in figure to move a load with constant velocity on given
surface

Identify the correct surface profile

(1)

(2)

(3)
(4)

Correct Answer : (1)


Explanation : Slope of surface should change from one constant value (non zero) to another
constant value (non zero) in terms of sign because force is constant piecewise

456. If a system undergoes contraction of volume then the work done by the system will be

(1) Zero
(2) Negligible
(3) Negative
(4) Positive

Correct Answer : (3)


Explanation : , here is negative so will be negative

457. Faintest stars are called

(1) Dwarfs
(2) Sixth magnitude stars

(3) Second magnitude


(4) Zero magnitude stars

Correct Answer : (2)

458. The velocity of electromagnetic waves in a dielectric medium of constant 4 is

(1)
(2)
(3)
(4)

Correct Answer : (1)

Explanation :

459. A vessel containing water is given a constant acceleration towards the right, along a
straight horizontal path. Which of the following diagram represents the surface of the liquid

(1) A
(2) B
(3) C
(4) D

Correct Answer : (3)


Explanation : Due to acceleration in forward direction, vessel is in an accelerated frame
therefore a Pseudo force will be exerted in backward direction. Therefore water will be
displaced in backward direction

460. A metallic block carrying current is subjected to a uniform magnetic induction as


shown in the figure. The moving charges experience a force given by …… which results in the
lowering of the potential of the face ….. Assume the speed of the carriers to be

(1)
(2)
(3)
(4)

Correct Answer : (1)


Explanation : As the block is of metal, the charge carriers are electrons, so for current along
positive -axis, the electrons are moving along negative -axis, and as the magnetic
field is along the -axis, . So for this case yield
[As ]

As force on electrons is towards the face , the electrons will accumulate on it an hence it
will acquire lower potential

461. An electric dipole has a pair of equal and opposite point charges q and –q separated by a
distance . The axis of the dipole is defined as

(1) Direction from positive charge to negative charge

(2) Direction from negative charge to positive charge

(3) Perpendicular to the line joining the two charges drawn at the centre and pointing upward
direction

(4) Perpendicular to the line joining the two charges drawn at the centre and pointing
downward direction

Correct Answer : (2)


Explanation : Axis of an electric dipole is always directed from negative charge to the positive
charge.

462. The point on the pressure-temperature phase diagram where all the three phases co-exist
is called
(1) Sublimation point
(2) Fusion point

(3) Triple point


(4) Vaporization point

Correct Answer : (3)


Explanation : In the diagram of water, if the three curves and are extended,
they came to meet at a point P, called the triple point.
Therefore, triple point of a substance is a point in the phase diagram representing a particular
set of pressure and temperature at which the solid, liquid and vopour phases of the substance
can coexist.
For water, the values of pressure and temperature corresponding to triple point P are 0.46 cm
of mercury and 273.16 K (or 0.01 ) respectively.

463. A particle is moving in a circle with uniform speed. Its motion is

(1) Periodic and simple harmonic


(2) Periodic but no simple harmonic

(3) A periodic
(4) None of the above

Correct Answer : (2)

464. The frequency 1057 MHz of radiation arising from two close energy levels in hydrogen
belongs to
(1) Radio waves
(2) Infrared waves
(3) Micro waves
(4)

Correct Answer : (2)

465. A person travels along a straight road for the first half time with a velocity and the next
half time with a velocity
The mean velocity of the man is

(1)

(2)
(3)

(4)

Correct Answer : (2)

466. An ideal transformer has 100 turns in the primary and 250 turns in the secondary. The
peak value of the ac is . The secondary voltage is nearest to

(1)
(2)
(3)
(4)

Correct Answer : (1)

Explanation :
467. A charged particle with charge enters a region of constant, uniform and mutually
orthogonal fields E and B with a velocity v perpendicular to both E and B, and comes out
without any change in magnitude or direction of v. Then

(1)
(2)
(3)
(4)

Correct Answer : (1)


Explanation : As v of charged particle is remaining constant, it means force acting on charged
particle is zero.
So,

468. Two immiscible liquids of refractive indices 1.5 and are filled in glass jar each of length 6
cm. A light of source is at the bottom of the jar, the apparent depth of light source will be

(1) 12.5 cm
(2) 17 cm
(3) 12 cm
(4) 8.5 cm

Correct Answer : (4)


Explanation : Apparent depth is given by
469. A stone is just released from the window of a train moving along a horizontal straight
track. The stone will hit the ground following

(1) Straight path


(2) Circular path
(3) Parabolic path
(4) Hyperbolic path

Correct Answer : (3)


Explanation : Due to constant velocity along horizontal and vertical downward force of gravity
stone will hit the ground following parabolic path

470. For one complete cycle of a thermodynamic process on a gas as shown in the -
diagram. Which of following is correct

(1)
(2)
(3)
(4)

Correct Answer : (1)


Explanation : , for a complete cycle and for given cycle work done is negative, so from
first law of thermodynamics will be negative,
471. If and are the potential energy and kinetic energy of the electron in stationary orbit in

the hydrogen atom, the value of is

(1) 2
(2)
(3) 1
(4)

Correct Answer : (4)


Explanation : As is known,

or

472. The characteristic impedance parallel line wire is

(1)

(2)

(3)

(4)

Correct Answer : (1)

Explanation : Choice (is correct as

473. In a hydrogen atom, the electron moves around the nucleus in a circular orbit of
radius m. Its time period is .The current associated with the electron
motion is (charge of electron is )
(1) 1.00 A
(2)
(3)
(4)

Correct Answer : (4)

474. Gravitational potential on the surface of earth is ( mass of the earth, radius of
earth)

(1)
(2)
(3)
(4)

Correct Answer : (2)


Explanation : Gravitational potential at a point on the surface of earth

475. In the certain process, 400 cal of heat are supplied to a system and at the same time 105 J
of mechanical work was done on the system. The increase in its internal energy is

(1) 20 cal
(2) 303 cal
(3) 404 cal
(4) 425 cal

Correct Answer : (4)


Explanation :

cal
Note is negative because work is done on the system
476. The coils of a step down transformer have 500 and 5000 turns. In the primary coil an ac
of at is sent. The value of the current and potential difference in the
secondary coil will be

(1)
(2)
(3)
(4)

Correct Answer : (3)

Explanation : . The transformer is step-down type, so primary coil will have more
turns. Hence

477. A parallel plate capacitor having a plate separation of is charged by connecting it to


a supply. The energy density is

(1)
(2)
(3)
(4)

Correct Answer : (2)


Explanation : The energy density of parallel plate capacitor is given by
478. Ozone layer blocks the radiations of wavelength

(1) Less than m


(2) Equal to m

(3) More than m


(4) All of the above

Correct Answer : (2)


Explanation : Ozone layer blocks the high energy radiations like UV

479. A small bar magnet oscillates in a horizontal plane with a period at a place where the
angle of dip is . When the same needle is made to oscillate in a vertical plane coinciding with
the magnetic meridian, its period will be

(1)
(2)
(3)
(4)

Correct Answer : (1)

Explanation :

480. Two capacitors each of capacitance are connected in parallel and are then charged
by supply. The total energy of their charges (in joules) is

(1) 0.01
(2) 0.02
(3) 0.04
(4) 0.06
Correct Answer : (3)

Explanation :

481. The radius of a nucleus with atomic mass number 7 is 2fermi. Find the radius of nucleus
with atomic number 189.

(1) 3
(2) 4
(3) 5
(4) 6

Correct Answer : (4)


Explanation :

482.

(1) The potential energy of the particle is zero

(2)

(3)

(4)
Correct Answer : (1)
Explanation : For horizontal planes potential energy remains constant equal to zero, if we
assumes surface to be the zero level.

483. The minimum wavelength of -rays produced in a coolidge tube operated at potential
difference of is

(1)
(2)
(3)
(4)

Correct Answer : (1)

Explanation :

484. An oil drop with charge is held stationary between two plates with an external potential
difference of 400 V. If the size of the drop is doubled without any change of charge, the
potential difference required to keep the drop stationary will be

(1) 400 V
(2) 1600 V
(3) 3200 V
(4) 4000 V

Correct Answer : (3)

Explanation : or or

485. Two soap bubbles of radii x and y coalesce to constitute a bubble of radius z. Then is equal
to
(1)
(2)
(3)

(4)

Correct Answer : (1)


Explanation :

If the process takes place is vacuum then

If process is isothermal

486. A convex lens of focal length forms a real, inverted image twice in size of the object.
The distance of the object form the lens is

(1) 0.5 m
(2) 0.166 m
(3) 0.33 m
(4) 1 m

Correct Answer : (1)


Explanation :

Or

Or

487. A simple pendulum has time period The bob is given negative charge and surface below
it is given positive charge. The new time period will be

(1) Less than


(2) Greater than
(3) Equal to
(4) Infinite

Correct Answer : (1)


Explanation : In this case time period of pendulum becomes

488. A gang capacitor is formed by interlocking a number of plates as shown in figure. The
distance between the consecutive plates is 0.885 cm and the overlapping area of the plates
is The capacity of the unit is
(1) 1.06 pF
(2) 4 pF
(3) 6.36 pF
(4) 12.72 pF

Correct Answer : (2)


Explanation : The given arrangement of nine plates is equivalent to the parallel combination of
8 capacitors.
The capacity of each capacitor,

Hence, the capacity of 8 capacitors

489. The disc of a siren containing 60 holes rotates at a constant speed of 360 rpm. The emitted
sound is in unison with a tuning fork of frequency

(1) 10 Hz
(2) 360 Hz
(3) 216 Hz
(4) 60 Hz

Correct Answer : (2)


Explanation : Speed =360 revolutions per min
=360/60 revolutions per sec=6
∴ frequency =6×60=360
490. Which of the following graphs show variation of potential energy (U) with position x.

(1)

(2)

(3)

(4)

Correct Answer : (3)


Explanation : The variation of potential energy(U)
With distance(x)is

Hence, parabolic graph is obtained.


491. A mass m is attached to the end of a rod of length l. The mass goes around a vertical
circular path with the other end hinged at the centre. What should be the minimum velocity of
mass at the bottom of the circle, so that the mass complete the circle?

(1)
(2)
(3)
(4)

Correct Answer : (3)


Explanation : When a particle is moved in a circle under the action of a torque then such
motion is non-uniform circular motion.
Applying principle of conservation of energy, total mechanical energy at L
=total mechanical energy at H

But

Or
Or
Hence for looping the vertical loop, the minimum velocity at the lowest point L IS
492. plots for two gases during adiabatic processes are shown in figure. Plots 1 and 2
should correspond respectively to

(1) He and
(2) and He
(3) He and
(4) and

Correct Answer : (2)


Explanation : As is clear from figure,
Slope of curve 2 > Slope of curve 1

As
adiabatic curve 2 corresponds to helium and adiabatic curve 1 corresponds to oxygen

493. The elastic energy stored per unit volume in a stretched wire is

(1)

(2)

(3)

(4)

Correct Answer : (1)


494. The deviation caused in red, yellow and violet colours for crown glass prism
are and respectively. The dispersive power of prism material is

(1) 0.268
(2) 0.368
(3) 0.468
(4) 0.568

Correct Answer : (1)

Explanation :

495. The co-ordinates of a moving particle at time are given by and The
instantaneous speed of the particle is

(1)
(2)
(3)
(4)

Correct Answer : (4)

Explanation : and

496. Figure given shows the distance –time graph of the motion of a car. It follows from the
graph that the car is
(1) At rest
(2) In uniform motion

(3) In non-uniform acceleration


(4) Uniformly accelerated

Correct Answer : (4)

Explanation : Since which is in form


Thus the motion is uniformly accelerated

497. A tuning fork of frequency 200 Hz is in unison with a sonometer wire. The number of beats
heard per second when the tension is increased by 1% is

(1) 1
(2) 2
(3) 4
(4) 1/2

Correct Answer : (1)

Explanation :

Number of beats

498. A coil of inductance 300 mH and resistance is connected to a source of voltage 2V. The
current reaches half of its steady state value in

(1) 0.05 s
(2) 0.1 s
(3) 0.15 s
(4) 0.3 s
Correct Answer : (2)
Explanation : The current at any instant is given by

499. A tangent galvanometer has a coil of 25 turns and a radius of 15 cm. The horizontal
component of the earth’s magnetic field is T. The current required to produce a
deflection of 45 in it is

(1) 0.29 A
(2) 0.14 A
(3) 1.2 A
(4) A

Correct Answer : (1)


Explanation :

From

A
500. The centre of mass of three particles of masses 1 kg, 2kg and 3kg is at (2, 2, 2). The position
of the fourth mass of 4 kg to be places in the system as that the new centre of mass is at (0, 0,
0) is

(1) (-3,-3,-3)
(2) (-3,3,-3)
(3) (2,3,-3)
(4) (2,-2,3)

Correct Answer : (1)


Explanation :
Position of centre of mass (2, 2, 2,)
kg
New position of centre of mass (0, 0, 0).
For initial position

Similarly,
and
For new position,

Similarly,

Position of fourth mass (-3, -3, -3)

501. The resistance of copper wire of diameter=2 mm is of the order of


(1)
(2)
(3)
(4)

Correct Answer : (1)

Explanation : From , when m,


mm,

502. Three bricks each of length and mass are arranged as shown from the wall. The
distance of the centre of mass of the system from the wall is

(1)
(2)
(3)
(4)

Correct Answer : (4)

Explanation :

From figure,
503. The modulation in which pulse duration varies in accordance with the modulating signal is
called

(1) PAM
(2) PPM
(3) PWM
(4) PCM

Correct Answer : (3)

504. In above question, the work done in the two wires is

(1) 0.5 J, 0.03 J


(2) 0.25 J, 0 J
(3) 0.03 J, 0.25 J
(4) 0 J, 0 J

Correct Answer : (1)

Explanation :
are constants.

…….(i)

Now ,

Again , ………..(ii)
Answer is confirmed by comparing Eqs. (i) and (ii) .

505. The dependence of binding energy per nucleon, on the mass number, , is represented
by

(1)

(2)

(3)

(4)

Correct Answer : (1)


Explanation : B.E. per nucleon is maximum for . For futher detail refer theory

506. Electromagnets are made of soft iron because soft iron has

(1) Low susceptibility and low retentivity


(2) Low susceptibility and high retentivity

(3) High permeability and low retentivity


(4) High permeability and high coercivity

Correct Answer : (3)


Explanation : Electromagnets are made of soft iron because soft iron has high permeability
and low retentivity

507. Figure shows four diagrams. Which of these curves represent isothermal and
adiabatic process?

(1) and
(2) and
(3) and
(4) and

Correct Answer : (1)


Explanation : In curves and , pressure and volume both increase. Therefore, temperature
must rise and heat must be supplied/work is done. Therefore, and cannot be required
curves. Out of and , slope of is smaller. Therefore, is isothermal curve and is
adiabatic curve

508. In India electricity is supplied for domestic use at 220 V. It is supplied at 110 V in USA. If the
resistance of a 60 W bulb for use in India is ,the resistance of a 60 W bulb for use in USA will
be

(1)
(2) 2
(3)
(4)
Correct Answer : (3)

Explanation : In India, power,

In USA, power,

As

or

509. The mutual inductance of an induction coil is . In the primary coil, the current reduces
from to zero in . What is the induced emf in the secondary coil

(1)
(2)
(3)
(4) Zero

Correct Answer : (2)

Explanation :

510. When a ceiling fan is switched on, it makes 10 rotations in the first 4 s. How many rotations
will it make in the next 4 s? (Assuming uniform angular acceleration)

(1) 10
(2) 20
(3) 40
(4) 30

Correct Answer : (4)

Explanation :

or
Let it make rotations in the first 8 s

Then,

or
The required number of rotations

511. A uniform rod of length and mass is free to rotate about point . The rod is
released from rest in horizontal position. Given that the moment of inertia of the rod

about is the initial angular acceleration of the rod will be

(1)

(2)

(3)

(4)

Correct Answer : (4)


Explanation : Weight of the rod will produce the torque

Angular acceleration
512. If the capacity of a condenser is 1 F, then its resistance in a DC circuit will be

(1) Zero
(2) infinity
(3)

(4)

Correct Answer : (2)


Explanation : Capacitive reactance( is given by

Where is angular frequency and C the capacitance.


Also, where is frequency.
In a DC circuit

513. An optical fiber made of glass with a core of refractive index of 1.55 and include with
another glass with a refractive index of 1.51. Launching takes place from air. What is the value
of critical angle for core-clad boundary?

(1)
(2)
(3)
(4)

Correct Answer : (3)

Explanation :

514. A particle moves in circle of radius 25 cm at the rate of two revolutions per second. The
acceleration of particle is
(1)
(2)
(3)
(4)

Correct Answer : (2)


Explanation : Acceleration of the particle is

515. In hydrogen atom, the electron is moving round the nucleus with
velocity in an orbit of radius The acceleration of the electron is

(1)
(2)
(3)
(4)

Correct Answer : (2)


Explanation : Given,
Acceleration of electron moving round the nucleus

516. A particle is projected with a velocity at an angle of . At the highest point, it


explodes into three particles of equal masses. One goes vertically upwards with a
velocity , the second particle goes vertically downwards. What is the velocity of third
particle?

(1) making angle with horizontal


(2) making angle with horizontal

(3)
(4)

Correct Answer : (3)


Explanation : If a particle is projected with velocity at an angle with the horizontal, the
velocity of the particle at the highest point is

If is the mass of the particle, then its initial momentum at highest point in the horizontal
direction . It means at the highest point, initially the particle has no momentum
vertically upwards or downwards. Therefore, after explosion, the final momentum of the
particles going upwards and downwards must be zero. Hence, the final momentum after
explosion is the momentum of the third particle, in the horizontal direction. If the third particle

moves with velocity , then its momentum ,


According to law of conservation of linear momentum,

We have or

517. When the forward bias voltage of a diode is changed from 0.6 V to 0.7 V, the current
changes from 5 mA to 15 mA. Then its forward bias resistance is

(1) 0.01
(2) 0.1
(3) 10
(4) 100

Correct Answer : (3)

Explanation : Forward biased resistance =

518. A current of enters one corner of an equilateral triangle having 3 wires of


resistances each and leaves by the corner Then the current and are
(1)
(2)
(3)
(4)

Correct Answer : (1)

Explanation :

519. In which of the following decay, the element does not change

(1) -decay
(2) -decay
(3) -decay
(4) None of these

Correct Answer : (3)


Explanation : As the particle has no charge and mass

520. The ratio transmitter operates on a wavelength of 1500 m at a power of 400 kW. The
energy of radio photon (in joule) is

(1) J
(2) J
(3) J
(4) J

Correct Answer : (2)


Explanation : The energy of photon is

Hence, energy of radio photon is


J

521. A body is moving in a straight line a shown in velocity-time graph. The displacement and
distance travelled by in 8s are respectively

(1) 12 m, 20 m
(2) 20m, 12 m
(3) 12 m, 12 m
(4) 20 m, 20 m

Correct Answer : (1)


Explanation : Displacement 12m
=20m

522. In a photocell bichromatic light of wavelength and are incident on cathode


whose work function is If a uniform magnetic field of exists parallel to the
plate, the radius of the path described by the photoelectron will be (mass of
electron )
(1)
(2)
(3)
(4)

Correct Answer : (2)


Explanation : Energy of photons corresponding to light of wavelength is

and that corresponding to

As and Photoelectric emission is possible with only. Maximum kinetic


energy of emitted photoelectrons
Photo electrons experience magnetic force and move along a circular path of radius

523. If rotational velocity of a dynamo armature is doubled, then induced e.m.f. will become

(1) Half
(2) Two times
(3) Four times
(4) Unchanged

Correct Answer : (2)


Explanation :

524. A student is standing at a distance of 50 metres from the bus. As soon as the bus begins its
motion with an acceleration of , the student starts running towards the bus with a
uniform velocity u. Assuming the motion to be along a straight road, the minimum value of ,
so that the student is able to catch the bus is
(1)
(2)
(3)
(4)

Correct Answer : (3)


Explanation : Let student catch the bus after sec. So it will cover distance

Similarly distance travelled by the bus will be for the given condition

To find the minimum value of

, so we get , then

525. Two wires, one made of copper and other of steel are joined end to end (as shown in
figure). The area of cross-section of copper wire is twice that of steel wire.

They are placed under compressive force of magnitudes F. The ratio for their lengths such that
change in lengths of both wires are same is (

(1) 2.1
(2) 1.1
(3) 1.2
(4) 2

Correct Answer : (2)

Explanation :

And
Here,

526. The radius of the first (lowest) orbit of the hydrogen atom is . The radius of the second
(next higher) orbit will be

(1)
(2)
(3)
(4)

Correct Answer : (1)


Explanation :

527. Consider elastic collision of a particle of mass moving with a velocity with another
particle of the same mass at rest. After the collision the projectile and the struck particle move
in directions making angles and respectively with the initial direction of motion. The sum
of the angles , is

(1)
(2)
(3)
(4)

Correct Answer : (2)


Explanation : If the masses are equal and target is at rest and after collision both masses
moves in different direction. Then angle between direction of velocity will be , if collision is
elastic

528. Quality of a musical note depends on

(1) Harmonics present


(2) Amplitude of the wave

(3) Fundamental frequency


(4) Velocity of sound in the medium

Correct Answer : (1)


Explanation : The quality of sound depends upon the number of harmonics present. Due to
different number of harmonics present in two sounds, the shape of the resultant wave is also
different

529. Which of the four arrangements in the figure correctly shows the vector addition of two

forces and to yield the third force

(1)

(2)

(3)
(4)

Correct Answer : (3)

530. Two charged particles are projected into a region in which a magnetic field is
perpendicular to their velocities. After they enter the magnetic field, you can conclude that

(1) The charges are deflected in opposite directions

(2) The charges continue to move in a straight line

(3) The charges move in circular paths

(4) The charges move in circular paths but in opposite directions

Correct Answer : (3)


Explanation : When a charged particle is projected into a region of magnetic field such that
magnetic field is perpendicular to the velocity vector of charged particles then charged particles
will follow circular path.

531. The equation represents a wave with

(1) Amplitude frequency and wavelength

(2) Amplitude frequency and wavelength

(3) Amplitude frequency and wavelength

(4) Amplitude frequency and wavelength

Correct Answer : (1)


Explanation : The given equation can be written as

Hence amplitude and frequency

And wave length

532. An aluminium rod, Young’s modulus , has a breaking strain of 0.2%. The
minimum cross-sectional area of the rod in in order to support a load of

(1)
(2)
(3)
(4)

Correct Answer : (4)

Explanation :

Or
= 0.71

533. Assuming to be frequency of first line in Balmer series, the frequency of the immediate
next( second) line is

(1) 0.50
(2) 1.35
(3) 2.05
(4) 2.70

Correct Answer : (2)


Explanation : For Balmer series, =2 and =3,4,5,….
Frequency, of 1st spectral line of Balmer series
or …(i)
Frequency, of 2nd spectral line of Balmer series

or ….(ii)
Form eqs. (i) and (ii), we have

534. The energy required to charge a parallel plate condenser of plate separation and plate
area of cross-section such that uniform electric field between the plates is ,is

(1)

(2)
(3)

(4)

Correct Answer : (3)


Explanation : Energy given by the cell

Here, = capacitance of capacitor =


potential difference across the plates =

Therefore,

535. The length of an elastic string is a metre when the tension is 44 N, and b metre when the
tension is 5 N. The length in metre when the tension is 9 N, is
(1)
(2)
(3)
(4)

Correct Answer : (2)


Explanation :

So,

…….(i)

…….(ii)
So, length of wire when tension is 9 N
change in length)

9=
Hence, final length =
=

536. Which of the following transition in Balmer series for hydrogen will have longest
wavelength?

(1) =2 to =1
(2) =6 to =1
(3) =3 to =2
(4) =6 to =2

Correct Answer : (3)


Explanation : According to the Bohr’s theory the wavelength of radiations emitted from
hydrogen atom given by
For maximum wavelength if then

537. To determine the half- life of radioactive element, a student plots graph of ln

t. Here is the rate of radioactive decay at time t. If the number of radioactive nuclei of this
element decreases by a factor of after 4.16 yr, the value of is

(1) 8
(2) 7
(3) 4
(4) 8.5

Correct Answer : (1)

Explanation : = |Activity of radioactive substance|

Taking log both sides

Hence, ln t graph is a straight line with slope- .


From the graph we can see that,

Now applying the equation


538. When power is drawn from the secondary coil of the transformer, the dynamic resistance

(1) Increases
(2) Decreases
(3) Remains unchanged
(4) Changes erratically

Correct Answer : (1)


Explanation : When the secondary coil circuit is open, the magnetic flux in the core is produced
by the primary current only. When the secondary circuit is closed, the currents in the secondary
coil also produce magnetic flux in the core but in opposite direction. This decreases the core
flux and hence reduces the back more current is drawn in the primary coil. Hence, power
factor is no longer zero. The power factor has increased or the phase difference is no
longer phase difference has decreased. Thus, dynamic resistance has increased

539. If represents the radius of the orbit of a satellite of mass moving around a planet of
mass , the velocity of the satellite is given by

(1)

(2)

(3)

(4)

Correct Answer : (4)

540. The figure shows variation of photocurrent with anode potential for a photo-sensitive
surface for three different radiations. Let and be the intensities and and be the
frequencies for the curves and respectively. Then
(1)
(2)
(3)
(4)

Correct Answer : (1)


Explanation : Saturation current is proportional to intensity while stopping potential increases
with increase in frequency. Hence,
while

541. A bullet fired at an angle of with the horizontal hits the ground 3 km away. By adjusting
its angle of projection, one can hope to hit a target 5 km away. Assume the muzzle speed to be
same and the air resistance is negligible

(1) possible to hit a target 5 km away


(2) not possible to hit a target 5 km away

(3) prediction is not possible


(4) None of the above

Correct Answer : (2)


Explanation : The body covers a horizontal distance AB during its flight. This horizontal range is
given by
Hence, maximum range with velocity of projection cannot be more than 3.464 m, Hence, it is
not possible to hit a target 5 km away.

542. At what temperature is the kinetic energy of a gas molecule double that of its value of

(1)
(2)
(3)
(4)

Correct Answer : (3)

Explanation :

543. A particle, with restoring force proportional to displacement and resisting force
proportional to velocity is subjected to a force.

If the amplitude of the particle is maximum for and the energy of the particle is
maximum for then

(1)
(2)

(3)
(4)

Correct Answer : (3)


Explanation : Amplitude resonance takes place at a frequency of external force which is less
than the frequency of undamped maximum vibrations. Velocity resonance takes place ( ,
maximum energy) when frequency of external periodic force is equal to natural frequency of
undamped vibrations

544. A particle moves in the plane under the influence of a force such that its linear
momentum is

Where and are constants. The angle between the force and momentum is

(1)
(2)
(3)
(4)

Correct Answer : (4)


Explanation :

The momentum and force are perpendicular to each other at


545. An asteroid of mass is approaching earth, initially at a distance of with speed . It
hits the earth with a speed ( are radius and mass of earth), then

(1)

(2)

(3)

(4)

Correct Answer : (3)


Explanation : Applying law of conservation of energy for asteroid at a distance and at
earth’s surface.

Substituting these values in Eq. (i), we get


546. A proton of energy is moving in a circular path in a uniform magnetic field. The energy
of an alpha particle moving in the same magnetic field and along the same path will be

(1) 4
(2) 2
(3) 8
(4) 6

Correct Answer : (3)

Explanation :

547. The frequency of the fundamental note in a wire stretched under tension T is v. if the
tension is increased to 25T, then the frequency of the fundamental note will be

(1) 25v
(2) 5v
(3) 10v
(4) V

Correct Answer : (2)


Explanation : The fundamental frequency
Or

548. One yard in SI units is equal

(1)
(2)
(3)
(4)

Correct Answer : (2)


Explanation :

549. At a certain instant of time the mass of rocket going up vertically is 100 kg. If it is ejecting 5
kg of gas per second at a speed of 400 m/s, the acceleration of the rocket would be
(Taking

(1)
(2)
(3)
(4)

Correct Answer : (2)


Explanation : The acceleration of a rocket is given by

550. Thermoelectric constant of a thermocouple are and Thermoelectric power at


inversion temperature is
(1)
(2)

(3)

(4)

Correct Answer : (2)

Explanation :
For inversion temperature

Thermo electric power

551. Two point charge and are situated at the origin and at the
point respectively. The point along the -axis where the electric field vanishes is

(1)
(2)

(3)

(4)

Correct Answer : (3)


Explanation :
Suppose the field vanishes at distance we have

or or

or or

552. With a standard rectangular bar magnet of length (l), breadth and magnetic
moment M, the time period of the magnet in vibration magnetometer is 4 s. If the magnet is
cut normal to its length into four equal pieces, the time period (in second) with one of the
pieces

(1) 16
(2) 2
(3) 1

(4)

Correct Answer : (3)


Explanation : Time period of magnet in vibration magnetometer

Where moment of inertia of magnet,


magnetic moment,
horizontal component of earth’s magnetic field.
553. A uniform spring of force constant k is cut into two pieces, the lengths of which are in the
ratio 1: 2. The ratio of the force constants of the shorter and longer piece is

(1) 1 : 2
(2) 2 : 1
(3) 1 : 3
(4) 2 : 3

Correct Answer : (2)


Explanation : Let be the force constant of the shorter part of the spring of length . In a
complete spring, three springs are in series each of force constant

554. and are long straight conductors, distance apart, carrying current . The
magnetic field at the midpoint of is
(1)

(2)

(3)

(4)

Correct Answer : (2)

Explanation : The field at the midpoint of due to is and the same is due

to . Therefore the total field is

555. A bubble of 8 mole of helium is submerged at a certain depth in water. The temperature of
water increases by . How much heat is added approximately to helium during expansion?

(1) 4000 J
(2) 3000 J
(3) 3500 J
(4) 4500 J

Correct Answer : (2)


Explanation : Heat added to helium during expansion

=
= ( =
3000 J

556. In which frequencies range space waves normally propagated

(1) HF
(2) VHF
(3) UHF
(4) SHF

Correct Answer : (3)


Explanation : Ultra high frequency waves are generally normally propagated

557. A body is thrown vertically up with a velocity . It passes three points and in its

upward journey with velocities and respectively. The ratio of the separations between

points and and between and is

(1) 1
(2) 2

(3)

(4)

Correct Answer : (4)

Explanation :
558. What will be the colour of sky as seen from the earth, if there were no atmosphere

(1) Black
(2) Blue
(3) Orange
(4) Red

Correct Answer : (1)


Explanation : Sky appears blue due to scattering. In absence of atmosphere no scattering will
occur

559. A long magnetic needle of length , magnetic moment and pole strength units is
broken into two pieces at the middle. The magnetic moment and pole strength of each piece
will be

(1)

(2)

(3)
(4)

Correct Answer : (3)


Explanation :
Pole strength of each part
Magnetic moment of each part

560. A body dropped from a height with an initial speed zero, strikes the ground with a
velocity . Another body of same mass is dropped from the same height with an initial
speed . Find the final velocity of second body with which it strikes the ground

(1)
(2)
(3)
(4)

Correct Answer : (3)


Explanation : For first case
For second case

561. Oxygen boils at . This temperature is approximately

(1)
(2)
(3)
(4)

Correct Answer : (2)

Explanation :
562. The potential energy between two molecules as a function of the distance between
them has been shown in the figure. The two molecules are

(1) Attracted when lies between and and are repelled when lies between and

(2) Attracted when lies between and and are repelled when lies between and

(3) Attracted when they reach

(4) Repelled when they reach

Correct Answer : (2)

Explanation :
In the region slope of the graph is positive
negative force is attractive in nature
In the region slope of the graph is negative
positive force is repulsive in nature

563. Choose the incorrect statement out of the following

(1) Every measurement by any measuring instrument has some error

(2) Every calculated physical quantity that is based on measured values has some error

(3) A measurement can have more accuracy but less precision and vice versa

(4) The percentage error is different from relative error

Correct Answer : (4)


564. In an intrinsic semiconductor, the Fermi level is

(1) Nearer to valency band than conduction band

(2) Equidistance from conduction band and valency band

(3) Nearer to conduction band than valency band

(4) Bisecting the conduction band

Correct Answer : (2)


Explanation : In intrinsic semiconductor of Fermi level is near the middle of the forbidden gap.

565. In the relation

(1) Number of molecules


(2) Atomic number
(3) Mass number
(4) Number of moles

Correct Answer : (4)

566. Maximum value of static friction is called

(1) Limiting friction


(2) Rolling friction
(3) Normal reaction
(4) Coefficient of friction

Correct Answer : (1)

567. A charged particle q is shot towards another charged particle Q which is fixed, with a
speed v. It approaches Q upto a closest distance and then returns. If was given a speed ,
the closest distance of approach would be
(1)
(2)
(3)
(4)

Correct Answer : (4)


Explanation : Let a particle of change having velocity v approaches Q upto a closest
distance and if the velocity becomes , the closest distance will be .’
The law of conservation of energy yields,
Kinetic energy of particle=electric potential energy between them at closest distance of
approach.

Or

Or …(i)

and …(ii)
Dividing Eq. (i) by Eq.(ii),

568. If the electric flux entering and leaving an enclosed surface respectively is and , then,
charge enclosed in closed surface is

(1)

(2)
(3)
(4)

Correct Answer : (4)


Explanation : Net electric flux of surface

569. Two wires of same material and radius have their lengths in ratio 1:2. If these wires are
stretched by the same force, the strain produced in the two wires will be in the ratio

(1) 2:1
(2) 1:1
(3) 1:2
(4) 1:4

Correct Answer : (3)

Explanation :

or
Therefore the strain produced in wire will be in ratio 1:2.

570. If the electron is a hydrogen atom jumps from an orbit with level to an orbit with
level the emitted radiation has a wavelength given by

(1)

(2)

(3)

(4)
Correct Answer : (1)
Explanation : Wavelength emitted is given by

571. Light travels in two media and with


speeds and respectively. Then the critical angle between them is

(1)

(2)

(3)

(4)

Correct Answer : (4)


Explanation : Here,

Light travels slower in denser medium. Hence medium is a denser medium and medium is
a rarer medium. Here, light travels from medium to medium .
Let be the critical angle between them

Refractive index of medium r.t medium is


572. A pendulum has time period If it is taken on to another planet having acceleration due
to gravity half and mass 9 times that of the earth then its time period on the other planet will be

(1)
(2)
(3)
(4)

Correct Answer : (4)

Explanation :

573. The frequency of tuning forks and are respectively 3% more and 2% less than the
frequency of tuning fork When and are simultaneously excited, 5 beats per second are
produced. Then the frequency of the tuning fork in (in ) is

(1) 98
(2) 100
(3) 103
(4) 105

Correct Answer : (3)


Explanation : Let be the frequency of fork then

and

But
574. A stone of mass tied to a light inextensible string of length is whirling in a
circular path of radius in a vertical plane. If the ratio of the maximum tension in the string to
the minimum tension in the string is 4 and if is taken to be the speed of the stone
at the highest point of the circle is

(1)
(2)
(3)
(4)

Correct Answer : (4)


Explanation : Since the maximum tension in the string moving in the vertical circle is at the
bottom and minimum tension is at the top

and

Or but

575. In a radiation spectrum obtained from a furnace of 2600 K has maximum intensity at
12000 wavelength. If the maximum intensity in spectrum of a star is at 5000 A. the
temperature of the outer surface of star is

(1) 7800 K
(2) 6240 K
(3) 5240 K
(4) 3640 K

Correct Answer : (2)


Explanation : Wavelength of radiation
Temperature of star
Wavelength of star spectrum
Temperature of star
From Wien law

576. Two stretched strings have length 𝒍 and 2𝒍 while tensions are T and 4T respectively. If they
are made of same material the ratio of their frequencies is

(1) 2:1
(2) 1:2
(3) 1:1
(4) 1:4

Correct Answer : (3)


Explanation : Frequency is given by

∴first frequency

And second frequency

∴hence, the ratio of frequencies


577. A microphone converts

(1) Sound signals into electrical signals


(2) Electrical signals into sound signals

(3) Both [a] and [b] above


(4) Neither [a] nor [b]

Correct Answer : (1)

578. A square loop of wire, side length 10 cm is placed at angle of 45 with a magnetic field that
changes uniformly from 0.1 T to zero in 0.7 s. The induced current in the loop (its resistance is
1 is

(1) 1.0 mA
(2) 2.5 mA
(3) 3.5 mA
(4) 4.0 mA

Correct Answer : (1)


Explanation : Initial magnetic flux linked with the loop

Final magnetic flux linked with the loop,

Now, induced emf in the loop ,

Induced current =

579. The mass defect per nucleon is called

(1) Binding energy


(2) Packing fraction
(3) Ionization energy
(4) Excitation energy

Correct Answer : (2)

580. An electromagnetic wave, going through vacuum is described by


Which of the following is independent of wavelength?

(1)
(2)
(3)
(4)

Correct Answer : (3)


Explanation : Here,
And

, where is the velocity of electromagnetic wave, which is


independent of wavelength of wave but depends upon the nature of medium of propagation of
wave

581. The ratio of minimum to maximum wavelength in Balmer series is

(1) 5:9
(2) 5:36
(3) 1:4
(4) 3:4

Correct Answer : (1)


Explanation : For maximum wavelength of Balmer series

…(i)
For minimum wavelength of Balmer series,

…(ii)
From Eqs.(i)and (ii), we have
582. A man standing between two parallel hills, claps his hand and hears successive echoes at
regular intervals of If velocity of sound is then the distance between the hills is

(1)
(2)
(3)
(4)

Correct Answer : (3)


Explanation : Let the man be at a distance from hill and from hill as shown in
figure. Let

The time interval between the original sound and echoes from and will be respectively

and
where is the velocity of sound
The distance between the hills is

583. The gas thermometers are more sensitive than liquid thermometers because

(1) Gases expand more than liquids


(2) Gases are easily obtained

(3) Gases are much lighter


(4) Gases do not easily change their states

Correct Answer : (1)


Explanation : For gases is more

584. A man standing between two parallel hills, claps his hand and hears successive echoes at
regular intervals of If velocity of sound is then the distance between the hills is

(1)
(2)
(3)
(4)

Correct Answer : (3)


Explanation : Let the man be at a distance from hill and from hill as shown in
figure. Let

The time interval between the original sound and echoes from and will be respectively

and
where is the velocity of sound
The distance between the hills is

585. A current of 1.5 A flows through a copper voltameter. The thickness of copper deposited
on the electrode surface of size is 20 min will be (density of
copper

(1) m
(2) 6.6 m
(3) 1.3 m
(4) 2.6 m
Correct Answer : (2)

Explanation : Thickness

m.

586. The temperature coefficient of resistance of a semiconductor

(1) Is always positive


(2) Is always negative

(3) Is zero
(4) May be positive or negative or zero

Correct Answer : (2)


Explanation : The temperature co-efficient of resistance of a semiconductor is always negative

587. A particle is projected with 200 , at an angle of 60 . At the highest point it explodes
into three particles of equal masses. One goes vertically upward with velocity 100 the
second particle goes vertically downward with the same velocity as the first. Then what is the
velocity of the third particle?

(1) 120 with 60 angle


(2) 200 with 30 angle

(3) 50 vertically upwards


(4) 300 horizontally

Correct Answer : (4)


Explanation : At the highest point momentum of particle before explosion

horizontally
Now as three is no external force during explosion, hence

However, since velocities of two fragments, of masses each, are 100 downward
and upward.
hence, or

m horizontally
horizontally

588. A biconvex lens form a real image of an object placed perpendicular to its principal axis.
Suppose the radii of curvature of the lens tend to infinity. Then the image would

(1) Disappear

(2) Remain as real image still

(3) Be virtual and of the same size as the object

(4) Suffer from aberrations

Correct Answer : (3)

Explanation :

For biconvex lens


Given so no focus at real distance

589. A polythene piece, rubbed with wool, is found to have negative charge of C. the
number of electrons transferred from wool to polythene is

(1)
(2)
(3)
(4)

Correct Answer : (2)


Explanation :

590. A body sitting on the topmost berth in the compartment of a train which is just going to
stop on a railway station, drops an apple aiming at the open hand of his brother sitting
vertically below his hands at a distance of about . The apple will fall

(1) Precisely on the hand of his brother

(2) Slightly away from the hand of his brother in the direction of motion of the train

(3) Slightly away from the hand of his brother in the direction opposite to the direction of
motion of the train

(4) None of the above

Correct Answer : (2)


Explanation : Horizontal velocity of apple will ren ain same but due to retardation of train,
velocity of train and hence velocity of boy ground decreases, so apple falls away from the
hand of boy in the direction of motion of the train

591. For light of wavelength 5000 photon energy is nearly 2.5 eV. For -rays of wavelength 1
, the photon energy will be close to

(1)
(2)
(3)
(4)

Correct Answer : (3)

Explanation : As energy ,
Therefore, energy corresponding to
592. Net magnetic field at the centre of the circle O due to a current through a loop as shown in
figure (

(1) zero
(2) Perpendicular to paper inwards

(3) Perpendicular to paper outwards


(4) Perpendicular to paper inwards if and perpendicular to paper outwards
if

Correct Answer : (2)


Explanation : The current through loop is anticlockwise. Hence, magnetic field at the points
within the loop is perpendicular to paper outwards. As the magnetic lines of force form a closed
path and tangent to line of force tells the direction of magnetic field at that point. Hence,
magnetic field at is perpendicular to paper inwards.

593. A marble block of mass lying on ice when given a velocity of is stopped by
friction in . Then the coefficient of friction is

(1)
(2)
(3)
(4)

Correct Answer : (4)

Explanation :
594. The temperature variation in the region of stratosphere lies from

(1) 290 K to 220 K


(2) 220 K to 280 K
(3) 220 K to 380 K
(4) 180 K to 700 K

Correct Answer : (2)

595. Four spheres of diameter and mass are placed with their centres on the four
corners of a square of side . Then the moment of inertia of the system about an axis along one
of the sides of the square is

(1)

(2)

(3)

(4)

Correct Answer : (2)


Explanation : We calculate moment of inertia of the system about

Moment of inertia of each of the sphere and about

Moment of inertia of each of the sphere and about

Using theorem of parallel axes


Total moment of inertia
596. In an AC series circuit, the instantaneous current is maximum when the instantaneous
voltage is maximum. The circuit element connected to the source will be

(1) Pure inductor


(2) Pure capacitor

(3) Pure resistor


(4) Combination of capacitor and an inductor

Correct Answer : (3)


Explanation : The circuit element connected to the AC source will be pure resistor. In pure
resistive AC circuit, voltage and current are in the same phase

597. represents

(1) Stress
(2) Young’s Modulus

(3) Pressure
(4) All of the above three quantities

Correct Answer : (4)

598. The potential energy of a certain spring when stretched through a distance ‘ ’ is .
The amount of work (in joule) that must be done on this spring to stretch it through an
additional distance ‘ ’ will be

(1) 30
(2) 40
(3) 10
(4) 20

Correct Answer : (1)

Explanation : [Given in the problem]

599. The potential energy of a particle in a force field is , where and are positive
constants and is the distance of particle from the centre of the field. For stable equilibrium,
the distance of the particle is

(1)
(2)
(3)
(4)

Correct Answer : (2)


Explanation : For equilibrium

For stable equilibrium

should be positive for the value of

Here is +ve value for

600. A motor having an armature of resistance is designed to operate at mains. At full


speed, it develops a back e.m.f. of . When the motor is running at full speed, the current in
the armature is

(1)
(2)
(3)
(4)

Correct Answer : (1)

Explanation :

601. If the mass of neutral kg, then the de-Broglie wavelength of neutral of
energy 3eV is

(1) m
(2) m
(3) m
(4) m

Correct Answer : (2)


Explanation : J

602. In a common emitter transistor amplifier and internal resistance of a


transistor is . The voltage amplification of amplifier will be

(1) 500
(2) 460
(3) 600
(4) 560

Correct Answer : (3)


Explanation :

603. A positive point charge is carried from a point to a point in the electric field of a point
charge + at . If the permittivity of free space is , the work done in the process is given by
(where a =OA and )

(1)

(2)

(3)

(4)

Correct Answer : (2)

Explanation :

604. When sunlight is scattered by atmospheric atoms and molecules, the amount of scattering
of light of wavelength 440 nm is . The amount of scattering for the light of wavelength 660 nm
is approximately

(1)
(2)
(3)

(4)
Correct Answer : (4)

Explanation : Amount of scattering of light


Now here
For let

then

605. Moment of inertia of a ring of mass and radius about an axis passing through the
centre and perpendicular to the plane is . What is the moment of inertia about its diameter

(1)

(2)

(3)
(4)

Correct Answer : (2)


Explanation : Moment of inertia of a ring of mass and radius about an axis passing
through the centre and perpendicular to the plane
…(i)
Moment of inertia of a ring about its diameter

[Using (i)]

606. The reciprocal of resistance is

(1) Conductance
(2) Resistivity
(3) Voltage
(4) None of the above

Correct Answer : (1)


Explanation : The reciprocal of resistance is called conductance

607. A ball of mass and another ball of mass are dropped from equal height. If time taken
by the balls are and respectively, then

(1)
(2)
(3)

(4)

Correct Answer : (2)


Explanation : The time of fall is independent of the mass

608. A wooden cube (density of wood ) of slide floats in a liquid of density with its upper
and lower surfaces horizontal. If the cube is pushed slightly down and released, its performs
simple harmonics motion of period , then is equal

(1)

(2)

(3)

(4)

Correct Answer : (2)


Explanation : Let at any instant, cube is at a depth from the equilibrium position then net
force acting on the cube = upthrust on the portion of length
…(i)
Negative sign shows that, force is opposite to . Hence equation of SHM
…(ii)
Comparing Eqs. (i) and (ii)

609. Radio wave diffract around building although light waves do not. The reason is that radio
waves

(1) Travel with speed target than


(2) Have much larger wavelength than light

(3) Carry news


(4) Are not electromagnetic waves

Correct Answer : (2)


Explanation : Diffraction takes places when the wavelength of wave is comparable with the size
of the obstacle in path. The wavelength of radio waves is greater than the wavelength of light
waves. Therefore, radio waves are diffracted around building

610. If there are no heat losses, the heat released by the condensation of of steam
at into water at can be used to convert of ice at into water at Then
the ratio is nearly
(1)
(2)
(3)
(4)

Correct Answer : (4)


Explanation : Heat released to convert of steam at to water at is
If of ice is converted from to water at it requires heat

or

611. Which of the following circuits is correct for verification of Ohm’s law?

(1)

(2)

(3)
(4) None of these

Correct Answer : (2)


Explanation : The circuit arrangement shown in figure (is the correct arrangement for
verification of Ohm’s law. For convenience the same figure has been redrawn here. In the
figure, is the resistance, for which Ohm’s law is to be verified. Voltmeter is connected to its
parallel and ammeter, cell and rheostat arrangement in the series.
612. One mole of a monoatomic ideal gas is mixed with one mole of a diatomic ideal gas. The
molar specific heat of the mixture at constant volume is

(1)
(2)
(3)
(4)

Correct Answer : (3)

Explanation :

613. Half-life of radioactive substance is 3.20 h. What is the time taken for a 75% of substance
to be used?

(1) 6.38 h
(2) 12 h
(3) 4.18 day
(4) 1.2 day

Correct Answer : (1)


Explanation : The activity or decay rate of radioactive substance is the number of decays per
second.

or

or
where is the activity of radioactive substance at time
According to question,
or

or

or t 6.38 h

614. A type kept outside in sunlight bursts off after sometime because of

(1) Increases in pressure


(2) Increases in volume
(3) Both [a] and [b]
(4) None of these

Correct Answer : (1)

615. Two rectangular blocks and of different metals have same length and same area of
cross-section. They are kept in such a way that their cross-sectional area touch each other. The
temperature at one end of is and that of at the other end is . If the ratio of their
thermal conductivity is 1 : 3, then under steady state, the temperature of the junction in contact
will be

(1)
(2)
(3)
(4)

Correct Answer : (1)

Explanation : It is given that then the temperature of the junction in


contact
616. Displacement between maximum potential energy position and maximum kinetic energy
position for a particle executing S.H.M. is

(1)
(2)
(3)
(4)

Correct Answer : (3)


Explanation : Maximum potential energy position
and maximum kinetic energy position is

617. Equivalent resistance between the points and is (in )

(1)

(2)

(3)

(4)
Correct Answer : (3)

Explanation :

618. 1 mol of gas occupies a volume of 200 mL at 100 mm pressure. What is the volume
occupied by two moles of gas at 400 mm pressure and at same temperature?

(1) 50 mL
(2) 100 mL
(3) 200 mL
(4) 400 mL

Correct Answer : (2)


Explanation : Given,
From Boyle’ Law

mL
Volume of 2 mol gas= mL

619. Two long parallel copper wires carry currents of each in opposite directions. If the wires
are separated by a distance of , then the force between the two wires is

(1) , attractive
(2) , repulsive

(3) , attractive
(4) , repulsive

Correct Answer : (2)

Explanation : [repulsive]

620. With the propagation of a longitudinal wave through a material medium, the quantities
transmitted in the propagation direction are

(1) Energy, momentum and mass


(2) Energy

(3) Energy and mass


(4) Energy and linear momentum

Correct Answer : (4)

621. The direction of induced e.m.f. during electromagnetic induction is given by

(1) Faraday’s law


(2) Lenz’s law
(3) Maxwell’s law
(4) Ampere’s law

Correct Answer : (2)

622. A 1000 Hz sound wave in air strikes the surface of a lake and penetrates into water. If
speed of sound in water is ,the frequency and wavelength of waves in water are

(1) 1500 Hz, 1m


(2) 1000 Hz, 1.5m
(3) 1000 Hz, 1m
(4) 1500 Hz, 1.5m

Correct Answer : (2)


Explanation : Frequency remains the same wavelength chances

623. A wheel is subjected to uniform angular acceleration about its axis. Initially its angular
velocity is zero. In the first , it rotates through an angle . In the next , it rotates
through an additional angle . The ratio of is

(1) 1
(2) 2
(3) 3
(4) 5

Correct Answer : (3)

Explanation : Using relation

…(i)
As
Now using same equation for

…(ii)

From (i) and (ii), and

624. When both the listener and source are moving towards each other, then which of the
following is true regarding frequency and wavelength of wave observed by the observer?

(1) More frequency, less wavelength

(2) More frequency, more wavelength

(3) Less frequency, less wavelength

(4) More frequency, constant wavelength

Correct Answer : (1)


Explanation : According to Doppler’s effect, whenever there is a relative motion between a
source of sound and listener, the apparent frequency of sound heard by the listener is different
from the actual frequency of sound emitted by the source. Let S be source of sound and L the
listener of sound. Let v be the actual frequency of sound emitted by the source and be the
actual wavelength of the sound emitted.
If v is velocity of sound in still air, then

If velocity of listener is and velocity of source is , then apparent frequency of sound waves
heard by the listener is

Here, both source and listener are approaching each other.

Then is positive and is negative.

Also,

So, listener listens more frequency and observes less wavelength.

625. What are the units of

(1)
(2)
(3)
(4) Unitless

Correct Answer : (2)


Explanation : Unit of Unit of

626. A body is projected with a velocity and after some time it returns to the point from
which it was projected. The average velocity and average speed of the body for the total time of
flight are

(1) and
(2) 0 and
(3) 0 and 0
(4) and 0

Correct Answer : (2)

Explanation :

Velocity of projection = [Given]

627. A medium shows relation between and as shown. If speed of light in the medium
is then value of is

(1) 1.5
(2) 2
(3)
(4)

Correct Answer : (4)

Explanation : From graph it is clear that

Also
628. The self inductance of a coil is , a current of change to
within through the coil. The value of induced e.m.f. will be

(1)
(2)
(3)
(4)

Correct Answer : (3)

Explanation :

629. A wire of density is stretched between two clamps 1m part and is subjected
to an extension of . The lowest frequency of transverse vibration in the wire
is

(1) 40 Hz
(2) 35 Hz
(3) 30 Hz
(4) 25 Hz

Correct Answer : (2)


Explanation : Given :
From the formula

Or

=
=44.1
630. The characteristic impedance of a co-axial cable is . If its inductance is 0.4 mH, its
capacitance would be

(1)
(2)
(3) 15.6 nF
(4) 15.6 pF

Correct Answer : (3)

Explanation :

631. The original temperature of a black body is . The temperature at which this black
body must be raised so as to double the total radiant energy, is

(1)
(2)
(3)
(4)

Correct Answer : (2)

Explanation :
632. The power factor of good choke coil is

(1) Nearly zero


(2) Exactly zero
(3) Nearly one
(4) Exactly one

Correct Answer : (1)

Explanation : In choke coil so

633. A simple pendulum is suspended from the ceiling of a lift. When the lift is at rest its time
period is . With what acceleration should the lift be accelerated upwards in order to reduce its
period to ? (g is acceleration due to gravity)

(1) 2 g
(2) 3 g
(3) 4 g
(4) g

Correct Answer : (2)


Explanation : Time period of simple pendulum is given by

…(i)
When the lift is moving up with an acceleration , then time period becomes

…(ii)
Dividing Eq.(ii) by Eq. (i), we get
634. In a radio receiver, the short wave and medium wave stations are tuned by using the same
capacitor but coils of different inductance and respectively then

(1)
(2)
(3)
(4) None of these

Correct Answer : (2)

Explanation : As

Also, and

635. The ratio of root mean square velocity of is

(1)
(2)
(3)
(4)

Correct Answer : (4)


Explanation : Root mean square velocity of gas molecules
636. If eye is kept at a depth inside water of refractive index and viewed outside, then the
diameter of the circle through which the outer objects become visible, will be

(1)

(2)

(3)

(4)

Correct Answer : (3)


Explanation : Let be the radius of circle through which other objects become visidble. The
rays of light must be incident at critical angle C

Diameter

637. Identify the wrong statement.


(1) The electrical potential energy of a system of two protons shall increase if the separation
between the two is decreased.

(2) The electrical potential energy of a proton-electron system will increase if the separation
between the two is decreased.

(3) The electrical potential energy of a proton-electron system will increase if the separation
between the two is increased.

(4) The electrical potential energy of system of two electrons shall increase if the separation
between the two is decreased.

Correct Answer : (3)


Explanation : Potential energy

Or
When decreases increases and . Moreover, potential energy as well as force is
positive, if there is repulsion between the particles and negative if there is attraction.

638. An infinitely long straight conductor is fixed and a current is passed through it. Another
movable straight wire of finite length and carrying current is held perpendicular to it and
released. Neglect weight of the wire

(1) The rod will move upwards parallel to itself

(2) The rod will move downward parallel to itself

(3) The rod will move upward and turn clockwise at the same time

(4) The rod will move upward and turn anti-clockwise at the same time

Correct Answer : (3)


Explanation : Since the force on the rod is non-uniform it will experience force and torque.
From the left hand side it can be seen that the force will be upward and torque is clockwise

639. Which of the following rays is emitted by a human body?

(1) X-rays
(2) UV rays
(3) Visible rays
(4) IR rays

Correct Answer : (4)


Explanation : Generally, temperature of human body is , corresponding to which IR and
microwave radiations are emitted from the human body

640. A force F is required to break a wire of length and radius r. What force is required to
break a wire, of the same material, having twice the length and six times the radius ?

(1)
(2)
(3) 9
(4) 36

Correct Answer : (4)


Explanation : Breaking force does not depend upon length.
Breaking force = breaking stress area of cross-section for a given material, breaking stress in
constant.

Or s
641. If a radio receiver amplifies all the signal frequencies equally well, it is said to have high

(1) Fidelity
(2) Distortion
(3) Sensibility
(4) Sensitivity

Correct Answer : (1)

642. A metal rod of length and mass is pivoted at one end. A thin disk of mass and
radius is attached at its centre to the free end of the rod. Consider two ways the disc is
attached case - the disc is not free to rotate about its centre and case – the disc is free to
rotate about its centre. The rod-disc system performs SHM in vertical plane after being released
from the same displaced position. Which of the following statement(s) is/are true?

(1) Restoring torque in case =Restoring torque in case


(2) Restoring torque in case

(3) Angular frequency for case


(4) Angular frequency for case

Correct Answer : (4)

Explanation :
= Restoring torque about point .
In case , moment of inertia will be more. Hence, angular acceleration will be less.
Therefore angular frequency will be less. Note Question is difficult because this type of SHM is
rarely.
643. Solar spectrum is an example for

(1) Line emission spectrum


(2) Continuous emission spectrum

(3) Band absorption spectrum


(4) Line absorption spectrum

Correct Answer : (4)


Explanation : Solar Spectrum is an example of line absorption Spectrum.

644. Which of the following statements is wrong?

(1) KE of a body is independent of the direction of motion

(2) In an elastic collision of two bodies ,the momentum and energy of each body is conserved

(3) If two protons are brought towards each other the PE of the system decreases.

(4) A body cannot have energy without momentum.

Correct Answer : (3)


Explanation : If momentum is Zero ie, if p=0,then kinetic energy

But potential energy cannot be zero, thus a body can have energy without momentum.

645. A cylindrical tube containing air is open at both ends. If the shortest length of the tube for
resonance with a given fork is 2 cm, the next shortest length for resonance with the same fork
will be
(1) 60 cm
(2) 40 cm
(3) 90 cm
(4) 80 cm

Correct Answer : (2)


Explanation : As the tube is open at both ends, therefore, next shortest length for
resonance .

646. A solenoid has core of a material with relative permeability 500 and its windings carry a
current of . The number of turns of the solenoid is 500 per metre. The magnetization of the
material is nearly

(1)
(2)
(3)
(4)

Correct Answer : (2)


Explanation : Here, turns/m,
Magnetic intensity,
As , where is the magnetic susceptibility of the material
or
Magnetisation,

647. There is a simple pendulum hanging from the ceiling of a lift. When the lift is stand still, the
time period of the pendulum is . If the resultant acceleration becomes , then the new time
period of the pendulum is

(1)
(2)
(3)
(4)

Correct Answer : (3)


Explanation : Time period of a simple pendulum of length , is given by

…(i)
Where, g is acceleration due to gravity.

When ,
New, time period is

…(ii)
Dividing Eq. (ii) by Eq. (i), we get

Hence, new time period becomes twice of the original value.

648. “Lux” is a unit of

(1) Luminous intensity of a source


(2) Illuminance on a surface

(3) Transmission coefficient of a surface


(4) Luminous efficiency of source of light

Correct Answer : (2)

649. The time constant of an circuit represents the time in which the current in the circuit

(1) Reaches a value equal to about 37% of its final value


(2) Reaches a value equal to about 63% of its final value

(3) Attains a constant value


(4) Attains 50% of the constant value
Correct Answer : (2)

650. Through two parallel wires , 10A and 2A of currents are passed respectively in
opposite directions. If the wire is infinitely long and the length of the wire is 2m, then force
on the conductor , which is situated at 10 cm distance from , will be

(1) N
(2) N
(3) N
(4) N

Correct Answer : (2)

Explanation :

651. When heat is given to a gas in an isothermal change, the result will be

(1) External work done


(2) Rise in temperature

(3) Increase in internal energy


(4) External work done and also rise in temp.

Correct Answer : (1)


Explanation : In isothermal change, temperature remains constant,
Hence
Also from

652. Two particles are executing simple harmonic motion of the same amplitude and
frequency along the -axis. Their mean position is separated by distance . If the
maximum separation between them is , the phase difference between their motions is
(1)

(2)

(3)

(4)

Correct Answer : (1)


Explanation :

The resultant motion cam be treated as a simple harmonic motion with amplitude 2

Given, maximum distance between the particles=


Amplitude of resultant SHM
=

653. A monoatomic gas is kept at room temperature Calculate the average kinetic energy
of gas molecule (Use units)

(1)
(2)
(3)
(4)
Correct Answer : (3)

Explanation : The average kinetic energy of monoatomic gas molecule is


Where is the Boltzmann constant and is the temperature of the gas in kelvin

654. The curves for potential energy and kinetic energy of a two particle system are
shown in figure. At what points the system will be bound

(1) Only at point


(2) Only at point
(3) At point and
(4) At points and

Correct Answer : (4)


Explanation : The system will be bound at points where total energy is negative. In the given
curve at point A, B and C the P.E. is more than K.E.

655. An particle of mass and charge is situated in a uniform


electric field of The velocity of the particle at the end of path when it
starts from rest is

(1)
(2)
(3)
(4)

Correct Answer : (4)


Explanation : Here, mass of particle
Charge of particle
Electric field
Force on particle in a uniform electric field is

Acceleration of the particle is

Given

[As ]

or

656. A long solenoid has 800 turns per metre length of solenoid. A current of 1.6 A flows
through it. The magnetic induction at the end of the solenoid on its axis is

(1) T
(2) T
(3) T
(4) T

Correct Answer : (2)

Explanation : T.

657. The displacement-time graph of a moving object is shown in figure. Which of the velocity-
time graphs shown in figure could represent the motion of the same body?
(1)

(2)

(3)

(4)

Correct Answer : (3)


Explanation : Think of the lope of the given displacement-time graph at different points and
you would arrive at the correct answer.
Alternatively, look at the self-illustrative figure.

658. Inside a cylinder closed at both ends is a movable piston. On one side of the piston is a
mass of a gas, and on the other side a mass 2 of the same gas. What fraction of the
volume of the cylinder will be occupied by the larger mass of the gas when the piston is in
equilibrium? The temperature is the same throughout.

(1)

(2)

(3)

(4)

Correct Answer : (1)


Explanation : When the piston is in equilibrium, the pressure is same on both the sides of the
piston. It is given that temperature and weight of gas on the two sides of piston not change.
From ideal gas equation, we have mass of the gas.

So,

Or

Or =

659. Moment of inertia of a disc about an axis which is tangent and parallel to its plane is .
Then the moment of inertia of disc about a tangent, but perpendicular to its plane will be

(1)

(2)

(3)

(4)

Correct Answer : (4)


Explanation : The moment of inertia of the disc about an axis parallel to its plane is

or
Now, moment of inertia about a tangent perpendicular to its plane is

660. The reading of the ideal voltmeter in the adjoining diagram will be

(1)
(2)
(3)
(4)

Correct Answer : (2)

Explanation : Current flowing in the circuit

P.D. across
P.D. across
661. Energy of a hydrogen atom with principal quantum number is given by =
.The energy of a photon ejected when the electron jumps from n =3 state to state of
hydrogen , is approximately

(1) 1.5
(2) 0.85
(3) 3.4
(4) 1.9

Correct Answer : (4)

Explanation : Given ,
Energy of photon ejected when electron jumps from =3 state to =2 state is given by

So, =
(approximately)

662. The displacement (in metre) of a particle in simple harmonic motion is related to time
(in second) as

The frequency of the motion will be

(1) 0.5 Hz
(2) 1.0 Hz

(3) Hz
(4) Hz

Correct Answer : (1)


Explanation : The standard equation in SHM is
…(i)
Where is amplitude, the angular velocity and ( ) the phase difference.

Also, is periodic time.


So, Eq. (i) becomes

…(ii)
Given, equation is

…(iii)
Comparing Eq. (ii) with Eq. (iii), we get

So, frequency Hz

663. When a mass is rotating in a plane about a fixed point, its angular momentum is directed
along

(1) A line perpendicular to the plane of rotation


(2) The line making an angle of to the plane of rotation

(3) The radius


(4) The tangent to the orbit

Correct Answer : (1)


Explanation : It’s always in axial direction

664. A public park, in the form of a square, has an area of . The side of park is

(1) m
(2) m
(3) m
(4) m

Correct Answer : (1)

Explanation : Percentage error inside

Absolute error inside

665. To break a wire of one metre length, minimum , is required. Then the wire of the
same material of double radius and length will require breaking weight

(1) -
(2) -
(3) -
(4) -

Correct Answer : (4)


Explanation : Breaking force = Breaking stress Area of cross section of wire
Breaking force (Breaking stress is constant)
If radius becomes doubled then breaking force will become 4 times

666. A solid conducting sphere having a charge us surrounded by an uncharged concentric


conducting hollow spherical shell. Let the potential difference between the surface of the solid
sphere and that of the outer surface of the hollow shell be V. if the shell is now given a
charge the new potential difference between the same two surface is

(1)
(2)
(3)
(4)

Correct Answer : (1)


Explanation : Due to additional charge of given to external spherical shell, the potential
difference between conducting sphere and the outer shell will not change because by presence
of charge on outer shell, potential everywhere inside and on the surface of the shell will change
by same amount. Therefore, the potential difference between sphere and shell remain
unchanged.

667. An object is viewed through a compound microscope and appears in focus when it is 5
mm away from the objective lens. When a sheet of transparent material 3 mm thick is placed
between the objective and the microscope, the objective lens has to be moved 1 mm to bring
the object back into the focus. The refractive index of the transparent material is

(1)
(2)
(3)
(4)

Correct Answer : (1)

Explanation : Shift

Or

668. Which of the following quantities remains constant in a step-down transformer ?

(1) Current
(2) Voltage
(3) Power
(4) None of these

Correct Answer : (3)


Explanation : Power remains constant in a ideal step down transformer.
669. A laser device produces amplification in the

(1) Microwave region


(2) Ultraviolet or visible region

(3) Infrared region


(4) None of the above

Correct Answer : (2)


Explanation : A laser device produces amplification in the ultraviolet or visible region.

670. For an ideal gas, in an isothermal process

(1) Heat content remains constant


(2) Heat content and temperature remain constant

(3) Temperature remains constant


(4) None of the above

Correct Answer : (3)


Explanation : In isothermal process temperature remains constant

671. A 5.5 m length of string has a mass of 0.035 kg. If the tension in the string is 77 N, the
speed of a wave on the string is

(1)
(2)
(3)
(4)

Correct Answer : (1)

Explanation :
672. Two short magnets with their axes horizontal and perpendicular to the magnetic meridian
are placed with their centres east and west of magnetic needle. If the needle
remains undeflected, the ratio of their magnetic moments is

(1)
(2)
(3)
(4)

Correct Answer : (3)

Explanation : For null deflection

673. To make the frequency double of a spring oscillator, we have to

(1) Reduce the mass to one fourth


(2) Quardruple the mass

(3) Double of mass


(4) Half of the mass

Correct Answer : (1)

Explanation :

674. For an ideal gas of diatomic molecules

(1)
(2)
(3)

(4)

Correct Answer : (4)

Explanation :

675. A light ray is incident upon a prism in minimum deviation position and suffers a deviation
of . If the shaded half of the prism is knocked off, the ray will

(1) Suffer a deviation of


(2) Suffer a deviation of

(3) Suffer a deviation of


(4) Not come out of the prism

Correct Answer : (3)

Explanation : By formula and in the second position

676. Consider the following statements concerning electrons :


I. Electrons are universal constituents of mater.
II. J J Thomson received the very first Nobel prize in Physics for discovering the electron.
III. The mass of the electron is about 1/2000 of a neutron.
IV. According to Bohr the linear momentum of the electron is quantised in the hydrogen atom.
Which of the above statements are not correct?

(1) I
(2) II
(3) III
(4) IV

Correct Answer : (4)

Explanation : The mass of electron is about times that of a neutron and angular
momentum of electron is quantised in the hydrogen atoms but not the linear momentum of
electron

677. A block of mass is placed on a smooth wedge of inclination . The whole system is
accelerated horizontally so that the block does not slip on the wedge. The force exerted by the
wedge on the block ( is acceleration due to gravity ) will be

(1)
(2)
(3)
(4)

Correct Answer : (4)

Explanation :
When the whole system is accelerated towards left then pseudo force works on a block
towards right
For the condition of equilibrium

Force exerted by the wedge on the block


678. Two wires of the same material and equal length are joined in parallel combination. If one
of them has half the thickness of the other and the thinner wire has a resistance of the
resistance of the combination is equal to

(1)

(2)

(3)

(4)

Correct Answer : (2)

Explanation : same, same,

By using

Hence,

679. The sound wave was produced in a gas is always

(1) Longitudinal
(2) Transverse
(3) Stationary
(4) Electromagnetic

Correct Answer : (1)


Explanation : Sound waves are longitudinal waves. They produce alternately the states of
compression and rarefaction at a point in the medium.
680. A packet is dropped from a balloon which is going upwards with the velocity , the
velocity of the packet after 2 seconds will be

(1)
(2)
(3)
(4)

Correct Answer : (3)


Explanation : When packet is released from the balloon, it acquires the velocity of balloon of
value . Hence velocity of packet after , will be

681. A police car horn emits a sound at a frequency 240 Hz the frequency 240 Hz when the car
is at rest. If the speed of sound is , the frequency heard by an observer who is
approaching the car at speed of 11 , is

(1) 248 Hz
(2) 244 Hz
(3) 240 Hz
(4) 230 Hz

Correct Answer : (1)


Explanation : From Doppler’s effect in sound, the perceived frequency (v’) is given by

Where is velocity of observer, v of sound and v the original frequency.


Give, v=240 Hz,
682. An object is placed at a distance of from a convex lens of power . Find the position
of the image

(1)
(2)
(3)
(4)

Correct Answer : (1)


Explanation : The focal length of the convex lens

Therefore, the image will be formed at a distance of on the behind the object

683. A Carnot engine used first ideal monoatomic gas and then an ideal diatomic gas, if the
source and sink temperatures are and , respectively and the engine extract 1000 J of
heat from the source in each cycle, then

(1) Area enclosed by the diagram is 10 J

(2) Heat energy rejected by engine is 1st case is 600 J while that in 2nd case in 113 J

(3) Area enclosed by the diagram is 500 J

(4) Efficiencies of the engine in both the cases are in ratio 21:25

Correct Answer : (3)


Explanation : Here,
684. Five equal resistances each of resistance are connected as shown in the figure. A battery
of volts is connected between and The current flowing in will be

(1)

(2)

(3)

(4)

Correct Answer : (2)


Explanation : The given circuit can be redrawn as follows

Equivalent resistance between and is and current

685. Consider elastic collision of a particle of mass m moving with a velocity u with another
particle of the same mass at rest. After the collision the projectile and the stuck particle move in
directions making angles and respectively with the initial direction of motion.
The sum of the angles
(1)
(2)
(3)
(4)

Correct Answer : (2)


Explanation : Let particle with mass , move with velocity u,and and be velocity after
collision. Since , elastic collision is one in which the momentum is conserved , we have

…..(i)
In perpendicular direction
…..(ii)
Also elastic collision occurs only if there is no conversion of kinetic energy into other from,
Hence

….(iii)
Squaring Esq.(i)and (ii)and adding we get

Using Eq.(iii),we get

Hence
Or
When two identical particles collide elastically and obliquely,
One being at rest, then they fly off in mutually perpendicular directions.

686. The coil of a galvanometer consists of 100 turns and effective area of . The
restoring couple is . The magnetic field between the pole pieces is . The
current sensitively of this galvanometer will be

(1)
(2)
(3)
(4)

Correct Answer : (4)

Explanation : Current sensitivity

687. A layer of ionosphere does not reflect waves with frequencies greater than 10 MHz; then
maximum electron density in this layer is

(1)
(2)
(3)
(4)

Correct Answer : (1)

688. A bomb is kept stationary at a point. It suddenly explodes into two fragments of
masses and . The total K.E. of the fragments is . What is the K.E. of the smaller
fragment

(1)
(2)
(3)
(4)

Correct Answer : (3)


Explanation :
As the momentum of both fragments are equal therefore

., …(i)
According to problem …(ii)
By solving equation (i) and (ii), we get
and

689. The displacement of a particle varies according to the relation The


amplitude of the particle is

(1) -4
(2) 4
(3) 4
(4) 8

Correct Answer : (3)


Explanation :

So, amplitude=

690. A very small magnet is placed in the magnetic meridian with its south pole pointing north.
The null point is obtained away from the centre of the magnet. If the earth’s magnetic
field (horizontal component) at this point is , the magnetic moment of the magnet is

(1)
(2)
(3)
(4)
Correct Answer : (2)

Explanation :
At neutral point

691. The graph between the time period and the length of a simple pendulum is

(1) Straight line


(2) Curve
(3) Ellipse
(4) Parabola

Correct Answer : (4)


Explanation : The time period of a simple pendulum is

Here, is the length of the pendulum.


On squaring both sides

So, the graph between time period and length of the pendulum is a parabola.
692. A wooden wheel of radius is made of two semicircular parts (see figure). The two parts
are held together by a ring made of a metal strip of cross sectional area and length . is
slightly less than To fit the ring on the wheel, it is heated so that its temperature rises
by and it just steps over the wheel. As it cools down to surrounding temperature, it presses
the semicircular parts together. If the coefficient of linear expansion of the metal is , and its
Young’s modulus is , the force that one part of the wheel applies on the other part is

(1)
(2)
(3)
(4)

Correct Answer : (4)


Explanation : If temperature increases by ,
Increase in length

Let tension developed in the ring is

(From figur
Where, is the force that one part of the wheel applies on the other part
693. A horizontal platform with an object placed on it is executing SHM in the vertical direction.
The amplitude of oscillation is 30.92 What must be the least period of these
oscillations, so that the object is not detached from the platform?

(1) 0.1256 s
(2) 0.1356 s
(3) 0.1456 s
(4) 0.1556 s

Correct Answer : (1)


Explanation : The object is not detached from the platform if

694. A wire of length is moving at a speed of perpendicular to its length and in a


homogenous magnetic field of . The ends of the wire are joined to a circuit of
resistance . The rate at which work is being done to keep the wire moving at constant speed
is

(1)

(2)

(3)
(4)

Correct Answer : (2)

Explanation : Rate of work also


695. The bodies of masses and are dropped gently from the top of a tower. At a
point from the ground, both the bodies will have the same

(1) Momentum
(2) Kinetic energy
(3) Velocity
(4) Total energy

Correct Answer : (3)


Explanation : Velocity of fall is independent of the mass of the falling body

696. In which direction, the magnetic field on the axis at a distance z from the centre of the bar
magnet would be?

(1) In the perpendicular direction of the magnetic moment (M) of the magnet

(2) In the direction of the magnetic dipole moment (M) of the magnet

(3) Its direction depends on the magnitude of the magnetic moment (M) of the magnet

(4) In the opposite direction of the magnetic dipole moment(M) of the magnet

Correct Answer : (1)

697. A coolie 1.5 m tall raises a load of 80 kg in 2 s from the ground to his head and then walks
a distance of 40 m in another 2 s. The power developed by the coolie is

(1) 0.2 kW
(2) 0.4 kW
(3) 0.6 kW
(4) 0.8 kW

Correct Answer : (3)

Explanation :
698. A motor pumps out water from a well of depth and fills a water tank of
volume at a height of from the ground. The running time of the motor to fill
the empty water tank is ( )

(1)
(2)
(3)
(4)

Correct Answer : (1)


Explanation : Volume of water to raise

699. A circular loop carrying a current is replaced by an equivalent magnetic dipole. A point on
the axis of the loop is

(1) An end-on position


(2) A broad side-on position

(3) Both [a] and [b]


(4) Neither [a] nor [b]

Correct Answer : (1)

700. The ratio of areas of the electron orbits for the first excited state and the ground state for
the hydrogen atom is

(1) 4:1
(2) 16:1
(3) 8:1
(4) 2:1

Correct Answer : (2)


Explanation : The radius of the orbit of the electron in the th excited state

For the first excited state

For the ground state of hydrogen atom

The ratio of radius

The ratio of area of the electron orbit for hydrogen atom

701. The magnetic field between the plate of a capacitor where is given by (where is the
distance from the axis of plates and is the radius of each plate of capacitor)

(1)

(2)

(3)
(4) Zero

Correct Answer : (3)


Explanation : Using Ampere circuit law,
or
or

702. A railway compartment is lit up by thirteen lamps each taking 2.1 A at 15 V. The heat
generated per second in each lamp will be

(1) 4.35 cal


(2) 5.73 cal
(3) 7.5 cal
(4) 2.5 cal

Correct Answer : (3)


Explanation : From Joule’s law, for a current carrying conductor at a definite temperature the
rate of production of heat is given by

Where is current, the potential difference and the time.


Given,

Also,

703. Emission spectrum of gas

(1) Is a line spectrum


(2) Is a band spectrum

(3) Is a continuous spectrum


(4) Does not fall in the visible region

Correct Answer : (2)

704. In Raman effect, Stokes’ lines are spectral lines having


(1) Frequency greater than that of the original line

(2) Wavelength equal to that of the original line

(3) Wavelength less than that of the original line

(4) Wavelength greater than that of the original line

Correct Answer : (4)


Explanation : In Raman effect, Stokes’ lines are spectral lines having lower frequency or greater
wavelength than that of the original line.

705. The coefficient of performance of a refrigerator work between and is

(1) 28.3
(2) 29.3
(3) 2
(4) Cannot be calculated

Correct Answer : (1)


Explanation :
Coefficient of performance

706. A particle starts from rest, acceleration at for and then goes with constant
speed for and then decelerates at till it stops. What is the distance travelled by it

(1)
(2)
(3)
(4)

Correct Answer : (1)


Explanation : Velocity required by body in

And distance travelled by it in

Then it moves with constant velocity for

After that due to retardation it stops

Total distance travelled

707. The constant of proportionality in Coulomb’s law has the following dimensions

(1)
(2)
(3)
(4)

Correct Answer : (1)

Explanation : According to Coulombs law

708. The current gain of a transistor is 0.9. The transistor is connected to common base
configuration. What would be the change in collector current when base current changes by 4
mA?

(1) 1.2 mA
(2) 12 mA
(3) 24 mA
(4) 36 mA

Correct Answer : (4)


Explanation : The ratio of collector current ) to emitter current ) is known as current gain (
) of a transistor. Therefore,

…(i)
Also, emitter current is equal to sum of change of base current and collector current. Therefore,
…(ii)
From Eqs. (i) and (ii), we get

Given, 0.9,

0.9 =
0.9 (4 + )=
3.6 + 0.9
3.6 = 0.1
mA

709. If a ladder weighing is placed against a smooth vertical wall having coefficient of
friction between it and floor is , then what is the maximum force of friction available at the
point of contact between the ladder and the floor

(1)
(2)
(3)
(4)

Correct Answer : (1)


Explanation :

710. Which of the following statements is correct


(1) Liquids obey fully the law

(2) Liquids obey partially the law

(3) There is no relation between current and p.d. for liquids

(4) None of the above

Correct Answer : (2)


Explanation : In VI graph, we will not get a straight line in case of liquids

711. The potential energy function for the force between two atoms in a diatomic molecule is

approximately given by , where and are constants and is the distance


between the atoms. If the dissociation energy of the molecule is
, is

(1)

(2)

(3)

(4)

Correct Answer : (4)

Explanation :
712. The dimensional formula for Boltzmann’s constant is

(1)
(2)
(3)
(4)

Correct Answer : (1)

Explanation :

713. At room temperature (27 the rms speed of the molecules of a certain diatomic gas is
found to be 1920 m . The gas is

(1)
(2)
(3)
(4)

Correct Answer : (4)

Explanation :

Or
=2 kg = 2g
Since, = 2 for the hydrogen molecule. Hence, the gas is hydrogen.
714. A deflection magnetometer is adjusted in the usual way. When a magnet is introduced, the
deflection observed is , and the period of oscillation of the needle in the magnetometer is .
When the magnet is removed, the period of oscillation is .Find the relation
between is

(1)
(2) cos

(3)

(4)

Correct Answer : (1)


Explanation : In the usual setting of deflecting magnetometer, field due to magnet and
horizontal component of earth’s field are perpendicular to each other. Therefore, the net
field on the magnetic needle is

…(i)
When magnet is removed,

…(ii)

Also,
Dividing Eq. (i) by Eq.(ii), we get

715. The specific heat of an ideal gas varies with temperature as

(1)
(2)
(3)
(4)

Correct Answer : (4)


Explanation : Specific heat of an ideal gas does not depend upon temperature

716. If the momentum of an electron is changed by then the de-Broglie wavelength


associated with it changes by 0.50%. The initial momentum of the electron will be

(1)

(2)
(3)
(4)

Correct Answer : (3)

Explanation :

717. The Rydberg constant for hydrogen is

(1)

(2)

(3)
(4)

Correct Answer : (4)

Explanation :

718. Binding energy per nucleon verses mass number curve for nuclei is shown in the
figure. and are four nuclei indicated on the curve. The process that would release
energy is

(1)
(2)
(3)
(4)

Correct Answer : (3)


Explanation : Energy is released in a process when total binding energy (B.E.) of the nucleus is
increased or we can say when total of products is more than the reactants. By calculation
we can see that only in case of option , this happens
Given
of reactants
and of products
of products > of reactants
719. How does time period of a pendulum very with length?

(1)

(2)

(3)
(4)

Correct Answer : (1)

Explanation : Time period of pendulum

720. The force constant of a wire is and that of another wire is . When both the wires are
stretched through same distance, then the work done

(1)
(2)
(3)
(4)

Correct Answer : (2)

Explanation :
If both wires are stretched through same distance then
. As so

721. The velocity of projection of an oblique projectile is . The speed of the


projectile at the highest point of the trajectory is

(1)
(2)
(3)
(4) Zero

Correct Answer : (1)


Explanation : At the highest point, velocity is horizontal

722. What remains constant in the field of central force

(1) Potential energy


(2) Kinetic energy
(3) Angular momentum
(4) Linear momentum

Correct Answer : (3)


Explanation : In the field central force, Torque = 0 angular momentum remains constant

723. Two blocks of masses kg and kg are connected to the ends of a string which
passes over a massless, frictionless pulley. The total downwards thrust on the pulley is nearly

(1) 27 N
(2) 54 N
(3) 0.8 N
(4) Zero

Correct Answer : (2)

Explanation :

Total downward thrust on the pulley


N
724. A resistance of and a wire of length and resistance are joined in series and
connected to a cell of e.m.f. and internal resistance A parallel combination of two
identical cells is balanced across of the wire. The e.m.f. of each cell is

(1)
(2)
(3)
(4)

Correct Answer : (2)

Explanation :

725. The weight of an object in the coal mine, sea level, at the top of the mountain
are and respectively, then

(1)
(2)
(3)
(4)

Correct Answer : (1)


Explanation : Because value of decreases when we move either in coal mine or at the top of
mountain
726. The ionisation potential of hydrogen atom is . An electron in the ground state of a
hydrogen atoms absorbs a photon of energy How many different spectral line can
one expect when the electron make a downward transition?

(1) 1
(2) 4
(3) 2
(4) 6

Correct Answer : (4)


Explanation :
Energy used for excitation is 12.75 eV

Energy levels of H-atom


The photon of energy 12.75 can excite the fourth level of H-atom
Therefore, six lines will be emitted.

727. With the propagation of a longitudinal wave through a material medium, the quantities
transmitted in the propagation direction are

(1) Energy, momentum and mass


(2) Energy

(3) Energy and mass


(4) Energy and linear momentum

Correct Answer : (4)

728. The electric potential at centre of metallic conducting sphere is

(1) Zero
(2) Half from potential at surface of sphere

(3) Equal from potential at surface of sphere


(4) Twice from potential at surface of sphere

Correct Answer : (3)


Explanation : Inside a charged sphere, electric field intensity at all points is zero and electric
potential is same at all the points.
Electrical potential,

Therefore, potential at the centre is equal to the potential at the surface.

729. In mode, the input characteristics of a transistor is the variation of

(1) against at constant


(2) against at constant

(3) against
(4) against

Correct Answer : (1)


Explanation : Input characteristics of transistor in mode is the curve between base current
and base-emitter voltage at constant collector-emitter voltage .

730. A wheel has angular acceleration of and an initial angular speed


of . In a time of it has rotated through an angle (In radian) of

(1) 6
(2) 10
(3) 12
(4) 4

Correct Answer : (2)


Explanation :

731. The time period of a geostationary satellite is

(1) 12 hours
(2) 24 hours
(3) 6 hours
(4) 48 hours

Correct Answer : (2)

732. The ozone layer of the atmosphere lies in the region called

(1) Troposphere
(2) Stratosphere
(3) Mesosphere
(4) Ionosphere

Correct Answer : (2)

733. The acceleration due to gravity increase by 0.5 % when we go from the equator to the
poles. What will be the time period of the pendulum at the equator which beats seconds at the
poles?

(1) 1.950 s
(2) 1.995 s
(3) 2.050 s
(4) 2.005 s

Correct Answer : (4)

Explanation :

or
Time period at equator

734. In space communication, the information can be passed from one place to another at a
distance of 100 km in

(1) 1 s
(2) 0.5 s
(3) 0.003 s
(4) None of these

Correct Answer : (4)


Explanation : In space communication, the information can be passed from one place to
another with the speed of light . Hence, time taken for a distance

of

735. A solid sphere of radius has moment of inertia about its geometrical axis. If it is melted
into a disc of radius and thickness . If its moment of inertia about the tangential axis (which is
perpendicular to plane of the disc), is also equal to , then the value of is equal to

(1)

(2)

(3)
(4)

Correct Answer : (1)

Explanation :

or

736. Hydrogen atom from excited state comes to the ground state by emitting a photon of
wavelength . If is the Rydberg constant, the principal quantum number of the excited state
is

(1)

(2)

(3)

(4)

Correct Answer : (1)


Explanation : Here, ,

or =1 or

or =
737. Television signals on earth cannot be received at distances greater than from the
transmission station. The reason behind this is that

(1) The receiver antenna is unable to detect the signal at a distance greater than

(2) The TV programme consists of both audio and video signals

(3) The TV signals are less powerful than radio signals

(4) The surface of earth is curved like a sphere

Correct Answer : (4)

738. In the given arrangement, number of equal masses are connected by strings of
negligible masses. The tension in the string connected to th mass is

(1)

(2)
(3)
(4)

Correct Answer : (1)


Explanation : The system may be represented as follow

From the force diagram,


…(i)
and …(ii)
From Eqs. (i) and (ii),
The fore diagram of th block is shown in figure

From the figure,

739. Two masses and are connected by a light inextensible string and
suspended by means of a weightless pulley as shown in figure.

Assuming that both the masses start from rest, the distance travelled by the centre of mass in 2
s is

(1)

(2)

(3)

(4)

Correct Answer : (1)


Explanation :
740. A motor is rotating at a constant angular velocity of 600 rpm. The angular displacement
per second is

(1)

(2)

(3)

(4)

Correct Answer : (4)

Explanation : rad

741. The temperature of hot and cold end of a long rod in thermal steady state are
at and respectively. Temperature at the centre of the rod is

(1)
(2)
(3)
(4)

Correct Answer : (2)

Explanation : Temperature gradient


Temperature of centre
742. In a photoelectric experiment for incident radiation, the potential difference to
stop the ejection is If the incident light is changed to then the potential required to
stop the ejection of electrons will be

(1)
(2) Less than
(3) Zero
(4) Greater than

Correct Answer : (4)


Explanation : According to Einstein’s photoelectric equation

Hence if decreases increases

743. Pick out the unmatched pair from the following

(1) Moderator – Heavy water

(2) Nuclear fuel -

(3) Pressurized water reactor – water as the heat exchange system

(4) Safety rods – Carbon

Correct Answer : (4)


Explanation : Control rods or safety rods used in a nuclear reactor are cadmium rods or boron
rods

744. A coil is suspended in a uniform magnetic field, with the plane of the coil parallel to the
magnetic lines of force. When a current is passed through the coil it starts oscillating; it is very
difficult to stop. But if an aluminium plate is placed near to the coil, it stops. This is due to
(1) Development of air current when the plate is placed

(2) Induction of electrical charge on the plate

(3) Shielding of magnetic lines of force as aluminium is a paramagnetic material

(4) Electromagnetic induction in the aluminium plate giving rise to electromagnetic damping

Correct Answer : (4)

745. For thorium =232, 90 at the end of some radioactive disintegration we obtain an
isotope of lead with 208 and 82, then the number of emitted particles are

(1)
(2)
(3)
(4)

Correct Answer : (3)


Explanation : Given …(i)
Change in mass number

No. of -particles emitted = =6


Now. Eq. (i) becomes

Further change in atomic number is

746. Planck’s constant has the dimensions (unit) of

(1) Energy
(2) Linear momentum
(3) Work
(4) Angular momentum

Correct Answer : (4)


Explanation : Angular momentum] =[

747. Pulling force making an angle to the horizontal is applied on a block of weight placed
on a horizontal table. If the angle of friction is , then the magnitude of force required to move
the body is equal to

(1)

(2)

(3)

(4)

Correct Answer : (3)

748. A TV tower has a height of 100 m. How much population is covered by the TV broadcast if
the average population density around the tower is (radius of the earth
m)

(1) 4 lakh
(2) 4 billion
(3) 40,000
(4) 40 lakh

Correct Answer : (4)


Explanation :
Population covered
population density
lakh
749. Which one of the following processes depends on gravity?

(1) Conduction
(2) Convection
(3) Radiation
(4) None of these

Correct Answer : (2)


Explanation : In convection, the heated lighter particles move upwards and colder heavier
particles move downwards to their place. This depends on weight and hence, on gravity.

750. A magnetic dipole is placed in two perpendicular magnetic fields and and is in
equilibrium taking angle with . Then

(1)
(2)
(3)
(4)

Correct Answer : (3)


Explanation : As is with , therefore according to tangent law,

751. The coil in a tangent galvanometer is 16 cm in radius. If a current of 20 mA is to produce a


deflection of then the number of turns wound on it, is (Take horizontal component of
earth’s magnetic field )

(1) 229
(2) 458
(3) 689
(4) 916
Correct Answer : (2)
Explanation : Current in coil of tangent galvanometer

752. Magnetic moment of bar magnet is M. The work done to turn the magnet by of magnet
in direction of magnetic field B will be

(1) Zero

(2)
(3) 2
(4) MB

Correct Answer : (4)


Explanation : Work done,

753. ‘Torr’ is the unit of

(1) Pressure
(2) Volume
(3) Density
(4) Flux

Correct Answer : (1)

754. Beats are produced by two travelling waves each of loudness I and nearly equal
frequencies and . The beat frequency will be …. and maximum loudness hard will be
(1) 2
(2) 4
(3) 3
(4)

Correct Answer : (2)


Explanation : Beat frequency=number of beats

And maximum loudness

755. Consider an initially pure 3.4 g sample of , an isotope that has a half-life of 78 h. What
is its initial decay rate?

(1)
(2)
(3)
(4)

Correct Answer : (3)


Explanation : Decay rate

Now

and
756. The equivalent resistance between in the given circuit is

(1)
(2)
(3)
(4)

Correct Answer : (4)


Explanation : The given circuit having parallel and series combination of resistance 3 , we can
calculate as

757. can be the unit of measure for

(1) Force
(2) Momentum
(3) Power
(4) Acceleration
Correct Answer : (1)

Explanation :
So can be the unit of force

758. Two gases of equal mass are in thermal equilibrium. If and and are their
respective pressure and volumes, then which relation is true

(1)
(2)

(3)
(4)

Correct Answer : (4)


Explanation : Thermal equilibrium implies that the temperature of gases is same. Hence
Boyle’s law is applicable

759. An object is subjected to a force in the north-east direction. To balance this force, a second
force should be applied in the direction

(1) North-East
(2) South
(3) South-west
(4) West

Correct Answer : (3)


Explanation : Direction of second force should be at

760. The modulation index of a FM signal is 0.25. If modulating frequency is 2kHz, the
maximum derivative in frequency would be
(1) 500 Hz
(2) 1000 Hz
(3) 1500 Hz
(4) 500 kHz

Correct Answer : (1)

Explanation : As,

=500 Hz

761. If a person can throw a stone to maximum height of metre vertically, then the maximum
distance through which it can be thrown horizontally by the same person is

(1)
(2)
(3)
(4)

Correct Answer : (3)

Explanation :
Maximum horizontal distance
762. In the figure shown, three AC voltmeters are connected. At resonance,

(1)
(2)
(3)
(4)

Correct Answer : (1)


Explanation : At resonance, and are equal in magnitude but have phase difference
of relative to each other

Hence, voltmeter read 0 volt

763. A ball dropped from a height of rebounds to a height of after hitting the ground.
Then the percentage of energy lost is

(1) 25
(2) 30
(3) 50
(4) 100

Correct Answer : (1)


Explanation : and

% energy lost
764. Two tangent galvanometer having coils of the same radius are connected in series. A
current flowing in them produces of and 45 respectively. The ratio of the number of turns
in the coil is

(1) 4/3
(2) /1

(3)

(4)

Correct Answer : (4)


Explanation : In series, same current flows through two tangent galvanometer.

765. Consider the statements:


(I)If magnetic field, B =0, then magnetic flux is also zero.
(II)If magnetic flux, then magnetic field is also zero.

(1) (I)is true , (II) may be true


(2) Both (I) and (II) are true

(3) (I) may be true, (II) is true


(4) (I) and (II) both are false

Correct Answer : (1)


Explanation : If B =0 then may or may not be zero
because angle between B and A may be 90
For same part may be positive and for remaining part, it may be negative so that the
resultant becomes zero but B is non-zero.
766. A disc is rolling on the inclined plane, what is the ration of its rotational KE to the total KE?

(1) 1:3
(2) 3:1
(3) 1:2
(4) 2:1

Correct Answer : (1)


Explanation : The rotational kinetic energy of the disc is

The translational kinetic energy is

where is the linear velocity of its centre of mass.


Now,

Therefore,

Thus,

767. Induced emf in the coil depends upon

(1) Conductivity of coil


(2) Amount of flux

(3) Rate of change of linked flux


(4) Resistance of coil

Correct Answer : (3)


Explanation : According to Faraday’s law, “the induced emf in a closed loop equals the time
rate of change of magnetic flux through the loop.”
Hence, induced emf in a coil depends on rate of change of flux.

768. Refractive index of a medium is . The incidence angle is twice that of refracting angle. The
angle of incidence is

(1)

(2)

(3)
(4)

Correct Answer : (3)


Explanation : Angle of incidence (Angle of refraction), [Given]

As

or or

769. Which of the following set up can be used to verify the Ohm’s law?

(1)

(2)
(3)

(4)

Correct Answer : (1)


Explanation : Ammeter is always connected in series and voltmeter in parallel.

770. If , then increase in internal energy when temperature of 2 moles of


this gas is increased from to

(1)
(2)
(3)
(4)

Correct Answer : (2)


Explanation :

771. A police car is travelling in a straight line with a constant speed A truck travelling in the
same direction with constant velocity 3v/2 passes, the police car at The police car starts
acceleration 10 s after passing the truck, at a constant rate of while truck continues to
move at constant speed. If he police car takes 10 s further to catch the truck, find the value of

(1)
(2)
(3)
(4)

Correct Answer : (2)

Explanation :
The diagram is showing the position of car and truck at various instants.

772. On a hilly region, water boils at 95 C.The temperature expressed in Fahrenheit is

(1) 100 F
(2) F
(3) 150 F
(4) 203

Correct Answer : (4)


Explanation : If the temperature of a body on Celsius and Fahrenheit scales are recorded
as C and F respectively, then

or
Here,

Or
773. A lead bullet at just melts when stopped by an obstacle. Assuming that 25% of heat is
absorbed by the obstacle, then the velocity of the bullet at the time of striking (M.P. of
lead , specific heat of lead , latent heat of fusion of
lead and )

(1)
(2)
(3)
(4) None of the above

Correct Answer : (1)


Explanation : If mass of the bullet is ,
Then total heat required for bullet to just melt down

Now when bullet is stopped by the obstacle, the loss in its mechanical energy
(As )
As 25% of this energy is absorbed by the obstacle,
The energy absorbed by the bullet

Now the bullet will melt if

774. The fundamental frequency of a sonameter wire is v. if its radius is doubled and its tension
becomes half, the material of the wire remains same, the new fundamental frequency will be

(1) V

(2)

(3)

(4)
Correct Answer : (4)
Explanation : Frequency of sonometer wire is given by

Where m is mass of string per unit length, and T is tension in the string.
Also, m=πr2d
R being radius of string per unit length, and T is tension in the string.
So,

Or

Or

Hence,

Or

775. A police car moving at , changes a motorcyclist. The police man sounds his horn at
176 Hz, while both of them move towards a stationary siren of frequency 165 Hz. Calculate the
speed of the motorcycle, if it is given that he does not observe any beats.

(1)
(2)
(3) Zero
(4)

Correct Answer : (2)


Explanation : The motorcyclist observes no beats. So the apparent frequency observed by him
from the two sources must be equal.

Solving this equation we get,

776. The equivalent resistance of the following infinite network of resistance is

(1) Less than


(2)

(3) More than but less than


(4)

Correct Answer : (3)


Explanation : Let the resultant resistance be If we add one more branch, then the resultant
resistance would be the same because this is an infinite sequence

or
cannot be negative, hence

777. What is the self-inductance of a coil which produces 5 V when the current changes from 3
A to 2 A in one millisecond?

(1) 5000 H
(2) 5 mH
(3) 50 H
(4) 5 H

Correct Answer : (2)

Explanation : H = 5 mH

778. When a pressure of 100 atmosphere is applied on a spherical ball, then its volume reduces
to 0.01%. The bulk modulus of the material of the rubber in is

(1)
(2)
(3)
(4)

Correct Answer : (3)

Explanation :

779. The speed of an electron having a wavelength of is

(1)
(2)
(3)
(4)

Correct Answer : (1)

Explanation : By using

780. Formation of covalent bonds in compounds exhibits

(1) Wave nature of electron

(2) Particle nature of electron

(3) Both wave and particle nature of electron

(4) None of the above

Correct Answer : (1)


Explanation : For motion of covalent bonds in compounds exhibits nature of electron.

781. Which is the correct expression for half-life

(1)

(2)

(3)

(4)

Correct Answer : (4)

Explanation :
782. A block of mass is in contact with the cart as shown in the figure

The coefficient of static friction between the block and the cart is . The acceleration of the
cart that will prevent the block from falling satisfies

(1)

(2)

(3)

(4)

Correct Answer : (4)

Explanation :
Here and but

So

783. Two circuits have coefficient of mutual induction of . Average e.m.f. induced in
the secondary by a change of current from 0 to in in the primary will
be

(1)
(2)
(3)
(4)
Correct Answer : (4)

Explanation :

784. The power of heater is 750 W at . What will be its power


at if ?

(1) 400 W
(2) 990 W
(3) 250 W
(4) 1500 W

Correct Answer : (2)


Explanation : and
Now,

So,
or = 990 W

785. A piston fitted in cylindrical pipe is pulled as shown in the figure. A tuning fork is sounded
at open end and loudest sound is heard at open length 13cm, 41 cm and 69 cm, the frequency
of tuning fork if velocity of sound is is

(1) 1250 Hz
(2) 625 Hz
(3) 417 Hz
(4) 715 Hz

Correct Answer : (2)


Explanation : In a closed organ pipe in which length of air-column can be increased or
decreased, the first resonance occurs at and second resonance occurs at
Thus, at first resonance

And a second resonance

Subtracting Eq.(i) from Eq.(ii), we have

Hence, frequency of tuning fork

786. The pointer reading load graph for a spring balance is as given in the figure. The spring
constant is
(1)
(2)
(3)
(4)

Correct Answer : (1)

Explanation : Spring constant Slope of curve

787. Time speed of revolution of a nearest satellite around a planet of radius is . Period of
revolution around another planet, whose radius is but having same density is

(1)
(2)
(3)
(4)

Correct Answer : (1)


Explanation : Time period of satellite which is very near to planet

time period of nearest satellite does not depends upon the radius of planet, it only depends
upon the density of the planet.
In the problem, density is same so time period will be same

788. For ionising an excited hydrogen atom, the energy required (in ) will be

(1) A little less than 13.6


(2) 13.6
(3) More than 13.6
(4) 3.4 or less

Correct Answer : (2)


Explanation : Hydrogen atom normally stays in lowest energy state ( =1), where its energy is

On being ionized its energy becomes zero. Thus, ionization of hydrogen atom is
= energy after ionisation – energy before ionisation
=

789. A charged particle of mass and charge travels in a circular path of radius that is
perpendicular to a magnetic field . The time taken by the particle to complete one revolution
is

(1)

(2)

(3)

(4)

Correct Answer : (2)

790. During an adiabatic process, the cube of the pressure is found to be inversely proportional
to the fourth power of the volume. Then the ratio of specific heats is

(1) 1
(2) 1.33
(3) 1.67
(4) 1.4
Correct Answer : (2)
Explanation : Equation of an adiabatic process is
---(i)

Given,

----(ii)
Comparing Eqs. (i) and (ii), we get

791. A man weighs 80 kg on earth surface. The height above ground where he will weigh 40kg,
is (radius of earth is 6400 km)

(1) 0.31 times


(2) 0.41 times
(3) 0.51 times
(4) 0.61 times

Correct Answer : (2)


Explanation : Weight of body at height above the earth’s surface is

792. For a junction diode the ratio of forward current and reverse current is
[ electronic charge, voltage applied across junction, Boltzmann constant,
temperature in kelvin]

(1)
(2)
(3)
(4)

Correct Answer : (4)


Explanation : Current in junction, is positive; In reverse bias is negative. Then

793. In the previous question, if the direction of is reversed, will be

(1) 20 V
(2) 15 V
(3) 10 V
(4) 5 V

Correct Answer : (2)


Explanation : When the direction of current is reversed, moving from

= 15 volt

794. Figure shows a thermodynamical process on one moles a gas. How does the work done in
the process change with time?

(1) Decrease continuously


(2) Increases continuously

(3) Remains constant


(4) First increase and then decreases
Correct Answer : (2)
Explanation : As work done in process = area under the curve, which increases continuously

795. Molecules of a gas behave like

(1) Inelastic rigid sphere


(2) Perfectly elastic non-rigid sphere

(3) Perfectly elastic rigid sphere


(4) Inelastic non-rigid sphere

Correct Answer : (3)


Explanation : Molecules of ideal gas behaves like perfectly elastic rigid sphere

796. The apparent frequency of the whistle of an engine changes in the ratio 9:8 as the engine
passes a stationary observer. If the velocity of the sound is 340 , then the velocity of the
engine is

(1)
(2)
(3)
(4)

Correct Answer : (1)


Explanation : From doppler’s effect, perceived frequency
797. Two masses and of and respectively are connected with a string passing
over a frictionless pulley fixed at the corner of a table as shown. The coefficient of static friction
of with table is 0.2. The minimum mass of that may be placed on to prevent it from
moving is

(1)
(2)
(3)
(4)

Correct Answer : (1)

Explanation : For limiting condition

798. The minimum phase difference between two simple harmonic oscillations,

, is

(1)

(2)

(3)

(4)
Correct Answer : (2)

Explanation : Here,

Similarly,

Phase difference

799. In bringing an electron towards another electron, the electrostatic potential energy of the
system

(1) Decreases
(2) Increases
(3) Remains same
(4) Becomes zero

Correct Answer : (2)


Explanation : Electrostatic potential energy of system of two electrons

Thus, as decreases, potential energy increases.

800. Four particles of mass 1 kg, 2 kg, 3 kg and 4 kg are placed at the
corners and respectively of a square of edge -axis and edge is taken along -
axis, the co-ordinates of centre of mass in SI is

(1) (1,1)
(2) (5,7)
(3) (0.5,0.7)
(4) None of these

Correct Answer : (3)


Explanation :

Similarly,

801. A body, thrown upwards with some velocity reaches the maximum height of . Another
body with the double the mass thrown up with double the initial velocity will reach a maximum
height of

(1)
(2)
(3)
(4)

Correct Answer : (2)


Explanation : , It body projected with double velocity then maximum height will
become four times

802. A wavelength is produced in air and it travels at a speed of It will be an

(1) Audible wave


(2) Infrasonic wave
(3) Ultrasonic wave
(4) None of the above

Correct Answer : (3)

Explanation :
Wave is ultrasonic

803. A current of ampere flows in a circular area of wire which subtends an angle
of radian at its centre, whose radius is The magnetic induction at the centre is

(1)
(2)
(3)
(4)

Correct Answer : (4)


Explanation : For a loop, magnetic induction at centre,

When loop subtends angle at centre, then

In the given problem,

804. Which of the following statements is correct

(1) Hooke’s law is applicable only within elastic limit

(2) The adiabatic and isothermal elastic constants of a gas are equal

(3) Young’s modulus is dimensionless


(4) Stress multiplied by strain is equal to the stored energy

Correct Answer : (1)


Explanation : In accordance with Hook’s law

805. For free expansion of the gas which of the following is true

(1) and
(2) and

(3) and
(4) and

Correct Answer : (1)

806. A 1000 Hz sound wave in air strikes the surface of a lake and penetrates into water. If
speed of sound in water is ,the frequency and wavelength of waves in water are

(1) 1500 Hz, 1m


(2) 1000 Hz, 1.5m
(3) 1000 Hz, 1m
(4) 1500 Hz, 1.5m

Correct Answer : (2)


Explanation : Frequency remains the same wavelength chances

807. The curve between and is ( is the angular momentum and is the linear
momentum)
(1)

(2)

(3)

(4)

Correct Answer : (1)


Explanation :
If graph is drawn between and then it will be straight line which will not pass
through the origin

808. Find

(1)
(2) 20
(3) 30
(4) None of these

Correct Answer : (1)


Explanation : Diode is in forwards biasing hence the circuit can be redrawn as follows

809. Oxygen and hydrogen are at the same temperature . The ratio of the mean kinetic
energy of oxygen molecules to that of the hydrogen molecules will be

(1)
(2)
(3)
(4)

Correct Answer : (2)


Explanation : The mean kinetic energy for gas molecules

So,
According to question both gases are at the same temperature .

So,

810. A point mass is placed inside a thin spherical shell of radius and mass at a
distance from the centre of the shell. The gravitational force exerted by the shell on the
point mass is

(1)
(2)
(3) Zero

(4)

Correct Answer : (3)


Explanation : Gravitational field inside hollow sphere will be zero

811. Which of the following sets have different dimensions?

(1) Pressure, Young’s modulus, Stress


(2) Emf, Potential difference, Electric potential

(3) Heat, Work done, Energy


(4) Dipole moment, Electric flux, Electric field

Correct Answer : (4)


Explanation :

812. The magnetic moment of a magnet of length and pole strength will be

(1) 0.4
(2) 1.6
(3) 20
(4) 8.0

Correct Answer : (1)


Explanation :
813. The half-life for the -decay of uranium is yr. If a rock contains sixty
percent of its original atoms, its age is

(1)
(2)
(3)
(4)

Correct Answer : (1)


Explanation : Here,

814. Two tuning fork, A and B produce notes of frequencies 258 Hz and 262 Hz. An unknown
note sounded with a produces certain beats. When the same note is sounded with B, the beat
frequency gets doubled, the unknown frequency is

(1) 256 Hz
(2) 254 Hz
(3) 300 Hz
(4) 280 Hz

Correct Answer : (2)


Explanation :

Let n is the frequency of unknown tuning fork. It produces x beats with 258 and 2x with 262

815. The basic magnetization curve for a ferromagnetic material is shown in figure. Then, the
value of relative permeability is highest for the point
(1)
(2)
(3)
(4)

Correct Answer : (2)

Explanation : slope of curve


According to the given graph, slope of the graph is highest at point

816. An alpha nucleus of energy bombards a heavy nuclear target of charge . Then the
distance of closest approach for the alpha nucleus will be proportional to

(1)
(2) 1/m
(3)
(4) 1/

Correct Answer : (2)


Explanation : At distance of closest approach relative velocity of two particles is . Here target
is considered as stationary, so -particle comes to rest instantaneously at distance of closest
approach. Let required distance is then from work energy-theorem.
817. The energy required to accelerate a car from to is how many times the
energy required to accelerate the car from rest to

(1) Equal
(2) 4 times
(3) 2 times
(4) 3 times

Correct Answer : (4)


Explanation : Kinetic energy for first condition

K.E. for second condition

818. A circular ring of radius is placed in a homogenous magnetic field perpendicular to the
plane of the ring. The field chnages with time according to the equation , where is a
constant and is the time. The electric field in the ring is

(1)

(2)

(3)

(4)

Correct Answer : (3)


Explanation :

or

or

819. of work is required for

(1) Increasing the temperature of of water through

(2) Increasing the temperature of of water through

(3) Increasing the temperature of of water through

(4) Increasing the temperature of of water through

Correct Answer : (2)


Explanation : Work done to raise the temperature of water through is

820. Modulation is the phenomenon of

(1) Superimposing the audio frequency signal over a carrier wave

(2) Separating the audio frequency signal from the carrier wave

(3) Separating carrier wave from the modulated wave

(4) Any of [a], [b], [c] above

Correct Answer : (1)


Explanation : In modulation, audio frequency signal is superimposed on radio frequency
carrier wave

821. A uniform electric field and a uniform magnetic field are acting along the same direction in
a certain region. If an electron is projected along the direction of the fields with a certain
velocity, then

(1) Its velocity will decrease

(2) Its velocity will increase

(3) It will turn towards right of direction of motion

(4) It will turns towards left of direction of motion

Correct Answer : (1)


Explanation : When E, v and B are all along same direction, then magnetic force experienced
by electron is zero while electric force is acting opposite to velocity of electron, so velocity of
electron will decrease.

822. -type semiconductor is formed when


A. impurity is mixed in
B. impurity is mixed in
C. impurity is mixed in
D. impurity is mixed in

(1) A and C
(2) A and D
(3) B and C
(4) B and D

Correct Answer : (3)


Explanation : The resistance of semiconductor decreases with the increase in temperature

823. The velocity of a freely falling body changes as where is acceleration due to gravity
and is the height. The values of and are

(1)
(2)

(3)
(4) 1, 1

Correct Answer : (2)


Explanation : (given)
By submitting the dimension of each quantity and comparing the powers on both sides we
get

824. An electric field of intensity is applied perpendicular to the direction of


motion of the electron. A magnetic field of induction is applied perpendicular
to both the electric field and direction of motion of the electron. What is the velocity of the
electron if it passes undeflected?

(1)
(2)
(3)
(4) It is never possible

Correct Answer : (1)


Explanation :

825. The distance of a geo-stationary satellite from the centre the earth (Radius is
nearest to

(1)
(2)
(3)
(4)
Correct Answer : (2)
Explanation : from the surface of earth and from the centre

826. The force acting on a particle moving in a straight line is shown in figure. What is the
work done by the force on the particle in the 1st meter of the trajectory

(1)
(2)
(3)
(4)

Correct Answer : (4)


Explanation : Work done Area under curve of - graph

= Area of triangle

827. Current in a coil changes from 5 A to 10 A in 0.2 s. If the coefficient of self-induction is 10 H,


then the induced emf is

(1) 112 V
(2) 250 V
(3) 125 V
(4) 230 V

Correct Answer : (2)


Explanation : Induced emf in the coil is given by

Or
828. Power of water pump is . If , the amount of water it can raise in one
minute to a height of is

(1)
(2)
(3)
(4)

Correct Answer : (4)

Explanation :

As volume
Volume

829. A copper rod of mass m slides under gravity on two smooth parallel rails distance apart
and set at an angle to the horizontal. At the bottom, the rails are joined by a resistance R,
figure. There is a uniform magnetic field B perpendicular to the plane of the rails. The terminal
velocity of the rod is

(1)

(2)

(3)
(4)

Correct Answer : (3)


Explanation : Terminal velocity of the rod is attained when magnetic force on the
rod balances the component of weight of the rod .

830. The coefficiency of apparent expansion of a liquid when determined using two different
vessels and respectively. If the coefficient of linear expansion of the
vessel the coefficient of linear expansion of vassel is

(1)

(2)

(3)

(4)

Correct Answer : (4)


Explanation : Using g, we get

831. Three capacitors of capacitance are connected in parallel to which a capacitor of


capacitance is connected in series. Effective capacitance is 3.75, then capacity of each
capacitor is

(1) 4
(2) 5
(3) 6
(4) 8

Correct Answer : (2)


Explanation : The effective capacitance of three capacitor connected in parallel=
When is connected in series to

832. A bullet of mass hits a target of mass hanging by a string and gets embedded in it. If
the block rises to a height as a result of this collision, the velocity of the bullet before collision
is

(1)

(2)

(3)

(4)
Correct Answer : (3)
Explanation : If initial velocity of bullet be then after collision combined velocity of bullet and
target is

and or

833. A thin rod of length lies along the axis of a concave mirror of focal length . One end of
its magnified image touches an end of the rod. The length of the image is

(1)

(2)
(3)

(4)

Correct Answer : (2)


Explanation : If end of rod acts an object for mirror then it’s image will be and if

So by using

Length of image
834. Four metal conductors having different shapes
1] A sphere
2] Cylinder
3] Pear
4] Lightning conductor
are mounted on insulating stands and charged. The one which is best suited to retain the
charges for a longer time is

(1) 1
(2) 2
(3) 3
(4) 4

Correct Answer : (1)


Explanation : The sphere will retain the charge for longer time, because in case of spherical
metal conductor, the charge quickly spreads uniformly over the entire surface.

835. Number of electric lines of force from 0.5 C if positive charge in a dielectric medium of
constant 10 is

(1)
(2)
(3)
(4)

Correct Answer : (1)

Explanation :

836. If and represent the work done in moving a particle from to along three
different paths 1, 2 and 3 respectively (as shown) in the gravitational field of a point mass m,
find the correct relation between and
(1)
(2)
(3)
(4)

Correct Answer : (2)


Explanation : Gravitational force is a conservative force and work done against it is a point
function does not depend on the path

837. If , where is the distance travelled by the body in kilometre while is the time
in second, then the units of are

(1)
(2)
(3)
(4)

Correct Answer : (3)


Explanation :

838. A block of mass reacts on a horizontal table. A string tied to the block is passed on a
frictionless pulley fixed at the end of the table and to the other end of string is hung another
block of mass . The acceleration of the system is

(1)
(2)
(3)

(4)

Correct Answer : (1)

839. A particle moves along a semicircle of radius in 5 seconds. The average velocity
of the particle is

(1)
(2)
(3)
(4)

Correct Answer : (4)

Explanation :

840. Masses of the three wires of same material are in the ratio of and their lengths in the
ratio of . Electrical resistance of these wires will be in the ratio of

(1)
(2)
(3)
(4)

Correct Answer : (4)


Explanation : Mass,
or
Resistance
So

Thus,

841. A simple pendulum is set up in a trolley which moves to the right with an acceleration on
a horizontal plane. Then the thread of the pendulum in the mean position makes an
angle with the vertical

(1) in the forward direction

(2) in the backward direction

(3) in the backward direction

(4) in the forward direction

Correct Answer : (2)


Explanation : In accelerated frame of reference, a fictitious force (pseudo forc acts on the
bob of pendulum as shown in figure

Hence,

in the backward direction


842. The length of a half wave dipole at 30 MHz is

(1) 10 m
(2) 50 m
(3) 5 m
(4) 100 m

Correct Answer : (3)

Explanation :

843. A ball is projected with kinetic energy at an angle of to the horizontal. At the highest
point during its flight, its kinetic energy will be

(1)
(2)
(3)
(4) Zero

Correct Answer : (3)

Explanation : Kinetic energy at highest point

844. X-rays are produced by jumping of

(1) Electrons from lower to higher energy orbit of atom


(2) Electrons from higher to lower energy orbit of atom

(3) Protons from lower to higher energy orbit of nucleus


(4) Proton from higher to lower energy orbit of nucleus

Correct Answer : (2)


845. Wires and are made from the same material. A has twice the diameter and three times
the length of . If the elastic limits are not reached, when each is stretched by the same
tension, the ratio of energy stored in to that in is

(1) 2 : 3
(2) 3 : 4
(3) 3 : 2
(4) 6 : 1

Correct Answer : (2)

Explanation : and are constant]

846. Consider the following statements. A and B and identify the correct answer given below.
1.Body initially at rest is acted upon by a constant force. The rate of change of its kinetic energy
varies linearly with time.
2.When a body is at rest, it must be in equilibrium.

(1) A and B are correct


(2) A and B are wrong

(3) A is correct and B is wrong


(4) A is wrong and B is correct

Correct Answer : (3)

Explanation : KE
Rate of change of KE,

So, statement is correct.


When the body is at rest then it may be or may not be in equilibrium, so statement is wrong.
847. The impedance of a circuit consists of resistance and reactance. The power
factor of the circuit is

(1) 0.4
(2) 0.6
(3) 0.8
(4) 1.0

Correct Answer : (2)


Explanation :

848. A divergent lens will produce

(1) Always a virtual image


(2) Always real image

(3) Sometimes real and sometimes virtual


(4) None of the above

Correct Answer : (1)


Explanation : A concave lens always from virtual image for real objects

849. A wire carrying current and other carrying in the same direction produces a magnetic
field at the mid point. What will be the field when wire is switched off

(1)
(2)
(3)
(4)

Correct Answer : (3)


Explanation : =

850. A projectile shot into air at some angle with the horizontal has a range of 200 m. If the time
of flight is 5 s, then the horizontal component of the velocity of the projectile at the highest
point of trajectory is

(1)
(2)

(3)
(4) Equal to the velocity of projection of the projectile

Correct Answer : (1)

Explanation :

Dividing,
or
It may be noted here that the horizontal component of the velocity of projection remains the
same during the flight of the projectile

851. The unit of percentage error is

(1) Same as that of physical quantity

(2) Different from that of physical quantity

(3) Percentage error is unit less


(4) Errors have got their own units which are different from that of physical quantity measured

Correct Answer : (3)

852. When a battery is connected across a series combination of self inductance and
Resistance the variation in the current with time is best represented by

(1)

(2)

(3)

(4)

Correct Answer : (2)

Explanation : At
At

853. An ideal monoatomic gas is taken around the cycle as shown in versus diagram.
Work done during the cycle is
(1)
(2)
(3)
(4)

Correct Answer : (1)


Explanation : Work done during the complete cycle= area

854. A count rate metre shows a count of 240 per minute from a given radioactive source later

the metre shows a count rate 0f . The half-life of the source is

(1) 80 min
(2) 120 min
(3) 20 min
(4) 30 min

Correct Answer : (3)


Explanation : The number of counts left after time t

or

or
Comparing the powers, we get
= 20 min

855. The correctness of velocity of an electron moving with velocity is 0.005%. The
accuracy with which its position can be measured will be

(1)
(2)
(3)
(4)

Correct Answer : (2)

Explanation : Here,

856. During adiabatic expansion of 10 moles of a gas, the internal energy decreases by 50 J.
Work done during the process is

(1) +50 J
(2) J
(3) Zero
(4) Cannot say

Correct Answer : (1)


Explanation : In adiabatic expansion, ,

857. The rate of the mass of the gas emitted from rear of a rocket is initially 0.1 . If the
speed of the gas relative to the rocket is and mass of the rocket is 2 kg, then the
acceleration of the rocket (in is

(1) 5
(2) 5.2
(3) 2.5
(4) 25

Correct Answer : (3)

Explanation :

858. The moment of inertia about an axis of a body which is rotating with angular velocity 1
is numerically equal to

(1) One-fourth of its rotational kinetic energy


(2) Half of the rotational kinetic energy

(3) Rotational kinetic energy


(4) Twice the rotational kinetic energy

Correct Answer : (4)


Explanation : Rotational kinetic energy,

Here,

or
, moment of inertia about an axis of a body is twice the rotational kinetic energy.

859. are capacitance, potential difference, energy stored and charge of a parallel
plate capacitor respectively. The quantities that increase when a dielectric slab is introduced
between the plates without disconnecting the battery are
(1)
(2)
(3)
(4)

Correct Answer : (3)


Explanation : When battery remains connected

Both increases.

860. Two identical bar magnets with a length and weight - are arranged freely
with their poles facing in a inverted vertical glass tube. The upper magnet hangs in the air
above the lower one so that the distance between the nearest pole of the magnet is . Pole
strength of the poles of each magnet will be

(1)
(2)
(3)
(4) None of these

Correct Answer : (1)


Explanation : The weight of upper magnet should be balanced by the repulsion between the
two magnets
861. A capacitor of capacity has charge and stored energy is If the charge is increased
to the stored energy will be

(1)
(2)
(3)
(4)

Correct Answer : (3)

Explanation :

862. A horizontal rod of mass and length is placed on a smooth plane inclined at
an angle of with the horizontal, with the length of the rod parallel to the edge of the inclined
plane. A uniform magnetic field of induction is applied vertically downwards. If the current
through the rod is , then the value of for which the rod remains stationary on the
inclined plane is

(1) 1.73

(2)
(3) 1
(4) None of the above

Correct Answer : (3)


Explanation : The given situation can be drawn as follows

863. of gas is compressed at 1 atm pressure and temperature What is the


pressure under adiabatic condition?

(1)
(2)
(3)
(4)

Correct Answer : (1)

Explanation : For adiabatic change,

or

864. Consider the following two statements A and B and identify the correct choice in the given
answers
A: Line spectra is due to atoms in gaseous state
B: Band spectra is due to molecules
(1) Both A and B are false
(2) A is true and B is false

(3) A is false and B is true


(4) Both A and B are true

Correct Answer : (4)


Explanation : Line and band spectrum are also known as atomic and molecular spectra
respectively

865. Two rectangular blocks and of masses 2kg and 3 kg respectively are connected by
spring of spring constant 10.8 and are placed on a frictionless horizontal surface. The
block was given an initial velocity of 0.15 in the direction shown in the figure. The
maximum compression of the spring during the motion is

(1) 0.01 m
(2) 0.02 m
(3) 0.05 m
(4) 0.03 m

Correct Answer : (3)


Explanation : As the block A moves with velocity with velocity 0.15 , it compresses the
spring Which pushes B towards right. A goes on compressing the spring till the velocity
acquired by B becomes equal to the velocity of A, i.e. 0.15 . Let this velocity be v. Now,
spring is in a state of maximum compression. Let x be the maximum compression at this stage.

According to the law of conservation of linear momentum, we get

Or

According to the law of conservation of energy


Or

866. The ratio of minimum to maximum wavelength in Balmer series is

(1) 5 : 9
(2) 5 : 36
(3) 1 : 4
(4) 3 : 4

Correct Answer : (1)

Explanation :

867. A particle is acted upon by a force of constant magnitude which is always perpendicular to
the velocity of the particle. The motion of the particle takes place in a plane it follows that

(1) Its velocity is constant


(2) Its acceleration is constant

(3) Its kinetic energy is constant


(4) It moves in a straight line
Correct Answer : (3)
Explanation : When a force of constant magnitude acts on velocity of particle perpendicularly,
then there is no change in the kinetic energy of particle.
Hence, kinetic energy remains constant.

868. A transformer is having 2100 turns in primary and 4200 turns in secondary. An AC source
of 120 V, 10 A is connected to its primary. The secondary voltage and current are

(1) 240 V,5 A


(2) 120 V, 10 A
(3) 240 V, 10 A
(4) 120 V, 20 A

Correct Answer : (1)

Explanation : V

869. Three samples of the same gas and have initially equal volume. Now the
volume of each sample is doubled. The process is adiabatic for isobaric for and isothermal
for . If the final pressure are equal for all three samples, the ratio of their initial pressures are

(1)
(2)
(3)
(4)

Correct Answer : (2)


Explanation : Let the initial pressure of the three samples be and ,
then and
870. The unit of mutual conductance of a triode valve is

(1) Siemen
(2)
(3)
(4)

Correct Answer : (1)

871. A stone is hung in air from a wire which is stretched over a sonometer. The bridges of the
sonometer are apart when the wire is in unison with a tuning fork of frequency When
the stone is completely immersed in water, the length between the bridges is for re-
establishing unison, the specific gravity of the material of the stone is

(1)

(2)

(3)

(4)

Correct Answer : (3)

Explanation : Frequency of vib. in stretched string


When the stone is completely immersed in water, length changes but frequency doesn’t (
unison re-establishe

Hence length
[Density of stone and density of water ]
872. If a current given by flows in an ac circuit across, which an ac potential
of has been applied, then the power consumption in the circuit will be

(1)
(2)

(3)
(4)

Correct Answer : (4)


Explanation : Phase angle so power

873. The speed of electromagnetic Wave in vacuum depends upon the source radiation. It

(1) Increases as we move from to radio waves

(2) Decreases as we move from to radio waves

(3) Is same for all of them

(4) None of the above

Correct Answer : (3)


Explanation : Speed of Electromagnetic Waves in vacuum

874. A particle free to move along the -axis has potential energy given
by for , where is a positive constant of appropriate
dimensions. Then
(1) At point away from the origin, the particle is in unstable equilibrium

(2) For any finite non-zero value of , there is a force directed away from the origin

(3) If its total mechanical energy is , it has its minimum kinetic energy at the origin

(4) For small displacements from , the motion is simple harmonic

Correct Answer : (4)


Explanation : Potential energy of the particle

Force on particle

For small displacement


motion is simple harmonic motion

875. The equation represents a wave with

(1) Amplitude frequency and wavelength

(2) Amplitude frequency and wavelength

(3) Amplitude frequency and wavelength

(4) Amplitude frequency and wavelength

Correct Answer : (1)


Explanation : The given equation can be written as

Hence amplitude and frequency

And wave length


876. Four pieces of iron heated in a furnace to different temperatures show different colours
listed below. Which one has the highest temperature

(1) White
(2) Yellow
(3) Orange
(4) Red

Correct Answer : (1)


Explanation : At low temperature short wavelength radiation is emitted. As the temperature
rises colour of emitted radiations are in the following order
Red Yellow Blue White (at highest temperature)

877. The S.I. unit of gravitational potential is

(1)
(2)
(3)
(4)

Correct Answer : (2)

Explanation : so, SI unit =

878. Choke coil works on the principle of

(1) Transient current


(2) Self induction
(3) Mutual induction
(4) Wattless current

Correct Answer : (2)


879. If the angular momentum of an electron is then the magnitude of the magnetic moment
will be

(1)

(2)
(3)

(4)

Correct Answer : (2)


Explanation : As we know for circulating electron magnetic moment

And angular momentum …(ii)

From equation (i) and (ii),

880. A stone falls from a balloon that is descending at a uniform rate of . The
displacement of the stone from the point of release after 10s is

(1) 725 m
(2) 610 m
(3) 510 m
(4) 490 m

Correct Answer : (2)


Explanation :

m
m=610m

881. At a certain distance from a point charge the electric field is and the potential
is . What is this distance
(1)
(2)
(3)
(4)

Correct Answer : (1)

Explanation :

882. The dimensions of electric dipole moment are

(1)
(2)
(3)
(4)

Correct Answer : (3)


Explanation : Electric dipole moment

The dimensions of electric dipole moment is

883. Large transformers, when used for some time, become hot and are cooled by circulating
oil. The heating of transformer is due to

(1) Heating effect of current alone


(2) Hysteresis loss alone

(3) Both the hysteresis loss and heating effect of current


(4) None of the above
Correct Answer : (3)

884. Three rods of the same dimension have thermal conductivities and . They are
arranged as shown in fig. Given below, with their ends at and . The
temperature of their junction is

(1)
(2)
(3)
(4)

Correct Answer : (2)


Explanation : Let the temperature of junction be then according to the following figure

885. The volume of a nucleus is directly proportional to

(1)
(2)

(3)
(4)
(where =mass number of the nucleus)

Correct Answer : (1)


Explanation : Radius of nucleus
Where

Volume of nucleus =

886. The amplitude of two waves are in ratio 5:2. If all other conditions for the two waves
Are same, then what is the ratio of their energy densities?

(1) 5:2
(2) 5:4
(3) 4:5
(4) 25:4

Correct Answer : (4)


Explanation : Energy density of wave is given by

Or

So,

887. The volume of a metal sphere increases by 0.24% when its temperature is raised by 40 .
The coefficient of linear expansion of the metal is … .

(1)
(2)
(3)
(4)

Correct Answer : (1)


Explanation : Change in volume,

888. Broadcasting antennas are generally

(1) Omnidirectional type


(2) Vertical type
(3) Horizontal type
(4) None of these

Correct Answer : (2)

889. Distribution of energy in the spectrum of a black body can be correctly represented by

(1) Wien’s law


(2) Stefan’s law
(3) Planck’s law
(4) Kirchhoff’s law

Correct Answer : (3)


Explanation : Because Planck’s law explains the distribution of energy correctly at low
temperature as well as at high temperature
890. A straight wire of length is carrying a current of and the magnetic field due
to it is measured at a point distance from it. If the wire is to be bent into a circle and is to
carry the same current as before, the ratio of the magnetic field at its centre to that obtained in
the first case would be

(1)
(2)
(3)
(4)

Correct Answer : (2)


Explanation : If a wire of length is bent in the form of a circle of radius then

Magnetic field due to straight wire also magnetic field due to circular
loop

891. The ratio of minimum wavelength of Lyman and Balmer series will be

(1) 10
(2) 5
(3) 0.25
(4) 1.25

Correct Answer : (3)


Explanation :

0.25

892. The emf of a generator is 6 V and internal resistance is 0.5 . The reading of a voltmeter
having an internal resistance of 2.5 is

(1)
(2)
(3)
(4)

Correct Answer : (3)


Explanation : Total resistant

Current, ;
Reading of voltmeter

893. A point source of Electromagnetic radiation has an average power output of 1500 W. The
maximum value of electric field at a distance of 3 m from this source in Vm-1- is

(1) 500
(2) 100

(3)

(4)

Correct Answer : (1)

894. A wire of Young’s modulus is stretched by a force so as to produce a


strain of The energy stored per unit volume is
(1)
(2)
(3)
(4)

Correct Answer : (4)


Explanation : Energy stored per unit volume

895. The working principle of the mass spectrograph is that for a given combination of
accelerating potential and magnetic field, the ion beam (with charge and mass ) to be
collected at different positions of ion collectors will depend upon the value of

(1)
(2)
(3)
(4)

Correct Answer : (3)

Explanation : In mass spectrograph,

and or

or
so is related with

896. Kepler’s second law regarding constancy of aerial velocity of a planet is consequence of the
law of conservation of

(1) Energy
(2) Angular momentum
(3) Linear momentum
(4) None of these

Correct Answer : (2)

Explanation : constant

897. The Kinetic energy of the electron in an orbit of radius in hydrogen atom is (
electronic charge)

(1)

(2)

(3)

(4)

Correct Answer : (2)

Explanation :

898. For ordinary terrestrial experiments, the observer is an inertial frame in the following
cases is

(1) A child revolving in a giant wheel

(2) A driver in a sports car moving with a constant high speed of on a straight rod

(3) The pilot of an aeroplane which is taking off


(4) A cyclist negotiating a sharp curve

Correct Answer : (2)

899. The fact that radiosignals reach the earth from outside the atmosphere, was discovered
accidently by

(1) K. G. Jansky
(2) Millikan
(3) Aryabhatta
(4) Prof. Kanu

Correct Answer : (1)


Explanation : K. G. Jansky discovered accidently the radio signals coming from outside the
atmosphere and reaching the earth

900. The lines of force due to earth’s horizontal component of magnetic field are

(1) Parallel straight lines


(2) Concentric circles
(3) Elliptical
(4) Parabolic

Correct Answer : (1)

You might also like